Set One: 32: Hypertension, 34: Heart Failure, 35: Dysrhythmias

Ace your homework & exams now with Quizwiz!

normal sinus rhythm detail

- 60-100 bpm - atrial and ventricular rates are regular - PR interval - 0.12-0.20 seconds - QRS interval - <0.12 seconds

sinus bradycardia details

- <60 bpm - atrial and ventricular rates are regular - PR interval - 0.12-0.20 seconds - QRS interval - <0.12 seconds

sinus tachycardia details

- >100 bpm - atria and ventricular rate is regular - PR interval is 0.12-0.20 seconds - QRS interval is <0.12 seconds

causes for premature atrial contraction (PAC)

- AMI - CMF - anxiety - cigarette smoking - caffeine or alcohol intake - hypervolemia - hypokalemia - treatment usually not required

treatment for pulseless electrical activity

- CPR - epinephrine - identify possible causes (H&T)

HCTZ (hydrochlorothiazide)

Very mild thiazides Work by itself, works a lot better when combined with Lasix

always pulsesless

Vfib

The nurse is doing discharge teaching with the patient and spouse of the patient who just received an implantable cardioverter-defibrillator (ICD) in the left side. Which statement by the patient indicates to the nurse that the patient needs more teaching? "I will call the cardiologist if my ICD fires." "I cannot fly because it will damage the ICD." "I cannot move my left arm until it is approved." "I cannot drive until my cardiologist says it is okay."

"I cannot fly because it will damage the ICD." The patient statement that flying will damage the ICD indicates misunderstanding about flying. The patient should be taught that informing TSA about the ICD can be done because it may set off the metal detector and if a hand-held screening wand is used, it should not be placed directly over the ICD. The other options indicate the patient understands the teaching.

At a clinic visit, the nurse provides dietary teaching for a patient recently hospitalized with an exacerbation of chronic heart failure. The nurse determines that teaching is successful if the patient makes which statement? "I will limit the amount of milk and cheese in my diet." "I can add salt when cooking foods but not at the table." "I will take an extra diuretic pill when I eat a lot of salt." "I can have unlimited amounts of foods labeled as reduced sodium."

"I will limit the amount of milk and cheese in my diet." Milk products should be limited to 2 cups per day for a 2500-mg sodium-restricted diet. Salt should not be added during food preparation or at the table. Diuretics should be taken as prescribed (usually daily) and not based on sodium intake. Foods labeled as reduced sodium contain at least 25% less sodium than regular.

At a clinic visit, the nurse provides dietary teaching for a patient recently hospitalized with an exacerbation of chronic heart failure. The nurse determines that teaching is successful if the patient makes which statement? "I will limit the amount of milk and cheese in my diet." "I can add salt when cooking foods but not at the table." "I will take an extra diuretic pill when I eat a lot of salt." "I can have unlimited amounts of foods labeled as reduced sodium."

"I will limit the amount of milk and cheese in my diet." Milk products should be limited to 2 cups per day for a 2500-mg sodium-restricted diet. Salt should not be added during food preparation or at the table. Diuretics should be taken as prescribed (usually daily) and not based on sodium intake. Foods labeled as reduced sodium contain at least 25% less sodium than regular.

A nurse is performing client health education with a 68-year-old man who has recently been diagnosed with heart failure. Which of the following statements demonstrates an accurate understanding of his new diagnosis?

"I'm trying to think of ways that I can cut down the amount of salt that I usually eat."

A client with heart failure asks, "Why am I taking a 'water pill9 when it's my heart that is having a problem?" While educating the client about the Frank-Starling mechanism, which of the following explanations is most appropriate to share?

"Since your heart is not pumping efficiently, the kidneys are getting less blood flow; therefore, the kidneys are holding on to sodium and water."

A patient in the nursing home wonders why he is having these signs and symptoms of left-sided failure. Which of the following explanations will the nurse give the patient?

"The left ventricle is having problems pumping blood forward, and this is causing blood to back up into your lungs."

An older adult patient with chronic heart failure (HF) and atrial fibrillation asks the nurse why warfarin has been prescribed to continue at home. What is the best response by the nurse? "The medication prevents blood clots from forming in your heart." "The medication dissolves clots that develop in your coronary arteries." "The medication reduces clotting by decreasing serum potassium levels." "The medication increases your heart rate so that clots do not form in your heart."

"The medication prevents blood clots from forming in your heart." Chronic HF causes enlargement of the chambers of the heart and an altered electrical pathway, especially in the atria. When numerous sites in the atria fire spontaneously and rapidly, atrial fibrillation occurs. Atrial fibrillation promotes thrombus formation within the atria with an increased risk of stroke and requires treatment with cardioversion, antidysrhythmics, and/or anticoagulants. Warfarin is an anticoagulant that interferes with hepatic synthesis of vitamin K-dependent clotting factors.

An older adult patient with chronic heart failure (HF) and atrial fibrillation asks the nurse why warfarin has been prescribed to continue at home. What is the best response by the nurse? "The medication prevents blood clots from forming in your heart." "The medication dissolves clots that develop in your coronary arteries." "The medication reduces clotting by decreasing serum potassium levels." "The medication increases your heart rate so that clots do not form in your heart."

"The medication prevents blood clots from forming in your heart." Chronic HF causes enlargement of the chambers of the heart and an altered electrical pathway, especially in the atria. When numerous sites in the atria fire spontaneously and rapidly, atrial fibrillation occurs. Atrial fibrillation promotes thrombus formation within the atria with an increased risk of stroke and requires treatment with cardioversion, antidysrhythmics, and/or anticoagulants. Warfarin is an anticoagulant that interferes with hepatic synthesis of vitamin K-dependent clotting factors.

A client awaiting a heart transplant is experiencing decompensation of her left ventricle that will not respond to medications. The physicians suggest placing the client on a ventricular assist device (VAD). The client asks what this equipment will do. The health care providers respond:

"This device will decrease the workload of the myocardium while maintaining cardiac output and systemic arterial pressure."

The client is prescribed digoxin (Lanoxin) for treatment of HR. Which of the following statements by the client indicates the need for further teaching by the nurse? "I should not get short of breath anymore." "This drug will help my heart muscle pump less." "I may notice my heart rate decrease." "I may feel tired during early treatment."

"This drug will help my heart muscle pump less." Rationale: The ability to increase the strength of contractions is a characteristic of cardiac glycosides. It may result in a decrease in pulse. Initially the client may experience some fatigue. Symptoms of CHF, such as dyspnea, should improve.

***

***

atrial flutter

- 250-350 atrial beats / 100 ventricular beats - PR not measurable - sawtooth P waves - normal QRS

What are the physiologic signs and symptoms of cardiogenic shock? (Select all that apply.)

- Rise in central venous pressure (CVP) - Increased extraction of O2 from hemoglobin - Decrease in mean arterial blood pressures

The sympathetic (adrenergic) nervous system is an important compensatory mechanism in heart failure. Which of the following statements regarding the sympathetic nervous system response to heart failure are correct? Select all that apply.

- The sympathetic nervous system responds rapidly to a fall in cardiac output. - The sympathetic nervous system increase in cardiac workload and oxygen use can cause ischemia and worsening of heart failure. - The sympathetic nervous system increase in heart rate and force of contraction support cardiac output.

treatment for sinus tachycardia

- alleviate underlying cause - administer IV fluids for hypovolemia - administer antipyretics for fever - synchronized cardioversion for hemodynamic instability

atrial flutter details

- atrial - regular rate, 250-400 bpm - ventricular - R-R intervals are regular or irregular based on fixed or variable block, 60-150 bpm - P waves are not discernible, has "sawtooth" appearance - PR interval - not measureable - QRS interval - <0.12 seconds

premature atrial contraction (PAC) details

- atrial and ventricular rates are regular but shortened - P wave is premature - PR interval is 0.12-0.20 seconds - QRS interval is <0.12 seconds

asystole details

- atrial rate - absent - ventricular rate - absent - P waves - absent - QRS - absent - PR and QRS intervals - absent

ventricular fibrillation details

- atrial rate - absent - ventricular rate - absent - P waves - absent - QRS - absent - PR interval and QRS interval - absent

premature ventricular contraction details

- atrial rate - absent/dissociated - ventricular rate - R-R is shorter with ectopic (premature or irregular) beat - P wave - absent/dissociated from the ectopic beat - QRS - widened or bizarre - PR interval - absent - QRS interval - ≥0.12 seconds

atrial fibrillation details

- atrial rate - no P-P interval, rate cannot be measured - ventricular rate - R-R intervals are irregular, rate is variable - no P wave - PR interval cannot be measured - QRS interval - <0.12 seconds

polymorphic ventricular tachycardia details

- atrial rate - obscured - ventricular rate - irregular and chaotic, 250-350 bpm - P wave - obscured - QRS - variable, wide and bizarre, not identical - PR interval - absent - QRS interval - ≥0.12 seconds

monomorphic ventricular tachycardia details

- atrial rate - obscured - ventricular rate - regular, R-R is regular, >150 bpm - P waves - obscured - PR interval - absent - QRS interval - ≥0.12 seconds

supraventricular tachycardia details

- atrial rate - regular, P-P interval is regular if it can be identified, 150-250 bpm - ventricular rate - regular, R-R is regular, 150-250 bpm - P waves - difficult to detect or hidden because of fast HR - PR intervals - not measureable - QRS intervals - <0.12 seconds

second-degree (type 1) block details

- atrial rate - regular, P-P is regular, 60-100 bpm - ventricular rate - R-R intervals decrease progressively until QRS drops, rate is variable - P wave - more P waves than QRS complexes - QRS - dropped in a cyclic pattern - PR intervals - increases with each beat - QRS - <0.12 seconds

first-degree block details

- atrial rate - regular, P-P is regular, 60-100 bpm - ventricular rate - regular, R-R is regular, 60-100 bpm - P wave - before every QRS - PR interval - >0.20 seconds - QRS interval - <0.12 seconds

second-degree (type 2) block details

- atrial rate - regular, P-P is regular, rate is variable - ventricular rate - regular until QRS is dropped, rate is less than atrial rate - P wave - more P waves than QRS complexes - PR interval - 0.12-0.20 seconds where they can be measured - QRS interval - variable

third-degree block/complete block

- atrial rate - regular, PP is regular is underlying rhythm is sinus, 60-100 bpm, can be variable (afib) - ventricular rate - regular, R-R is regular ≤60 bpm (40 to 60 bpm if junctional, 15 to 40 bpm if ventricular) - P waves - more P waves than QRS complexes (complete A-V dissociation) - PR interval - increases with each beat - QRS interval - variable, but usually >0.12 seconds

pulseless electrical activity

- atrial rate - variable - ventricular rate - variable - P waves - may be present or absent - QRS - variable - PR interval - may be present or absent - QRS interval - variable

causes of ventricular fibrillation

- cardiac arrest - untreated or unsuccessfully treated ventricular tachycardia - cardiomyopathy - valvular heart disease - avid-base and electrolyte imbalance - electrical shock

treatment of heart block

- determine whether patient is stable or unstable - discontinue drugs if indicated and continue to monitor - pacemaker may be necessary

causes of heart block

- digitalis - beta-blockers - calcium channel blockers - increased vagal tone - hyperkalemia - myocarditis - rheumatic fever - degeneration of normal conductive pathways as associated with aging

treatment of atrial flutter

- goal is to convert to sinus rhythm - beta-blockers, digitalis, calcium channel blockers - if hemodynamically unstable, immediate cardioversion - anticoagulation prophylaxis - long term treatment - ablation

treatment of atrial fibrillation

- goal is to convert to sinus rhythm - diltiazem - if hemodynamically unstable, immediate cardioversion - anticoagulants prophylaxis - beta-blockers and digitalis

causes of atrial flutter

- hyperthyroidism - ischemic heart disease or AMI - cardiomyopathy - pericarditis - CHF - post-operative cardiac surgery - valvular disease - HTN - pulmonary disease - myocarditis - SSS

causes of pulseless electrical activity

- hypo/hyperkalemia - hypo/hyperglycemia - hypo/hyperthermia - hypoxemia - hypovolemia - trauma - toxins - tamponade - tension pneumothorax - thrombus

treatment of ventricular fibrillation

- immediate defibrillation - CPR - epinephrine - titrate to patient response - vasopressin as alternative to epinephrine

treatment of polymorphic ventricular tachycardia

- immediate defibrillation - precordial thump - CPR if pulseless - epinephrine - amiodarone - lidocaine

causes of supraventricular tachycardia

- nicotine or caffeine intake - stress/anxiety - hypoxemia - underlying CAD or cardiomyopathy

causes of premature ventricular contraction

- nicotine, caffeine, alcohol intake - cardiac ischemia or infarction - increased cardiac workload - digitalis toxicity - hypoxia - acidosis - hypokalemia

treatment for sinus bradycardia

- not necessary unless pt is symptomatic - discontinue medications - have patient cough to decrease vagal tone - atropine - pacemaker may be necessary

what can cause sinus bradycardia?

- parasympathetic nervous stimulation - ischemic SA node - some medications (digoxin, propranolol, barbiturates)

causes of polymorphic ventricular tachycardia

- same as PVCs - low ejection fractions - deadly rhythm with no perfusion although could have a pulse

what can cause sinus tachycardia?

- sympathetic nervous system stimulation - pain - anxiety or fear - drugs (caffeine, nicotine, amphetamine, cocaine, ecstasy, aminophylline, atropine) - hypoxia - hypovolemia - left ventricular failure, shock, or extension of infarct

treatment of premature ventricular contraction

- treated with amiodarone if persistent or frequent

causes of atrial fibrillation

- valvular disease - CHF - sick sinus syndrome (SSS) - cardiomyopathy - pulmonary disease - ischemic heart disease - post-operative cardiac surgery - hyperthyroidism - congenital disease

treatment for supraventricular tachycardia

- vasovagal maneuver (carotid sinus massage, gag, breath-holding, immerse face in cold water, blow in sealed syringe) - administer adenosiney - calcium channel blocker - cardioversion

A 75-year-old patient is being evaluated for heart failure. Which questions will the nurse ask to confirm common signs and symptoms observed in older adults experiencing heart failure? Select all that apply.

-"Do you easily get tired?" -"Do you have swelling in your ankles?" -"Are you feeling depressed?" -"Do you get up often during the night to urinate?"

Ventricular Tachycardia

-150-250 bpm -MI, Dig toxicity, Drug abuse, Electrolyte imbalance. -no pulse in this rhytm treat as v-fib and start CPR

couplet

-2 PVCs together (PVC, PVC, sinus, PVC, PVC, sinus, etc)

triplet

-3 PVCs together (PVC, PVC, PVC, sinus, PVC, PVC, PVC, sinus, etc)

Junctional Rhythm

-40-60 -underlying heart disease -if asymptomatic no treatment -if symptomatic treat with atropine, o2, transcutaneous pacing)

enalapril (Vasotec)

-ACE inhibitor -administer 1 hour ac or 2 hours pc -empty stomach enhances absorption

captopril (Capoten)

-ACE inhibitor -monitor the BP for several hours after 1st dose -take on an empty stomach -avoid antacids, NSAIDs, caution with K+sparing diuretics -monitor creatinine, K+ levels, CBC, UA -report cough -hypotension may occur as a first dose effect, esp in volume depleted people

-sartan endings

-ARBs -monitor BP for several hours after 1st dose -take on an empty stomach -avoid antacids, NSAIDs, caution with K+sparing diuretics -monitor creatinine, K+ levels, CBC, UA -decreased problem with cough

Regarding heart failure, the nurse knows that which of the following statements are correct? Select all that apply.

-Aortic stenosis can cause left-sided failure. -Cardiomyopathy is a common cause of heart failure. -Compensated heart failure may be clinically asymptomatic.

Atrial Flutter

-Atrial rate greater than 230 -r/t underlying cardiac causes -No treatment if flutter if venticular response is WDL, if not - diltiziem, magnesium, beta blockers

Atrial Fibrillation

-Controlled (70-90) -uncontrolled (110-180) -Usually started from PAC (decreased cardiac output) TREATMENT 1. control HR- diltiazem,BB, magnesium 2. convert to NSR: amiodaron, Ibultilide 3. Prvent stroke -cardioversion (if less than 48 hrs, if after admin anticoagulants for 3-4 weeks and then attempt)

non-ST-segment elevation MI (NSTEMI)

-ECG shows ST segment depression or prominent T wave inversion and/or positive biomarkers of necrosis (rise in troponin)

ST-segment elevation MI (STEMI)

-ECG shows ST segment elevation -occurs when thrombus completely blocks coronary artery -causes: acute MI, pericarditis, injury, LV aneurysms, broken heart syndrome (more common in elderly women)

First degree heart block

-Found in all agres with normal and diseased hearts, may be caused by meds -none, usually well tolerated -symptomatic treatment- atropine and pacemaker

The diagnosis is left-sided heart failure. The nurse knows that the most common causes of left-sided failure include which of the following? Select all that apply.

-Hypertension -Acute myocardial infarction

Normal Sinus Rhythm with PVC

-No P wave -if infrequent no treatment -if frequent treat with antiarrhytmics, check K+ and dig levels, treat underlying cause: give 02, decrease pain, electrolyte balance.

Third degree block

-Occasionally congenital, advanced age, acute MI, myocarditis, cardiac surgery, dig toxicity -transverse pacemaker, atropine, isproproteronol, pace maker

Circulatory shock is characterized by an inability of the circulatory system to provide adequate oxygen to body tissues. Which of the following damaging effects at the cellular level can cause hypoxia? Select all that apply.

-Pyruvate converted to lactic acid -Cellular edema -Deranged sodium/potassium balance -Impaired cellular production of adenosine triphosphate (ATP)

Ventricular Fibrillation

-Watch for PVCs and treat as ordered -CPR and defibrillate ASAP -may have Implatable Cardioverte Defibrillator

Supraventricular Tachycardia

-atria and ventricular rates = 150-300 BPM -hidding t waves -asymptomatic if episodes are short, if sustained they cause dyspnea, dizziness, palpitation, hypotension, syncope, anxiety or chest pain. -TREAtMENT- O2, IV access, vagal maneuver, adnosine, calcium channel blockers and beta blockers to slow the heart

Dopamine

-beta receptors in the heart increases CO, and renal perfussion

digitalis/digoxin (Lanoxin)

-cardiac glycosides -positive inotropic agent (increase force of heart contraction) -take apical pulse for full minute--note pattern, regularity before giving dose -monitor level: 0.5-2.0 -observe for signs of toxicity: confusion, visual changes, dysrhythmias, anorexia, N/V, fatigue, muscle weakness -notify HCP if <60

beta blockers

-carvedilol (Coreg), metoprolol (Toprol XL) -monitor BP and pulse initially and with each dose increase -teach patient: twice weekly- BP checks, pulse, activity tolerance, orthopnea -teach pt to take daily weights, report evidence of volume overload (increased weight, dyspnea, edema) -teach pt never to stop medication abruptly or without HCP advice -diabetics monitor glucose carefully -arise carefully -stopping therapy quickly may cause angina, dysrhythmias, or MI

ventricular fibrillation

-chaotic quivering of ventricles -grossly irregular electrical activity -unable to recognize ANY waveforms -results in CV COLLAPSE :( -causes: drugs that inc QT, ischemia, electrolyte abnormalities, reduces EF <30% -tx: CALL CODE (CPR, defib, epi, amiodarone) -think of Greys (VFIB, PUSH ONE OF EPI)

Second degree block Mobitz II

-damage to ventricular septum (MI) -may progress to third degree block -transvaneous pacing, atropine, epi

P Wave

-depolarization of the atria or contraction of the atria -No longer than 0.12 seconds

QRS Complex

-depolarization of the ventricular muscle -less than 0.12 seconds

bigeminy

-every other beat is PVC (PVC, sinus, PVC, sinus, etc)

asystole

-flatline -absence of electrical activity -result of prolonged cardiac arrest -successful resuscitation not likely -PEA: pulseless electrical activity (rhythm present but NO pulse) -tx: compressions, EPI, oxygenate, fluid challenge, find cause!! -causes (H/T): hypoxemia, hypovolemia, hypothermia, hypo/hyperkalemia, hydrogen ion (acidosis), tamponade, tension pneumothorax, toxins (drugs), thromboembolism (pulmonary or coronary)

loop diuretics

-furosemide (Lasix), bumetanide (Bumex) -give in the early AM -assess DW, I&O, breath sounds, edema -monitor K+, Na+ -monitor for signs of dehydration -note ringing in ears -urine increases 6-8 hours after oral dose -weight is the most accurate noninvasive measurement of volume status

Sinus Tachycardia

-greater than 100 BPM -usually harmless if less than 150 bpm but cause needs to be identified -S/sx- dizziness, hypotension -TREATMENT- eliminate cause

second degree heart block

-grouped beatings -PR interval gets progressively longer followed by a QRS complex -inferior MI or dig toxicity, may be normal -no treatment except check drug levels

Sinus Bradycardia

-heart rate is less than 60 bpm -no treatment if asymptomatic -if symptomatic- atropine and oxygen, transcutaneous pacing, dopamine, epinepherine.

Normal Sinus Rhythm

-hr 60-100 bpm

thiazide diuretics

-hydrochlorothiazide, chlorothiazide -inexpensive -measure I&O, daily weight -assess for edema -monitor electrolytes, glucose, BUN, Cr, Uric acid, bili, Mg, Chol -give in the morning with food -monitor for digitalis toxicity

P-R Interval

-impulse that passes from the SA node, through the atria and AV node to the ventricles -0.12-0.20 seconds

Sinus Arrest

-in elderly pt with disease of SA node -results from med intoxication -acute MIs -Treatment-- look for cause, discontinue drugs that may cause the problem, may need to use atropine or pacer

Epinephrine

-increases contraction, HR, CO, and vasoconstriction

a-fib

-irregular R-R interval -no P waves -fibrillatory waves -SA node no longer in control of heart -impulses coming from everywhere in atria -no ventricular rate (HR can be slow or fast) -causes: CHF, HTN, valve disorder, CMP, MI, fluid overload, pericarditis, hyperthyroidism, idiopathic -emboli can develop -tx: rate and rhythm control, radio-frequency ablation, anticoagulation

Asystole

-lack of firing throughout the heart -check pt first -cpr -check another lead -epi, atropine, pacemaker

Normal Sinus Rhythm with Premature Atrial Contractions

-may occur in normal people - if healthy heart no treament, depends on symptoms -Treatment- sedation and removal of exciting cause, may admin digoxin, propanolol, quinindine -monitor so it doesn't progress to a-fib

Nitro-Dur

-nitrate -assess for HA, dizziness,hypotension, tachycardia -HA diminishes with tolerance

isosorbide

-nitrate -observe for postural hypotension -supervise ambulation -dizziness -avoid erectile dysfunction meds

Nitrodisc

-nitrate -place on hairless part of body

Sinus Dysrhythmias

-normal in young personw tih respirations -may indivate disaeas of SA node in the elderly -may be by meds -usually not treated

sinus bradycardia

-rate <60 bpm -causes: hyperkalemia, vagal stimulation, hypoxia/sleep apnea, athlete, dec blood flow, hypothyroid, inc ICP, meds

ventricular tachycardia

-rate >100 bpm -WIDE QRS -AV dissociation (atria and ventricles no longer working together, ventricles took over) -causes: electrolyte imbalance, ischemia, acute MI, CAD, CHF, reduced EF -tx: determine if pulse or pulseless, correct electrolytes, cardioversion *PULSE: state 12 lead EKG, VS, vagal maneuvers, drug therapy *PULSELESS: call CODE, CPR, defib, epi, amiodarone

sinus tachycardia

-rate >100 bpm (adults) -sx: fatigue, weakness, SOB, orthopnea, dec SpO2, dec BP -tx: maybe none, treat underlying cause

T Wave

-recovery phase after ventricular contraction. -antyhing that interferes with repolarization can invert the t waves.

morphine

-reduces preload and after load -decreases respirations and anxiety, which improves gas exchange and decreases dyspnea -decreases oxygen demand

atrial flutter

-regular flutter waves (rapid and regular) -ventricular rate irregular -unlike a fib, can have regular ventricular response -rate of atrial 140-150 -worse than a fib bc results in faster HR and cont to pump blood -most common adult arrythmia -causes: CHF, HTN, valve disorder, CMP, MI, fluid overload, pericarditis, hyperthyroidism, idiopathic -tx: rate and rhythm control, radio-frequency ablation, anticoagulation

potassium-sparing diuretics

-spironolactone (Aldactone) -assess potassium level -monitor digitalis level if given also -give in the morning -may be given with a loop diuretic

ST Segment

-time period between completion of depolarization and the beginning of repolarization

QT interval

-ventricullar depolarization and repolarization. -0.36-0.44

premature ventricular complexes (PVCs)

-wide, bizzare QRS -BEAT, not rhythm (can have PVC w sinus rhythm) -premature complex -not associated w/P waves (P are atrial) -originate from ventricles -can have sinus rhythm, sinus brady, or sinus tachy w/PVC -bigeminy, triplet, couplet -worry if EF <30%, >6 PVCs/min, >2PVCs together, polymorphic -causes: ischemia/injury, drugs or electrolyte imbalance, invasive lines -tx: determine frequency, assess for cause and treat that, DO NOT routinely give drugs to suppress bc pt can die, can be ablated if excessive and symptomatic

how many seconds in small box?

.04 seconds

PR interval length

0.12 - 0.2 seconds

QRS interval length

0.12 seconds or less

how many seconds in large box?

0.2 seconds

The nurse is caring for a patient newly diagnosed with heart failure. The patient is to receive a first dose of digoxin 0.125 mg IV push. An ampule containing 0.25 mg/mL is available. Calculate the number of milliliters the nurse should draw up to administer this dose. ______ mL

0.5

The nurse is caring for a patient newly diagnosed with heart failure. The patient is to receive a first dose of digoxin 0.125 mg IV push. An ampule containing 0.25 mg/mL is available. Calculate the number of milliliters the nurse should draw up to administer this dose. ______mL

0.5

Anaphylactic shock is most frequently due to a Type______Correct Response(50 %)hypersensitivity reaction, which involves______Correct Response(50 %)antibodies.

1 IgE

A nurse provides education to a hypertensive patient related to lifestyle modifications to reduce cardiovascular risks associated with high blood pressure (BP). Which statement made by the patient indicates effective learning? Select all that apply. 1. "I should achieve and maintain a healthy weight." 2. "I can continue to smoke, because nicotine does not affect blood pressure." 3. "I should exercise for at least 30 minutes daily." 4. "I can have up to five alcoholic drinks per day." 5. "I should restrict my salt intake to less than 1500 mg/day."

1. "I should achieve and maintain a healthy weight." 3. "I should exercise for at least 30 minutes daily." 5. "I should restrict my salt intake to less than 1500 mg/day." Lifestyle modifications play a vital role in reducing blood pressure and cardiovascular risk. Overweight people are at higher risk of cardiovascular disease. A weight loss of 22 lb may decrease systolic blood pressure by approximately 5 to 20 mm Hg. Being physically active is essential to maintain good health. It decreases the cardiovascular risk of hypertension. Sodium reduction helps to control blood pressure. A hypertensive patient should lower salt intake to 1500 mg/day. The nicotine in tobacco causes vasoconstriction and increases blood pressure. Therefore, smokers who are hypertensive should stop smoking. Excessive alcohol consumption increases the risk of hypertension. Consuming three or more drinks per day increases the risk of cardiovascular disease and stroke. Text Reference - p. 715

A patient is diagnosed with primary hypertension and asks the nurse what caused this condition. Which is the best response by the nurse? 1. "There is no one identifiable reason." 2. "Kidney disease is the most common reason." 3. "It is caused by a decrease in plasma renin levels." 4. "There is too much plaque in the blood vessels."

1. "There is no one identifiable reason." There is not one exact cause of primary hypertension; there are several contributing factors. Renal or kidney disease is a cause of secondary hypertension. An increase, not a decrease, in plasma renin levels is a contributing factor in the development of primary hypertension. Hypertension speeds up the process of atherosclerosis in the peripheral blood vessels. Text Reference - p. 712

The nurse is obtaining a health history from a patient with hypertension. Nonmodifiable risk factors for the development of hypertension include which of these? Select all that apply. 1. Age 65 years 2. Excessive dietary sodium 3. African American ethnicity 4. Excessive alcohol consumption 5. A family history of hypertension

1. Age 65 years 3. African American ethnicity 5. A family history of hypertension Nonmodifiable risk factors for hypertension include increasing age, African American ethnicity, and a family history of hypertension. Consumption of excessive dietary sodium and excessive alcohol consumption are considered modifiable risk factors. Text Reference - p. 713

The nurse is taking the blood pressure of an older patient and compares the result to the previous reading. The nurse observes that the systolic blood pressure of the previous reading was 30 mm Hg higher than the reading the nurse just obtained. Which of these is a possible explanation? 1. An aucultatory gap may be present. 2. The patient's hypertension is improving. 3. The equipment the nurse used is not working properly. 4. The patient was more relaxed with the most recent blood pressure measurement.

1. An aucultatory gap may be present. Careful technique is important in assessing blood pressure in older adults. Some older people have a wide gap between the first Korotkoff sound and subsequent beats. This is called the auscultatory gap. Failure to inflate the cuff high enough may result in underestimating systolic blood pressure. Text Reference - p. 725

A nurse is monitoring the digoxin level for a client who has been taking a daily dose of digoxin for 1 month. the digoxin level is 0.25 ng/mL. The nurse should notify the provider and anticipate which of the following: 1. An increase in the client's digoxin dose. 2. A decrease in the client's digoxin dose. 3. No change in the client's digoxin dose. 4. Dicontinuation of the client's digoxin dose.

1. An increase in the client's digoxin dose.

The nurse is creating a plan of care for a patient with a new diagnosis of hypertension. Which is a potential nursing diagnosis for the patient taking antihypertensive medications? Select all that apply. 1. Anxiety 2. Constipation 3. Impaired memory 4. Sexual dysfunction 5. Urge urinary incontinence

1. Anxiety 4. Sexual dysfunction Nursing diagnoses associated with patients taking medications for hypertension include anxiety (related to complexity of management regimen) and sexual dysfunction (related to side effects of antihypertensive drugs). Constipation, impaired memory, and urge urinary incontinence are not side effects of antihypertensive drugs. Text Reference - p. 720

A nurse is caring for a patient admitted to the health care facility with acute ischemic stroke. The patient is receiving IV antihypertensive drugs. Which interventions should the nurse perform for this patient? Select all that apply. 1. Assess blood pressure (BP) and pulse every 30 minutes. 2. Titrate drug according to mean arterial pressure (MAP) or BP as prescribed. 3. Measure hourly urine output. 4. Provide assistance to get up as patient desires. 5. Perform frequent neurologic checks.

1. Assess blood pressure (BP) and pulse every 30 minutes. 2. Titrate drug according to mean arterial pressure (MAP) or BP as prescribed. 5. Perform frequent neurologic checks. Drugs should be titrated according to MAP or BP as prescribed. The nurse should measure hourly urine output to assess renal perfusion and should perform frequent neurologic checks. Antihypertensive IV drugs have a rapid onset of action; hence BP and pulse should be assessed every two to three minutes using a noninvasive BP machine. The patient should be restricted to bed; severe cerebral ischemia or fainting may result if the patient tries to get up. Text Reference - p. 272

The nurse providing dietary instruction to a patient with hypertension would advise the patient to cut down on the intake of which foods? Select all that apply. 1. Canned vegetables 2. Red meat 3. Baked chicken 4. Canned fruit 5. Processed cheeses

1. Canned vegetables 2. Red meat 5. Processed cheeses Foods high in fat and sodium—including canned vegetables, red meat, and processed cheeses—should be avoided by the patient with hypertension. Baked chicken and canned fruit are low in sodium and fat. Text Reference - p. 716

A patient has a prescription for nadolol 50 mg by mouth (PO) daily. The nurse questions the prescription after noting which medical diagnosis in the patient's health record? 1. Chronic obstructive pulmonary disease (COPD) 2. Renal insufficiency 3. Diabetes mellitus 4. Hypertension

1. Chronic obstructive pulmonary disease (COPD) Nadolol is a nonselective β1-adrenergic-blocking agent that reduces blood pressure and could affect the β2 receptors in the lungs with larger doses or with drug accumulation. It should be used cautiously in patients with COPD, because it could trigger bronchospasm, a potentially life-threatening adverse effect. Nadolol will not worsen renal insufficiency and diabetes and will treat, not worsen, hypertension. Text Reference - p. 718

A patient whose blood pressure is 200/120 mm Hg is diagnosed with hypertensive crisis. The patient receives a prescription for clonidine. The nurse instructs the patient to avoid engaging in hazardous activities like operating machinery. What is the reason for this instruction? 1. Clonidine causes drowsiness. 2. Clonidine causes orthostatic hypotension. 3. Clonidine causes a rapid decrease in BP. 4. Clonidine causes rebound hypertension

1. Clonidine causes drowsiness. Clonidine is an adrenergic inhibitor that reduces sympathetic outflow from the central nervous system. It lowers BP by reducing peripheral sympathetic tone, dilating the blood vessels, and decreasing the systemic vascular resistance. Drowsiness is a side effect of the drug, and the patient is advised to avoid engaging in hazardous activities, because this can increase the risk of injury. The drug also causes orthostatic hypotension, so the patient is advised to change position slowly. The drug does not cause a rapid decrease in BP. The drug can cause rebound hypertension if discontinued abruptly. Text Reference - p. 717

A patient is prescribed lisinopril for the treatment of hypertension. The patient asks about side effects of this medication. Which side effects should the nurse include? Select all that apply. 1. Cough 2. Edema 3. Dizziness 4. Impotence 5. Hypotension 6. Muscle stiffness

1. Cough 3. Dizziness 4. Impotence Cough, dizziness, and hypotension are side effects of angiotensin-converting enzyme (ACE) inhibitors. Peripheral edema is a side effect of calcium channel blockers. Impotence is a side effect of thiazide diuretics, aldosterone receptor blockers, central-acting alpha-adrenergic antagonists, peripheral-acting alpha-adrenergic antagonists, beta-adrenergic blockers, and mixed alpha 1 and beta 1 blockers. Muscle stiffness is not associated with an ACE inhibitor. Test-Taking Tip: Avoid spending excessive time on any one question. Most questions can be answered in one to two minutes. Text Reference - p. 724

A patient is being discharged from the hospital. The primary health care provider prescribes propranolol for hypertension. Which instruction should the nurse include in the patient's discharge teaching plan? 1. Do not stop taking abruptly. 2. Take initial doses at bedtime. 3. Monitor for peripheral edema. 4. Take with orange juice.

1. Do not stop taking abruptly. Patients should not stop taking this medication abruptly, because this may cause rebound hypertension. The initial dose of alpha-1 adrenergic blockers should be taken at bedtime because of the possible profound orthostatic hypotension with syncope within 90 minutes after the initial dose. Calcium channel blockers may cause peripheral edema. Beta blockers are not potassium wasting, so it is not necessary to take them with orange juice. Text Reference - p. 725

A nurse is monitoring the blood pressure (BP) of a patient visiting the health care facility. What should the nurse ensure when recording the BP? Select all that apply. 1. Ensure the patient has not exercised within 30 minutes. 2. Seat the patient and begin measurement. 3. Support the patient's arm at heart level. 4. Palpate the radial pulse for auscultatory measurement. 5. Deflate the cuff at the rate of 5 mm Hg/sec.

1. Ensure the patient has not exercised within 30 minutes. 3. Support the patient's arm at heart level. 4. Palpate the radial pulse for auscultatory measurement. The nurse should ensure that the patient has not exercised, smoked, or ingested caffeine within 30 minutes before measurement. The patient's arm should be supported at heart level. The radial pulse is palpated for auscultatory measurement. The nurse should begin measurement only after the patient has rested patiently for 5 minutes after sitting. The cuff should be deflated at a rate of 2 to 3 mm Hg/sec. Text Reference - p. 723

The nurse teaches a patient with hypertension that symptoms of uncontrolled hypertension may include which of the following? Select all that apply. 1. Fatigue 2. Dizziness 3. Palpitations 4. Shortness of breath 5. Cluster headaches

1. Fatigue 2. Dizziness 3. Palpitations Uncontrolled hypertension may result in fatigue, dizziness, and palpitations. Cluster headaches and shortness of breath do not occur with uncontrolled hypertension. Test-Taking Tip: Be alert for details about what you are being asked to do. In this question type, you are asked to select all options that apply to a given situation or client. All options likely relate to the situation, but only some of the options may relate directly to the situation. Text Reference - p. 713

A nurse is providing teaching to a client who has a new prescription for digoxin (Lanoxin) Which of the following may indicate dig toxicity & should be reported to the provider? 1. Fatigue 2. constipation 3. Anorexia 4. Rash 5. Diplopia

1. Fatigue Not constipation but -- nausea, vomiting & diarrhea 3. Anorexia b/c GI disturbances 5. Diplopia -- visual changes , halo, yellow-tinged vision.

Lisinopril (Prinivil) is part of the treatment regimen for a client with HF. The nurse monitors the client for the development of which of teh following adverse effects of this drug? SELECT ALL THAT APPLY 1. Hyperkalemia 2.Hypocalcemia 3. Cough 4. Dizziness 5. Heartburn

1. Hyperkalemia 2.Hypocalcemia 3. Cough 4. Dizziness

The nurse teaches a patient with hypertension that uncontrolled hypertension may damage organs in the body primarily by which mechanism? 1. Hypertension promotes atherosclerosis and damage to the walls of the arteries. 2. Hypertension causes direct pressure on organs, resulting in necrosis and replacement of cells with scar tissue. 3. Hypertension causes thickening of the capillary membranes, leading to hypoxia of organ systems. 4.Hypertension increases blood viscosity, which contributes to intravascular coagulation and tissue necrosis distal to occlusions.

1. Hypertension promotes atherosclerosis and damage to the walls of the arteries. Hypertension is a major risk factor for the development of atherosclerosis by mechanisms not yet fully known. However, once atherosclerosis develops, it damages the walls of arteries and reduces circulation to target organs and tissues.

A nurse is preparing to measure the blood pressure of a patient who is lying down on a bed. What technique should the nurse use to ensure that the BP reading is accurate? 1. Measure the BP twice, waiting one minute between measurements. 2. Rest the patient's arms on the bed during the BP measurement. 3. Measure the BP in both arms and record the lowest reading. 4. Place the BP cuff on the forearm when measuring the BP.

1. Measure the BP twice, waiting one minute between measurements. When recording BP, two measurements should be taken one minute apart. This allows the blood to drain from the arm and prevents inaccurate readings. If the patient is in a lying down position, the arm should be placed on a pillow so that it is at the heart level. Atherosclerosis in the subclavian artery may result in a falsely low BP in the affected side; therefore, the arm which has the highest recording of the BP should be used for further measurements. The upper arm, not the forearm, is the preferred site of BP cuff placement due to its accuracy of recordings. Text Reference - p. 723

A nurse works in a medical unit. The nurse has assessed the patients and planned care for them. Which activities can be delegated to unlicensed assistive personnel (UAP)? Select all that apply. 1. Report high or low BP readings to the registered nurse. 2. Make appropriate referrals to other health care professionals. 3. Teach patients about lifestyle management and medication use. 4. Check for postural changes in BP. 5. Assess patients for hypertension risk factors and develop risk modification plans.

1. Report high or low BP readings to the registered nurse. 4. Check for postural changes in BP. Reporting high or low BP readings and checking for postural changes in BP are repetitive activities and do not require nursing judgment. Therefore, these activities can be delegated to unlicensed assistive personnel. Making appropriate referrals requires understanding of the collaborative care and judgment regarding the requirement of the referrals; this activity cannot be delegated and is the role of a registered nurse. Patient education about lifestyle management and medication use requires sound knowledge; therefore, this activity should be performed by the nurse. Assessment and development of risk modification plans requires assessment and planning skills; this activity should not be delegated and should be performed by the nurse. Text Reference - p. 724

A nurse is preparing discharge teaching for a patient with orthostatic hypotension. Which instructions should be a part of the discharge plan? Select all that apply. 1. Rise slowly from a supine to sitting position. 2. Avoid sleeping with the head elevated. 3. Lie down or sit if dizziness occurs. 4. Do not stand still for prolonged periods. 5. Perform leg exercises to increase venous return.

1. Rise slowly from a supine to sitting position. 3. Lie down or sit if dizziness occurs. 4. Do not stand still for prolonged periods. 5. Perform leg exercises to increase venous return. Orthostatic hypotension is a condition in which there is a decrease in blood pressure upon rising to a standing position from a lying down or sitting position. The patient should be instructed to rise slowly from the sitting and lying down positions and to move only when no dizziness occurs. The patient should sit or lie down if there is dizziness. This prevents the risk of falling. Standing still for prolonged periods may cause venous stasis and worsen hypotension. Doing leg exercises helps to increase venous return to the heart and lowers blood pressure. Sleeping with the head elevated helps to keep the blood flow to the brain uniform and prevents orthostatic hypotension. Text Reference - p. 723

The nurse is teaching the male patient about the most common side effects of antihypertensive medications. Which information would the nurse discuss with the patient? Select all that apply. 1. Sexual dysfunction 2. Resistant hypertension 3. Orthostatic hypotension 4. Frequent voiding and dry mouth 5. Rebound hypotension if the drug is stopped abruptly

1. Sexual dysfunction 3. Orthostatic hypotension 4. Frequent voiding and dry mouth Reduced libido or erectile dysfunction are examples of sexual dysfunction as a side effect of antihypertensive medications. Alteration of the autonomic nervous system mechanism by antihypertensive medications leads to orthostatic hypotension. Diuretics are one class of medications for treatment of hypertension that cause frequent urination and dry mouth. "Resistant hypertension" is a term used to describe failure to reach desired blood pressure (BP) in the patient who takes multiple antihypertensive medications. Rebound hypertension results from abrupt stopping of antihypertensive medication use. Text Reference - p. 721

The nurse is preparing a presentation on complications of hypertension. Which information would the nurse include? Select all that apply. 1. Stroke as a result of carotid artery atherosclerosis 2. Heart failure as a result of increased heart contractility 3. Blurred vision or loss of vision secondary to retinal damage 4. Right ventricular hypertrophy as a result of increased workload 5. Coronary artery disease caused by an increase in the elasticity of arterial walls

1. Stroke as a result of carotid artery atherosclerosis 3. Blurred vision or loss of vision secondary to retinal damage Embolic stroke may be a result of cerebral blood flow obstruction by a portion of atherosclerotic plaque or a blood clot formed in the carotid arteries. Hypertension leads to retinal damage that is manifested by blurred vision or loss of vision and retinal hemorrhage. Heart failure is a result of decreased heart contractility along with decreased stroke volume and cardiac output. Hypertension leads to increased cardiac workload that causes left ventricular hypertrophy. Coronary artery disease is caused by decreased elasticity of arterial walls and narrowing of the lumen. Text Reference - p. 713

A patient arrives at a medical clinic for a check-up. The patient's blood pressure (BP) is 150/94 mm Hg. All other assessment findings are within normal limits. The nurse reviews the patient's file from previous visits, and there is no history of elevated blood pressure. What could be the reason for a falsely high blood pressure? 1. The blood pressure cuff might have been too small. 2. There may be atherosclerosis in the subclavian artery. 3. The patient may have smoked the day before the BP measurement. 4. The patient may have engaged in strenuous exercises the day before the BP measurement.

1. The blood pressure cuff might have been too small. BP measurements should be performed using proper technique to get an accurate reading. BP measurements may be falsely high if the BP cuff is too small as it puts undue pressure on the artery. If the subclavian artery has atherosclerosis, the BP measurement would be falsely low. Smoking and engaging in strenuous exercise should be avoided 30 minutes before the BP measurement, because they can alter the measurement. Smoking or engaging in strenuous exercise one day before a BP measurement will not affect the readings. Text Reference - p. 723

Which test result would indicate the presence of target organ damage resulting from uncontrolled hypertension? 1. Urine protein 3+ 2. Blood urea nitrogen (BUN) 18 mg/dL 3. Uric acid 8.2 mg/dL 4. Triglycerides 144 mg/dL

1. Urine protein 3+ Urine protein should not be present. This increased level indicates target organ damage to the kidneys. The BUN is normal, the elevated uric acid level indicates gout, and the triglyceride level is normal. Test-Taking Tip: Avoid taking a wild guess at an answer. However, should you feel insecure about a question, eliminate the alternatives that you believe are definitely incorrect, and reread the information given to make sure you understand the intent of the question. This approach increases your chances of randomly selecting the correct answer or getting a clearer understanding of what is being asked. Although there is no penalty for guessing on the NCLEX examination, the subsequent question will be based, to an extent, on the response you give to the question at hand; that is, if you answer a question incorrectly, the computer will adapt the next question accordingly based on your knowledge and skill performance on the examination up to that point. Text Reference - p. 714

When computing a heart rate from the ECG tracing, the nurse counts 15 of the small blocks between the R waves of a patient whose rhythm is regular. From these data, the nurse calculates the patient's heart rate to be 60 beats/min. 75 beats/min. 100 beats/min. 150 beats/min.

100 beats/min. Since each small block on the ECG paper represents 0.04 seconds, 1500 of these blocks represents 1 minute. By dividing the number of small blocks (15, in this case) into 1500, the nurse can calculate the heart rate in a patient whose rhythm is regular (in this case, 100).

a. Although obtaining the ECG, chest radiograph, and blood work are all important, the nurse's priority action should be to relieve the crushing chest pain. Therefore, administering morphine sulfate is the priority action.

10] A 60-year-old male client comes into the emergency department with a complaint of crushing substernal chest pain that radiates to his shoulder and left arm. The admitting diagnosis is acute myocardial infarction (MI). Immediate admission orders include oxygen by nasal cannula at 4L/minute, blood work, a chest radiograph, a 12-lead electrocardiogram (ECG), and 2mg of morphine sulfate given IV. The nurse should first: a. Administer the morphine b. Obtain a 12-lead ECG c. Obtain the blood work d. Order the chest radiograph

In reviewing medication instructions with a patient prescribed lisinopril, the nurse should include which statement? 1. "You should not take this medication if you have asthma." 2. "You may develop a dry cough while taking this medication." 3. "Never take this medication on an empty stomach." 4. "Discontinue use of this medication if you develop a drop in your blood pressure."

2. "You may develop a dry cough while taking this medication." Lisinopril is an ACE-inhibitor. A common side effect is a dry cough. This medication is safe for use with asthma, can be taken on an empty stomach, and should not be discontinued unless instructed to do so by a health care provider. Text Reference - p. 719

The nurse records normal blood pressure (BP) for a patient with a family history of hypertension and diabetes. What should the nurse teach the patient to specifically address the risks of hypertension? Select all that apply. 1. Increase caloric intake. 2. Avoid foods high in sodium. 3. Reduce the use of tobacco products. 4. Take brisk walks. 5. Avoid overexertion with muscle-strengthening activities

2. Avoid foods high in sodium. 4. Take brisk walks. The nurse should teach the patient to adopt lifestyle changes, such as avoiding foods high in sodium and taking brisk walks. A decrease in caloric intake helps to reduce weight and prevent hypertension. The patient should completely avoid use of tobacco products, because the nicotine contained in tobacco causes vasoconstriction and increases BP. All adults should perform muscle-strengthening activities to maintain and increase endurance and strength of muscles. Text Reference - p. 716

The nurse is caring for a patient admitted with a history of hypertension. The patient's medication history includes hydrochlorothiazide daily for the past 10 years. Which parameter would indicate the optimal intended effect of this drug therapy? 1. Weight loss of 2 lb 2. Blood pressure 128/86 3. Absence of ankle edema 4. Output of 600 mL per eight hours

2. Blood pressure 128/86 Hydrochlorothiazide may be used alone as monotherapy to manage hypertension or in combination with other medications if not effective alone. After the first few weeks of therapy, the diuretic effect diminishes, but the antihypertensive effect remains. Because the patient has been taking this medication for 10 years, the most direct measurement of its intended effect would be the blood pressure. Text Reference - p. 717

A nurse is administering a dopamine infusion at a moderate dose to a client who has severe HF. Which of the following is an expected effect? 1. Lowered heart rate 2. Increased myocardial contractility 3. Decreased conduction through the AV node D. Vasoconstriction of the renal blood vessels

2. Increased myocardial contractility -- thus increasing CO

A nurse is providing care for a patient who continues to experience hypertension despite taking a calcium channel blocker daily. A diuretic has been prescribed. How does a diuretic help control blood pressure? Select all that apply. 1. It causes vasodilation. 2. It reduces plasma volume. 3. It promotes sodium and water excretion. 4. It reduces the vascular response to catecholamines. 5. It prevents extracellular calcium from moving into the cells.

2. It reduces plasma volume. 3. It promotes sodium and water excretion. 4. It reduces the vascular response to catecholamines. Diuretics are an important component of BP treatment. Diuretics tend to reduce the plasma volume by promoting excretion of sodium and water. The net result is a reduction in the circulating volume, which causes a decrease in the BP. Diuretics also reduce the vascular response to catecholamines. The blood vessels do not constrict in response to catecholamines; as a result, the BP is reduced. Diuretics do not cause vasodilation or prevent the movement of extracellular calcium into the cells; these effects are brought about by calcium channel blockers. Text Reference - p. 720

The nurse understands that which medication may increase the risk of hypoglycemia unawareness in a patient with diabetes mellitus? 1. Cardizem 2. Metoprolol 3. Prednisone 4. Hydromorphone

2. Metoprolol Metoprolol, a beta blocker, slows the heart rate. Tachycardia is a classic symptom of hypoglycemia; therefore patients must be made aware of failure of the heart rate to respond to decreasing blood sugars and should be instructed to look for other signs of hypoglycemia. Cardizem and hydromorphone will not affect blood sugars or signs/symptoms of hypoglycemia. Prednisone will increase, not decrease, blood sugar levels. Text Reference - p. 718

The nurse is obtaining data from a patient who has been on medication for hypertension and diabetes for four years. The patient has been experiencing blurred vision due to retinal damage caused by hypertension. What are the other manifestations of target organ disease? Select all that apply. 1. Pneumonia 2. Nocturia 3. Aneurysm 4. Transient ischemic attack 5. Anemia

2. Nocturia 3. Aneurysm 4. Transient ischemic attack Hypertension affects the kidneys; the earliest manifestation of renal disease is nocturia. Hypertension speeds up the process of atherosclerosis in the peripheral blood vessels, leading to aneurysms. Adequate control of blood pressure (BP) reduces the risk of transient ischemic attack. Pneumonia and anemia are not manifestations of target organ disease. Text Reference - p. 714

When teaching a patient about dietary management of stage 1 hypertension, which instruction is most appropriate? 1. Restrict all caffeine 2. Restrict sodium intake 3. Increase protein intake 4. Use calcium supplements

2. Restrict sodium intake The patient should decrease intake of sodium. This will help to control hypertension, which can be aggravated by excessive salt intake, which in turn leads to fluid retention. Caffeine and protein intake do not affect hypertension. Calcium supplements are not recommended to lower blood pressure. Text Reference - p. 712

When providing dietary instruction to a patient with hypertension, the nurse would advise the patient to restrict intake of which meat? 1. Broiled fish 2. Roasted duck 3. Roasted turkey 4. Baked chicken breast

2. Roasted duck Roasted duck is high in fat, which should be avoided by the patient with hypertension. Weight loss may slow the progress of atherosclerosis and overall cardiovascular disease risk. Broiled fish, roasted turkey, and baked chicken breast are lower in fat and are therefore acceptable in the diet. Test-Taking Tip: Once you have decided on an answer, look at the stem again. Does your choice answer the question that was asked? If the question stem asks "why," be sure the response you have chosen is a reason. If the question stem is singular, then be sure the option is singular, and the same for plural stems and plural responses. Many times, checking to make sure that the choice makes sense in relation to the stem will reveal the correct answer. Text Reference - p. 715

A patient has been asked to monitor blood pressure (BP) levels at home twice a day. What should the nurse teach the patient about measuring BP in the supine position? 1. Take at least two consecutive readings one after another. 2. Support the arm with a pillow during measurement. 3. Take the reading immediately after lying down. 4. Use the arm with the lowest BP for all future measurements.

2. Support the arm with a pillow during measurement. When measuring BP in a supine position, the patient should support the arm with a small pillow to raise the position of the hand to the level of the heart. Record the average pressure by taking two consecutive readings at least one minute apart; this allows the blood to drain from the arm and prevents inaccurate readings. The first reading should be taken after two to three minutes of rest in a supine position. If bilateral BP measurements are not equal, the patient should use the arm with the highest BP for all future measurements. Text Reference - p. 723

A nurse is caring for an older adult client who has a new prescription for digoxin and takes multiple other medications. concurrent use of which of the following medications places the client at risk for dig toxicity? 1. Phenytoin (Dilantin) 2. Verapamil (Calan) 3. Warfarin (Coumadin) 4. Aluminum hydroxide (Amphojel)

2. Verapamil (Calan) -- CCB, can increase digoxin levels. if used together dig dosage may need to be lowered.

Atrial Flutter ? bpm Pattern (4) - that originate from _ high _ rates of over _ bpm and _ can _ (3) increased risk for _ _ is the treatment of choice causes: (5).

250-300 bpm regular, recurring, saw-tooth flutter waves that originate from single ectopic foci in the right atrium high ventricular rates of over 100 bpm and the loss of the atrial kick (in the p wave) can decrease CO and cause chest pain/ HF increased risk for stroke catheter ablation is the treatment of choice causes: CAD, hypertension, mitral valve disorders, pulmonary emboli, drugs ect..

In hypovolemic shock, renal perfusion and urinary output decline. The nurse will monitor urinary output and knows that output below which of the following levels indicates inadequate renal perfusion?

20 mL/hour

The nurse is monitoring hourly urine output of a client diagnosed with hypovolemic shock. The nurse is most concerned if the client's output is:

20 mL/hour

b. Furosemide is a loop diuretic that acts to increase urine output. Furosemide does not increase blood pressure, decrease pain, or decrease arrhythmias

20] Which of the following is an expected outcome when a client is receiving an IV administration of furosemide? a. Increased blood pressure b. Increased urine output c. Decreased pain d. Decreased premature ventricular contractions

d. The woman who is 65-years-old, over weight and has an elevated LDL is at greatest risk. Total cholesterol >200, LDL >100, HDL <40 in men, HDL <50 in women, men 45-years and older, women 55-years and older, smoking and obesity increase the risk of CAD. Atorvastatin is a medication to reduce LDL and decrease risk of CAD. The combination of postmenopausal, obesity and high LDL cholesterol places this client at greatest risk.

23] The nurse is assessing clients at a health fair. Which client is at greatest risk for coronary artery disease? a. a 32-year-old female with mitral valve prolapse who quit smoking 10 years ago. b. a 43-year-old male with a family history of CAD and cholesterol level of 158 c. A 56-year-old male with an HDL of 60 who takes atorvastatin (Lipitor) d. A 65-year-old female who is obese with an LDL of 188

c. CRP is a marker of inflammation and is elevated in the presence of cardiovascular disease. The high sensitivity CRP (hs-CRP) is the blood test for greater accuracy in measuring the CRP to evaluate cardiovascular risk. The family history, post-menopausal age, LDL above optimum levels and elevated CRP place the client at risk of CAD. Statin medications can decrease LDL, whereas statins and aspirin can reduce CRP and decrease the risk of MI and stroke.

24] A 58-year-old female with a family history of CAD is being seen for her annual physical exam. Fasting lab test results include: Total cholesterol 198; LDL cholesterol 120; HDL cholesterol 58; Triglycerides 148; Blood sugar 102; and C-reactive protein (CRP) 4.2. The health care provider informs the client that she will be started on a statin medication and aspirin. The client asks the nurse why she needs to take these medications. Which is the best response by the nurse? a. "The labs indicate severe hyperlipidemia and the medications will lower your LDL, along with a low-fat diet." b. "The triglycerides are elevated and will not return to normal without these medications." c. "The CRP is elevated indicating inflammation seen in cardiovascular disease, which can be lowered by the medications ordered." d. "The medications are not indicated since your lab values are all normal."

c. A basic principle of behavior modification is that behavior that is learned and continued is behavior that has been rewarded. Other reinforcement techniques have not been found to be as effective as reward.

25] If a client displays risk factors for coronary artery disease, such as smoking cigarettes, eating a diet high in saturated fat, or leading a sedentary lifestyle, techniques of behavior modification may be used to help the client change the behavior. The nurse can best reinforce new adaptive behaviors by: a. Explaining how the old behavior leads to poor health b. Withholding praise until the new behavior is well established c. Rewarding the client whenever the acceptable behavior is performed d. Instilling mild fear into the client to extinguish the behavior

The nurse is teaching the patient about the Dietary Approaches to Stop Hypertension (DASH) diet. Which statement indicates that the patient understood the teaching? 1. "I should eat more red meat, such as pork or beef." 2. "I should drink no more than three glasses of whole milk per day." 3. "I should include four to five servings of fruits and vegetables daily." 4. "I should consume whole grain products no more than once per week."

3. "I should include four to five servings of fruits and vegetables daily." The DASH diet encourages consumption of fruits and vegetables. Pork and beef are high in fat and therefore have to be restricted according to the DASH diet; poultry and fish have to be consumed instead of red meat. Fat-free or low-fat milk has to be used instead of whole milk according to the DASH recommendations. The DASH diet recommends a few servings of whole grain products daily. Text Reference - p. 715

A patient has a new prescription for doxazosin. When providing education about this drug, the nurse will include which instructions? 1. "Weigh yourself daily, and report any weight loss to your prescriber." 2. "Increase your potassium intake by eating more bananas and apricots." 3. "Take this drug at bedtime because of the risk of orthostatic hypotension." 4. "The impaired taste associated with this medication usually goes away in two to three weeks."

3. "Take this drug at bedtime because of the risk of orthostatic hypotension." A patient who is starting doxazosin should take the first dose while lying down because there is a first-dose hypotensive effect with this medication. Taking the drug at bedtime reduces risks associated with orthostatic hypotension. The patient does not need to increase potassium intake. Doxazosin does not cause impaired taste. It does not cause weight loss, because it is not a diuretic. Test-Taking Tip: Relax during the last hour before an exam. Your brain needs some recovery time to function effectively. Text Reference - p. 723

For what change in vital signs would the nurse assess a patient experiencing postural hypotension? 1. Increased systolic blood pressure, decreased pulse rate 2. Increased diastolic blood pressure, increased pulse rate 3. Decreased systolic blood pressure, decreased diastolic blood pressure, increased pulse rate 4. Decreased systolic blood pressure, increased diastolic blood pressure, no change in pulse rate

3. Decreased systolic blood pressure, decreased diastolic blood pressure, increased pulse rate A decrease in both systolic and diastolic blood pressure and an increase in pulse would be seen in a patient with postural hypotension. Blood pressure drops as the volume of circulating blood decreases when a patient abruptly stands from a lying or sitting position. The pulse rate increases as the heart attempts to compensate by increasing the amount of circulating blood by increasing cardiac output. Increased systolic blood pressure and decreased pulse rate; increased diastolic blood pressure and increased pulse rate; and decreased systolic blood pressure, increased diastolic blood pressure, and no change in pulse rate are all incorrect. Text Reference - p. 723

The patient has chronic hypertension. Today the patient has gone to the emergency department and the patient's blood pressure has risen to 200/140. What is the priority assessment for the nurse to make? 1. Is the patient pregnant? 2.Does the patient need to urinate? 3. Does the patient have a headache or confusion? 4. Is the patient taking antiseizure medications as prescribed?

3. Does the patient have a headache or confusion? The nurse's priority assessments include neurologic deficits, retinal damage, heart failure, pulmonary edema, and renal failure. The headache or confusion could be seen with hypertensive encephalopathy, from increased cerebral capillary permeability leading to cerebral edema. Pregnancy can lead to secondary hypertension. Needing to urinate and taking antiseizure medication do not indicate a hypertensive emergency. Text Reference - p. 726

The nurse understands that catapres has which side effects? 1. Cough and confusion 2. Sweating and shaking 3. Dry mouth and sedation 4. Gynecomastia and dizziness

3. Dry mouth and sedation Dry mouth and sedation are side effects of central-acting alpha-adrenergic antagonists. Cough is a possible side effect of angiotensin-converting enzyme inhibitors. Confusion, sweating, and shaking are not common side effects for hypertension drug therapy. Gynecomastia is a side effect of aldosterone receptor blockers. Dizziness is a side effect for hypertension drug therapy; gynecomastia is not. Text Reference - p. 717

A patient is scheduled a dose of metoprolol. The nurse should withhold the dose and consult the health care provider after noting which assessment finding? 1. Migraine headache 2. Pulse 112 beats/minute 3. Expiratory wheezing 4. Blood sugar 217 mg/dL

3. Expiratory wheezing Metoprolol is a β-adrenergic-blocking agent that reduces blood pressure and could affect the β2 receptors in the lungs with larger doses or with drug accumulation. It should be used cautiously in patients with wheezing or respiratory disorders because it could cause bronchospasm, a potentially life-threatening adverse effect. Metoprolol will not worsen migraine, will decrease the elevated pulse rate, and will not lower or further elevate the blood sugar. Text Reference - p. 718

The nurse is assessing a patient and auscultates a "swooshing" sound heard over the chest wall when the stethoscope is lifted just off of the chest. The nurse would document this finding as a(n): 1. Severe bruit 2. Atrial gallop 3. Grade VI murmur 4. Pericardial friction rub

3. Grade VI murmur A murmur is classified as turbulent blood flow, which produces the classic swooshing sound as it passes through the valve and is graded on a scale of I toVI, with VI being the loudest, heard when the stethoscope is not touching the chest wall. A bruit is auscultated over arteries. An atrial gallop is an extra heart sound and is not associated with turbulent blood flow. A pericardial friction rub is a scratching sound caused when inflamed surfaces of the pericardium move against each other, indicating cardiac inflammation. Text Reference - p. 726

When teaching how lisinopril will help lower the patient's blood pressure, which mechanism of action should the nurse use to explain it? 1. Blocks β-adrenergic effects 2. Relaxes arterial and venous smooth muscle 3. Inhibits conversion of angiotensin I to angiotensin II 4. Reduces sympathetic outflow from the central nervous system (CNS)

3. Inhibits conversion of angiotensin I to angiotensin II Lisinopril is an angiotensin-converting enzyme (ACE) inhibitor that inhibits the conversion of angiotensin I to angiotensin II, which reduces angiotensin II-mediated vasoconstriction and sodium and water retention. Beta blockers result in vasodilation and decreased heart rate. Direct vasodilators relax arterial and venous smooth muscle. Central acting α-adrenergic antagonists reduce sympathetic outflow from the CNS to produce vasodilation and decreased systemic vascular resistance (SVR) and blood pressure (BP). Test-Taking Tip: Do not read too much into the question or worry that it is a "trick." If you have nursing experience, ask yourself how a classmate who is inexperienced would answer this question from only the information provided in the textbooks or given in the lectures. Text Reference - p. 719

The nurse is teaching a patient, recently diagnosed with hypertension (HTN), about diagnostic studies prescribed by a primary health care provider. Which information would the nurse include? Select all that apply. 1. Echocardiography to evaluate cardiac status 2. ECG to evaluate degree of left ventricular hypertrophy 3. Lipid profile to provide information about the risk factor for HTN 4. Uric acid level because it frequently decreases with diuretic therapy 5. Blood urea nitrogen (BUN) and serum creatinine levels to provide information on renal function

3. Lipid profile to provide information about the risk factor for HTN 5. Blood urea nitrogen (BUN) and serum creatinine levels to provide information on renal function An elevated lipid profile is an additional risk factor for hypertension because having elevated blood lipids leads to development of atherosclerosis. BUN, creatinine, and urinalysis provide information about baseline renal function and help to identify renal damage. Echocardiography evaluates the degree of ventricular hypertrophy, whereas ECG is used to assess baseline cardiac function. Diuretic therapy frequently leads to an increase in uric acid. Text Reference - p. 715

A nurse in a provider's office is monitoring serum electrolytes for 4 older adult clients who take digoxin(Lanoxin) & furosemide(Lasix). Which of the following electrolyte values puts a client at risk for dig toxicity? 1. Calcium 9.2 mg/dL 2. Calcium 10.3 mg/dL 3. Potassium 3.4 mEq/L 4. Potassium 4.8 mEq/L

3. Potassium 3.4 mEq/L

In caring for a patient admitted with poorly controlled hypertension, the nurse should understand that which laboratory test result would indicate the presence of target organ damage? 1. Blood urea nitrogen (BUN) of 15 mg/dL 2. Serum uric acid of 3.8 mg/dL 3. Serum creatinine of 2.6 mg/dL 4. Serum potassium of 3.5 mEq/L

3. Serum creatinine of 2.6 mg/dL The normal serum creatinine level is 0.6 to 1.3 mg/dL. This elevated level indicates target organ damage to the kidneys. BUN of 15 mg/dL, serum uric acid of 3.8 mg/dL, and serum potassium of 3.5 mEq/L are within normal limits. Text Reference - p. 715

A patient reports chest pain and is admitted to the emergency department. The patient is obese, smokes cigarettes, and drinks alcohol in moderate amounts. The patient had taken labetalol for high blood pressure (BP) for one week and then stopped taking the medication the morning of admission. The nurse recognizes that the probable reason for the patient's angina is what? 1. Leading a sedentary lifestyle after a lifetime of obesity 2. Smoking cigarettes 3. Stopping labetalol abruptly after a week of treatment 4. Alcohol consumption

3. Stopping labetalol abruptly after a week of treatment Labetalol is an alpha- and beta-adrenergic blocker and reduces BP by causing vasodilatation and a decrease in heart rate. The patient should not stop the drug abruptly, because it may precipitate angina and heart failure. Obesity, a sedentary lifestyle, smoking, and alcohol consumption are risk factors for cardiovascular disease but are unlikely to cause angina. Text Reference - p. 718

The nurse just received the shift report. Which patient should the nurse assess first? 1. The patient who is complaining about dizziness and whose blood pressure (BP) is 150/92. 2. The patient with a hip fracture who is complaining about pain 2 out of 10 3. The patient who is complaining about severe headache and has a nose bleed 4. The patient complaining of fatigue and who just received an angiotensin-converting enzyme (ACE) inhibitor.

3. The patient who is complaining about severe headache and has a nose bleed Severe headache and nose bleed are signs of hypertensive crisis that is an emergency situation, and therefore the nurse has to see this patient first. Dizziness is one of the symptoms of hypertension and the patient has an elevated blood pressure, but it is not an emergency situation. Pain 2 out of 10 is mild pain and therefore this patient is not a priority. Fatigue is one of the symptoms of hypertension, but the patient just received antihypertensive medication. Text Reference - p. 714

Ventricular Tachycardia

>100 BPM Rhythm usually regular but may be slightly irregular If P wave present that usually bear no relation to the QRS complexes sometimes appearing randomly as notches in between the ventricular complexes. PR interval absent QRS complex wide bizarre shape >0.12 sec; similar shape and points in one direction-monoform V tach; When QRS complexes gradually changes direction and twist around the electrical axis-torsade de pointes

Joe has an EDV of 145 mL and an ESV of 100 mL. What is his ejection fraction? Give your answer as a % with no decimals - i.e. round your answer. Do NOT include the % sign.______Correct Response(50 %)Would this be consistent with systolic dysfunction or diastolic dysfunction? Just give the term systolic or diastolic in your response.______Correct Response(50 %)

31 systolic

d. Crackles are auscultated over fluid-filled alveoli. Crackles heard on lung auscultation do not have to be associated with cyanosis. Bronchospasm and airway narrowing generally are associated with wheezing sounds.

31] Crackles heard on lung auscultation indicate which of the following? a. Cyanosis b. Bronchospasm c. Airway narrowing d. Fluid-filled alveoli

atrial fibrillation

350-500 atrial beats/min and irregular - PR not measurable - fast undefined P wave - QRS normal

c. Pasta, tomato sauce, salad, and coffee would be the best selection for the client following a low-cholesterol diet. Hamburgers, milkshakes, liver, and fried foods tend to be high in cholesterol.

36] The client who experiences angina has been told to follow a low-cholesterol diet. Which of the following meals should the nurse tell the client would be best on her low-cholesterol diet? a. Hamburger, salad, and milkshake b. Baked liver, green beans, and coffee c. Spaghetti with tomato sauce, salad, and coffee d. Fried chicken, green beans, and skim milk

In reviewing medication instructions with a patient being discharged on antihypertensive medications, which statement would be most appropriate for the nurse to make when discussing guanethidine? 1. "A fast heart rate is a side effect to watch for while taking guanethidine." 2. "Stop the drug and notify your health care provider if you experience any nausea or vomiting." 3. "Because this drug may affect the lungs in large doses, it also may help your breathing." 4. "Make position changes slowly, especially when rising from lying down to a standing position."

4. "Make position changes slowly, especially when rising from lying down to a standing position." Guanethidine is a peripheral-acting α-adrenergic antagonist and can cause marked orthostatic hypotension. For this reason, the patient should be instructed to rise slowly, especially when moving from a recumbent to a standing position. Support stockings also may be helpful. Tachycardia or lung effects are not evident with guanethidine, nor are nausea and vomiting. Test-Taking Tip: Start by reading each of the answer options carefully. Usually at least one of them will be clearly wrong. Eliminate this one from consideration. Now you have reduced the number of response choices by one and improved the odds. Continue to analyze the options. If you can eliminate one more choice in a four-option question, you have reduced the odds to 50/50. While you are eliminating the wrong choices, recall often occurs. One of the options may serve as a trigger that causes you to remember what a few seconds ago had seemed completely forgotten. Text Reference - p. 718

A 65-year-old patient without any past medical problems has his or her blood pressure checked at a primary health care provider's office during an annual physical examination. The blood pressure (BP) reading is 158/92. The patient is asking the nurse who was checking the blood pressure: "Does this mean that I have hypertension?" What is the most appropriate answer from the nurse? 1. "Do not worry, everything is fine." 2. "It is a normal blood pressure reading for a person of your age." 3. "Yes, you have hypertension, because your blood pressure is over 140/90." 4. "You need to have a follow-up appointment to recheck your blood pressure."

4. "You need to have a follow-up appointment to recheck your blood pressure." The diagnosis of hypertension is made based on two or more elevated blood pressure readings. Considering the fact that the patient does not have any medical problems and that this reading is the first elevated blood pressure reading, a follow-up office visit is required. Providing false reassurance to the patient is leading to misinformation. For any person of age 18 and older, BP higher than 140/90 is considered elevated. Diagnosing the patient with a medical diagnosis is not within the nursing scope of practice and cannot be done based on one elevated BP reading. Test-Taking Tip: Avoid looking for an answer pattern or code. There may be times when four or five consecutive questions have the same letter or number for the correct answer. Text Reference - p. 715

The nurse is checking blood pressure for people at a health fair. Which patient is at higher risk to develop primary hypertension? 1. 65-year-old retired Caucasian with a body mass index (BMI) of 15 2. 60-year-old who has chronic pain caused by cancer 3. 45-year-old blue collar worker who smokes one pack of cigarettes per day 4. 59-year-old African American with a BMI of 35 who has a high stress job

4. 59-year-old African American with a BMI of 35 who has a high stress job The patient has four risk factors for primary hypertension: advanced age, African American race, morbid obesity with a BMI of 35, and a high level of stress. All of the other patients have fewer risk factors for primary hypertension: in the 45-year-old smoker, smoking is the only risk factor; in the 60-year-old with cancer, advanced age and pain are the only risk factors; and in the 65-year-old retiree, the only risk factor is advanced age. Text Reference - p. 713

Which test result would indicate the presence of target organ damage resulting from uncontrolled hypertension? 1. Check for history of depression. 2. Do not give with grapefruit juice. 3. Monitor for cardiac dysrhythmias. 4. Assess for orthostatic hypotension

4. Assess for orthostatic hypotension Low blood pressure or postural hypotension can cause a fall from dizziness. The peripheral-acting alpha-adrenergic antagonist reserpine is contraindicated in patients with a history of depression. Administrating grapefruit juice with certain calcium channel blockers may increase the serum concentrations, resulting in toxicity. The direct vasodilator minoxidil may cause EKG changes of flattened and inverted T waves. Text Reference - p. 718

The patient with osteoporosis and hypertension understands dietary teaching when the patient selects which meal for dinner? 1. Ham and Swiss cheese sandwich on whole-wheat bread, steamed broccoli, and an apple 2. Baked chicken with one cup of yogurt and steamed rice 3. A two-egg omelet with 2 oz. of American cheese, one slice of whole-wheat toast, and half a grapefruit 4. Baked salmon with one cup of spinach and steamed carrots

4. Baked salmon with one cup of spinach and steamed carrots The highest calcium content is present in the dinner containing salmon and spinach, also taking into account fat and sodium restrictions required to manage hypertension. Ham and cheese are both high in sodium and should be avoided in the patient with hypertension. Eggs are not a large source of calcium, and chicken, yogurt, and rice, although lower in sodium, do not have the highest calcium content. Text Reference - p. 716

A nurse is measuring the blood pressure (BP) of a 68-year-old patient. What intervention should the nurse perform for this patient? 1. Measure BP one hour after eating. 2. Inflate the cuff until the pulse disappears. 3. Recommend a BP goal of 120/80 mm Hg. 4. Check for an auscultatory gap

4. Check for an auscultatory gap The nurse measuring the BP of a 68-year-old patient should check for an auscultatory gap. Some elderly patients have a wide gap between the first Korotkoff sound and subsequent beats. Elderly patients experience a postprandial drop in BP; the greatest drop occurs approximately one hour after eating. The BP returns to preprandial levels three to four hours after eating. When measuring BP, the nurse should inflate the cuff 20 to 30 mm Hg after the radial pulse disappears. The recommended BP goal for this patient would be less than 140/90 mm Hg. Text Reference - p. 725

Which item on the patient's dinner tray should not be taken in large quantities by the patient prescribed furosemide for hypertension? 1. Coffee 2. Ice cream 3. Grapefruit juice 4. Chicken noodle soup

4. Chicken noodle soup Furosemide, a diuretic, causes fluid loss to decrease blood pressure. Chicken noodle soup is high in sodium and may cause increased fluid retention, negating the effects of the medication and increasing the blood pressure. Ice cream, grapefruit juice, and coffee will not decrease the effectiveness of furosemide. Text Reference - p. 717

A nurse provides care to a patient who is admitted to an emergency department with hypertensive crisis. The patient had been taking sodium nitroprusside for the past three days. What is the reason that blood tests to assess thiocyanate levels are prescribed for this patient? 1. The patient may have very low BP due to the sodium nitroprusside. 2. The patient may have adverse effects on target organs. 3. The patient may have reduced excretion of sodium nitroprusside. 4. The patient may have toxic levels of sodium nitroprusside.

4. The patient may have toxic levels of sodium nitroprusside. Sodium nitroprusside causes arterial vasodilation and reduces systemic vascular resistance. This in turn decreases the blood pressure. Sodium nitroprusside is metabolized to cyanide and then to thiocyanate, which can reach lethal levels. Therefore, thiocyanate levels should be monitored in patients receiving the drug for more than three days or at doses greater than 4mcg/kg/min. In hypertensive crisis, the patient usually has very high BP despite the BP-lowering effect of sodium nitroprusside. Serum thiocyanate levels do not indicate adverse effects of hypertension on target organs or reduced excretion of sodium nitroprusside. Text Reference - p. 719

b. Late onset of puberty is not generally considered to be a risk factor for the development of atherosclerosis. Risk factors for atherosclerosis include family history of atherosclerosis, cigarette smoking, hypertension, high blood cholesterol level, male gender, diabetes mellitus, obesity, and physical inactivity

40] Which of the following is not a risk factor for the development of atherosclerosis? a. Family history of early heart attack b. Late onset of puberty c. Total blood cholesterol level greater than 220 mg/dL d. Elevated fasting blood glucose concentration

c. Nitroglycerin produces peripheral vasodilation, which reduces myocardial oxygen consumption and demand. Vasodilation in coronary arteries and collateral vessels may also increase blood flow to the ischemic areas of the heart. Nitroglycerin does not have an effect on pericardial spasticity or conductivity in the myocardium.

41] As an initial step in treating a client with angina, the physician prescribes nitroglycerin tablets, 0.3 mg given sublingually. This drug's principal effects are produced by: a. Antispasmodic effects on the pericardium b. Causing an increased myocardial oxygen demand c. Vasodilation of peripheral vasculature d. Improved conductivity in the myocardium

c. The correct protocol for nitroglycerin use involves immediate administration, with subsequent doses taken at 5-minute intervals as needed, for a total dose of three tablets. Sublingual nitroglycerin appears in the bloodstream within 2 to 3 minutes and is metabolized within about 10 minutes.

43] Sublingual nitroglycerine tablets begin to work within 1 to 2 minutes. How should the nurse instruct the client to use the drug when chest pain occurs? a. Take one tablet every 2 to 5 minutes until the pains stops b. Take one tablet and rest for 10 minutes. Call the physician if pain persists after 10 minutes c. Take one tablet, then an additional tablet every 5 minutes for a total of three tablets. Call the physician if pain persists after three tablets d. Take one tablet. If pain persists after 5 minutes, take two tablets. If pain still persists 5 minutes later, call the physician

Physiological Integrity COMPLETION 1. When analyzing an electrocardiographic (ECG) rhythm strip of a patient with a regular heart rhythm, the nurse counts 30 small blocks from one R wave to the next. The nurse calculates the patient's heart rate as ____.

50 There are 1500 small blocks in a minute, and the nurse will divide 1500 by 30. DIF: Cognitive Level: Remember (knowledge) REF: 789-790 TOP: Nursing Process: Assessment MSC:

b. A low urine output and confusion are signs of decreased tissue perfusion. Orthopnea is a sign of left-sided heart failure. Crackles, edema and weight gain should be monitored closely, but the levels are not as high a priority. With atrial fibrillation there is a loss of atrial kick, but the blood pressure and heart rate are stable

7] The nurse has completed an assessment on a client with a decreased cardiac output. Which findings should receive highest priority? a. BP 110/62, atrial fibrillation with HR 82, bibasilar crackles b. Confusion, urine output 15mL over the last 2 hours, orthopnea. c. SpO2 92 on 2 liters nasal cannula, respirations 20, 1+ edema of lower extremities. d. Weight gain of 1kg in 3 days, BP 130/80, mild dyspnea with exercise.

The nurse is assessing the blood pressure of a client who is experiencing cardiogenic shock. Which of the following blood pressure readings is most likely?

90/75

Physiological Integrity 25. Which action will the nurse include in the plan of care for a patient who was admitted with syncopal episodes of unknown origin? a. Instruct the patient to call for assistance before getting out of bed. b. Explain the association between various dysrhythmias and syncope. c. Educate the patient about the need to avoid caffeine and other stimulants. d. Tell the patient about the benefits of implantable cardioverter-defibrillators.

A A patient with fainting episodes is at risk for falls. The nurse will plan to minimize the risk by having assistance whenever the patient up. The other actions may be needed if dysrhythmias are found to be the cause of the patient's syncope, but are not appropriate for syncope of unknown origin. DIF: Cognitive Level: Apply (application) REF: 807 TOP: Nursing Process: Planning MSC:

Safe and Effective Care Environment 27. Which action by a new registered nurse (RN) who is orienting to the progressive care unit indicates a good understanding of the treatment of cardiac dysrhythmias? a. Injects IV adenosine (Adenocard) over 2 seconds to a patient with supraventricular tachycardia b. Obtains the defibrillator and quickly brings it to the bedside of a patient whose monitor shows asystole c. Turns the synchronizer switch to the "on" position before defibrillating a patient with ventricular fibrillation d. Gives the prescribed dose of diltiazem (Cardizem) to a patient with new-onset type II second degree AV block

A Adenosine must be given over 1 to 2 seconds to be effective. The other actions indicate a need for more education about treatment of cardiac dysrhythmias. The RN should hold the diltiazem until talking to the health care provider. The treatment for asystole is immediate CPR. The synchronizer switch should be "off" when defibrillating. DIF: Cognitive Level: Analyze (analysis) REF: 795 OBJ: Special Questions: Multiple Patients TOP: Nursing Process: Evaluation MSC:

Physiological Integrity 9. A patient with dilated cardiomyopathy has new onset atrial fibrillation that has been unresponsive to drug therapy for several days. The priority teaching needed for this patient would include information about a. anticoagulant therapy. b. permanent pacemakers. c. electrical cardioversion. d. IV adenosine (Adenocard).

A Atrial fibrillation therapy that has persisted for more than 48 hours requires anticoagulant treatment for 3 weeks before attempting cardioversion. This is done to prevent embolization of clots from the atria. Cardioversion may be done after several weeks of anticoagulation therapy. Adenosine is not used to treat atrial fibrillation. Pacemakers are routinely used for patients with bradydysrhythmias. Information does not indicate that the patient has a slow heart rate. DIF: Cognitive Level: Apply (application) REF: 796 OBJ: Special Questions: Prioritization TOP: Nursing Process: Planning MSC:

Physiological Integrity 8. After the nurse gives IV atropine to a patient with symptomatic type 1, second-degree atrioventricular (AV) block, which finding indicates that the medication has been effective? a. Increase in the patient's heart rate b. Increase in strength of peripheral pulses c. Decrease in premature atrial contractions d. Decrease in premature ventricular contractions

A Atropine will increase the heart rate and conduction through the AV node. Because the medication increases electrical conduction, not cardiac contractility, the quality of the peripheral pulses is not used to evaluate the drug effectiveness. The patient does not have premature atrial or ventricular contractions. DIF: Cognitive Level: Apply (application) REF: 798 TOP: Nursing Process: Evaluation MSC:

Physiological Integrity 14. A 20-year-old has a mandatory electrocardiogram (ECG) before participating on a college soccer team and is found to have sinus bradycardia, rate 52. Blood pressure (BP) is 114/54, and the student denies any health problems. What action by the nurse is most appropriate? a. Allow the student to participate on the soccer team. b. Refer the student to a cardiologist for further diagnostic testing. c. Tell the student to stop playing immediately if any dyspnea occurs. d. Obtain more detailed information about the student's family health history.

A In an aerobically trained individual, sinus bradycardia is normal. The student's normal BP and negative health history indicate that there is no need for a cardiology referral or for more detailed information about the family's health history. Dyspnea during an aerobic activity such as soccer is normal. DIF: Cognitive Level: Apply (application) REF: 793 TOP: Nursing Process: Implementation MSC:

6. A patient in the intensive care unit with ADHF complains of severe dyspnea and is anxious, tachypneic, and tachycardic. All these medications have been ordered for the patient. The first action by the nurse will be to a. administer IV morphine sulfate 2 mg. b. give IV diazepam (Valium) 2.5 mg. c. increase dopamine (Intropin) infusion by 2 mcg/kg/min. d. increase nitroglycerin (Tridil) infusion by 5 mcg/min.

A Rationale: Morphine improves alveolar gas exchange, improves cardiac output by reducing ventricular preload and afterload, decreases anxiety, and assists in reducing the subjective feeling of dyspnea. Diazepam may decrease patient anxiety, but it will not improve the cardiac output or gas exchange. Increasing the dopamine may improve cardiac output but will also increase the heart rate and myocardial oxygen consumption. Nitroglycerin will improve cardiac output and may be appropriate for this patient, but it will not directly reduce anxiety and will not act as quickly as morphine to decrease dyspnea. Cognitive Level: Analysis Text Reference: pp. 828-829 Nursing Process: Implementation NCLEX: Physiological Integrity

21. A patient with ADHF who is receiving nesiritide (Natrecor) asks the nurse how the medication will work to help improve the symptoms of dyspnea and orthopnea. The nurse's reply will be based on the information that nesiritide will a. dilate arterial and venous blood vessels, decreasing ventricular preload and afterload. b. improve the ability of the ventricular myocardium to contract, strengthening contractility. c. enhance the speed of impulse conduction through the heart, increasing the heart rate. d. increase calcium sensitivity in vascular smooth muscle, boosting systemic vascular resistance.

A Rationale: Nesiritide, a recombinant form of BNP, causes both arterial and venous vasodilation, leading to reductions in preload and afterload. Inotropic medications, such as dopamine and dobutamine, may be used in ADHF to improve ventricular contractility. Nesiritide does not increase impulse conduction or calcium sensitivity in the heart. Cognitive Level: Application Text Reference: p. 829 Nursing Process: Implementation NCLEX: Physiological Integrity

19. An outpatient who has developed heart failure after having an acute myocardial infarction has a new prescription for carvedilol (Coreg). After 2 weeks, the patient returns to the clinic. The assessment finding that will be of most concern to the nurse is that the patient a. has BP of 88/42. b. has an apical pulse rate of 56. c. complains of feeling tired. d. has 2+ pedal edema.

A Rationale: The patient's BP indicates that the dose of carvedilol may need to be decreased because the mean arterial pressure is only 57. Bradycardia is a frequent adverse effect of -Adrenergic blockade, but the rate of 56 is not as great a concern as the hypotension. -adrenergic blockade will initially worsen symptoms of heart failure in many patients, and patients should be taught that some increase in symptoms, such as fatigue and edema, is expected during the initiation of therapy with this class of drugs. Cognitive Level: Application Text Reference: p. 832 Nursing Process: Assessment NCLEX: Analysis

1. A patient with a history of chronic heart failure is admitted to the emergency department with severe dyspnea and a dry, hacking cough. The patient has pitting edema in both ankles, blood pressure (BP) of 170/100, an apical pulse rate of 92, and respirations 28. The most important assessment for the nurse to accomplish next is to a. auscultate the lung sounds. b. assess the orientation. c. check the capillary refill. d. palpate the abdomen.

A Rationale: When caring for a patient with severe dyspnea, the nurse should use the ABCs to guide initial care. This patient's severe dyspnea and cough indicate that acute decompensated heart failure (ADHF) is occurring. ADHF usually manifests as pulmonary edema, which should be detected and treated immediately to prevent ongoing hypoxemia and cardiac/respiratory arrest. The other assessments will provide useful data about the patient's volume status and should also be accomplished rapidly, but detection (and treatment) of fluid-filled alveoli is the priority. Cognitive Level: Application Text Reference: pp. 824-825 Nursing Process: Assessment NCLEX: Physiological Integrity

A client with HF has an order for lisnopril (Prinivil, Zestril) Which of the following conditions in the client's history would lead a nurse to confirm the order with the provider? 1. A history of HT previously treated with diuretics. 2. A history of seasonal allergies currently treated with antihistamines. 3. A history of angioedema after taking enalapril (Vasotec) 4. A history of alcoholism, currently abstaining.

A history of angioedema after taking enalapril (Vasotec)

1. To determine whether there is a delay in impulse conduction through the atria, the nurse will measure the duration of the patient's a. P wave. b. Q wave. c. P-R interval. d. QRS complex.

A The P wave represents the depolarization of the atria. The P-R interval represents depolarization of the atria, atrioventricular (AV) node, bundle of His, bundle branches, and the Purkinje fibers. The QRS represents ventricular depolarization. The Q wave is the first negative deflection following the P wave and should be narrow and short. DIF: Cognitive Level: Understand (comprehension) REF: 791 TOP: Nursing Process: Assessment MSC:

Physiological Integrity 12. Which intervention by a new nurse who is caring for a patient who has just had an implantable cardioverter-defibrillator (ICD) inserted indicates a need for more education about care of patients with ICDs? a. The nurse assists the patient to do active range of motion exercises for all extremities. b. The nurse assists the patient to fill out the application for obtaining a Medic Alert ID. c. The nurse gives amiodarone (Cordarone) to the patient without first consulting with the health care provider. d. The nurse teaches the patient that sexual activity usually can be resumed once the surgical incision is healed.

A The patient should avoid moving the arm on the ICD insertion site until healing has occurred in order to prevent displacement of the ICD leads. The other actions by the new nurse are appropriate for this patient. DIF: Cognitive Level: Apply (application) REF: 803 TOP: Nursing Process: Evaluation MSC:

Physiological Integrity 18. A patient's cardiac monitor shows a pattern of undulations of varying contours and amplitude with no measurable ECG pattern. The patient is unconscious and pulseless. Which action should the nurse take first? a. Perform immediate defibrillation. b. Give epinephrine (Adrenalin) IV. c. Prepare for endotracheal intubation. d. Give ventilations with a bag-valve-mask device.

A The patient's rhythm and assessment indicate ventricular fibrillation and cardiac arrest; the initial action should be to defibrillate. If a defibrillator is not immediately available or is unsuccessful in converting the patient to a better rhythm, the other actions may be appropriate. DIF: Cognitive Level: Apply (application) REF: 801 OBJ: Special Questions: Prioritization TOP: Nursing Process: Implementation MSC:

At a clinic visit, the nurse provides dietary teaching for a 56-year-old woman who was recently hospitalized with an exacerbation of chronic heart failure. The nurse determines that teaching is successful if the patient makes which statement? A "I will limit the amount of milk and cheese in my diet." B "I can add salt when cooking foods but not at the table." C "I will take an extra diuretic pill when I eat a lot of salt." D "I can have unlimited amounts of foods labeled as reduced sodium ."

A "I will limit the amount of milk and cheese in my diet." Milk products should be limited to 2 cups per day for a 2500-mg sodium-restricted diet. Salt should not be added during food preparation or at the table. Diuretics should be taken as prescribed (usually daily) and not based on sodium intake. Foods labeled as reduced sodium contain at least 25% less sodium than regular.

A70-year-old woman with chronic heart failure and atrial fibrillation asks the nurse why warfarin (Coumadin) has been prescribed for her to continue at home. Which response by the nurse is accurate? A "The medication prevents blood clots from forming in your heart." B "The medication dissolves clots that develop in your coronary arteries." C "The medication reduces clotting by decreasing serum potassium levels." D "The medication increases your heart rate so that clots do not form in your heart."

A "The medication prevents blood clots from forming in your heart." Chronic heart failure causes enlargement of the chambers of the heart and an altered electrical pathway, especially in the atria. When numerous sites in the atria fire spontaneously and rapidly, atrial fibrillation occurs. Atrial fibrillation promotes thrombus formation within the atria with an increased risk of stroke and requires treatment with cardioversion, antidysrhythmics, and/or anticoagulants. Warfarin is an anticoagulant that interferes with hepatic synthesis of vitamin K-dependent clotting factors.

The evaluation team for cardiac transplantation is evaluating patients. Which patient is most likely to receive the most benefit from a new heart?

A 52 year old woman with end-stage coronary artery disease who has limited financial resources but is emotionally stable and has strong social support

A patient admitted with heart failure appears very anxious and complains of shortness of breath. Which nursing actions would be appropriate to alleviate this patient's anxiety (select all that apply)? A Administer ordered morphine sulfate. B Position patient in a semi-Fowler's position. C Position patient on left side with head of bed flat. D Instruct patient on the use of relaxation techniques. E Use a calm, reassuring approach while talking to patient.

A Administer ordered morphine sulfate. B Position patient in a semi-Fowler's position. D Instruct patient on the use of relaxation techniques. E Use a calm, reassuring approach while talking to patient. Morphine sulfate reduces anxiety and may assist in reducing dyspnea. The patient should be positioned in semi-Fowler's position to improve ventilation that will reduce anxiety. Relaxation techniques and a calm reassuring approach will also serve to reduce anxiety.

The patient has heart failure (HF) with an ejection fraction of less than 40%. What core measures should the nurse expect to include in the plan of care for this patient (select all that apply)? A Left ventricular function is documented. B Controlling dysrhythmias will eliminate HF. C Prescription for digoxin (Lanoxin) at discharge D Prescription for angiotensin-converting enzyme (ACE) inhibitor at discharge E Education materials about activity, medications, weight monitoring, and what to do if symptoms worsen

A Left ventricular function is documented. D Prescription for angiotensin-converting enzyme (ACE) inhibitor at discharge E Education materials about activity, medications, weight monitoring, and what to do if symptoms worsen The Joint Commission has identified these three core measures for heart failure patients. Although controlling dysrhythmias will improve CO and workload, it will not eliminate HF. Prescribing digoxin for all HF patients is no longer done because there are newer effective drugs and digoxin toxicity occurs easily related to electrolyte levels and the therapeutic range must be maintained.

A patient diagnosed with heart failure has a pulmonary artery catheter (PAC) in place. What information about the patient's hemodynamic functioning will the healthcare provider obtain from this monitoring device? A Left ventricular functioning B Pulmonary valve function C Coronary artery patency D Stroke volume

A Left ventricular functioning

A patient with a recent diagnosis of heart failure has been prescribed furosemide (Lasix) in an effort to physiologically do what for the patient? A Reduce preload. B Decrease afterload. C Increase contractility. D Promote vasodilation.

A Reduce preload. Diuretics such as furosemide are used in the treatment of HF to mobilize edematous fluid, reduce pulmonary venous pressure, and reduce preload. They do not directly influence afterload, contractility, or vessel tone.

The patient with chronic heart failure is being discharged from the hospital. What information should the nurse emphasize in the patient's discharge teaching to prevent progression of the disease to ADHF? A Take medications as prescribed. B Use oxygen when feeling short of breath. C Only ask the physician's office questions. D Encourage most activity in the morning when rested.

A Take medications as prescribed. The goal for the patient with chronic HF is to avoid exacerbations and hospitalization. Taking the medications as prescribed along with nondrug therapies such as alternating activity with rest will help the patient meet this goal. If the patient needs to use oxygen at home, it will probably be used all the time or with activity to prevent respiratory acidosis. Many HF patients are monitored by a care manager or in a transitional program to assess the patient for medication effectiveness and monitor for patient deterioration and encourage the patient. This nurse manager can be asked questions or can contact the health care provider if there is evidence of worsening HF.

A Prinzmetal's or variant angina is atypical angina that occurs unpredictably (unrelated to activity) and often at night. It is caused by coronary artery spasm with or without an atherosclerotic lesion. This client is at risk for silent ischemia and is associated with a higher relative risk of serious or fatal cardiac events. Stable angina occurs with exertion and is relieved with rest and nitroglycerin tablets. Unstable angina occurs with increasing frequency, severity, and duration. Pain is unpredictable, occurs with decreasing levels of activity or stress, and may occur at rest. It is a precursor to myocardial infarction. The client has classic symptoms of variant angina and so is not experiencing nonanginal pain. Variant angina is prolonged, severe, and occurs at the same time every day, usually in the morning.

A client tells the nurse that the anginal pain is unpredictable but usually occurs at night. The nurse questions the client and family further about precipitating factors at home; however, there does not appear to be any environmental or emotional cause for the pain. The nurse concludes that the client should contact the physician to facilitate admission to the hospital, because the client has which type of angina? A) Stable angina B) Variant angina C) Unstable angina D) Nonanginal pain

Which statements accurately describe heart failure

A common cause of diastolic failure is left ventricular hypertrophy A primary risk factor for heart failure is CAD

Which action will the nurse include in the plan of care for a 72-year-old woman admitted with multiple myeloma? a. Monitor fluid intake and output. b. Administer calcium supplements. c. Assess lymph nodes for enlargement. d. Limit weight bearing and ambulation.

A high fluid intake and urine output helps prevent the complications of kidney stones caused by hypercalcemia and renal failure caused by deposition of Bence-Jones protein in the renal tubules. Weight bearing and ambulation are encouraged to help bone retain calcium. Lymph nodes are not enlarged with multiple myeloma. Calcium supplements will further increase the patient's calcium level and are not used

C

A patient has been admitted to the cardiac unit with a diagnosis of right ventricular failure. Which of the following assessment findings would the healthcare provider expect to observe? is most likely to be observed by the healthcare provider? A) Fatigue and hemoptysis B) Bradycardia and circumoral cyanosis C) Peripheral edema and jugular vein distension D) Dyspnea and pulmonary crackles

D) Edema in the feet is common for Right ventricular failure

A patient who has a history of pulmonary valve stenosis tells the healthcare provider, "I don't have a lot of energy anymore, and both of my feet get swollen in the late afternoon." Which of these problems does the healthcare provider conclude is the likely cause of these clinical findings? Please choose from one of the following options. A) Acute pericarditis B) Deep vein thrombosis (DVT) C) Peripheral artery disease D) Right ventricular failure

avoid standing near anti theft devices in doorways

A patient with a sinus node dysfunction has a permanent pacemaker inserted. Before discharge, what should the nurse include when teaching the patient?

an increase in infarct size

A patient with an acute MI has sinus tachycardia of 126 bpm. The nurse recognizes that if this dysrhthymia is not treated, the patient is likely to experience?

ST segment elevation

A with patient with chest pain that is unrelieved by nitroglycerin is admitted to the coronary care unit for observation and diagnosis. While the patient has continuous ECG monitoring, what finding would most concern the nurse?

A patient admitted with heart failure appears very anxious and complains of shortness of breath. Which nursing actions would be appropriate to alleviate this patient's anxiety (select all that apply)? A. Administer ordered morphine sulfate. B. Position patient in a semi-Fowler's position. C. Position patient on left side with head of bed flat. D. Instruct patient on the use of relaxation techniques. E. Use a calm, reassuring approach while talking to patient.

A, B, D, E. Morphine sulfate reduces anxiety and may assist in reducing dyspnea. The patient should be positioned in semi-Fowler's position to improve ventilation that will reduce anxiety. Relaxation techniques and a calm reassuring approach will also serve to reduce anxiety.

Physiological Integrity OTHER 1. When preparing to defibrillate a patient. In which order will the nurse perform the following steps? (Put a comma and a space between each answer choice [A, B, C, D, E].) a. Turn the defibrillator on. b. Deliver the electrical charge. c. Select the appropriate energy level. d. Place the paddles on the patient's chest. e. Check the location of other staff and call out "all clear."

A, C, D, E, B This order will result in rapid defibrillation without endangering hospital staff. DIF: Cognitive Level: Analyze (analysis) REF: 802 TOP: Nursing Process: Implementation MSC:

The patient has heart failure (HF) with an ejection fraction of less than 40%. What core measures should the nurse expect to include in the plan of care for this patient (select all that apply)? A. Left ventricular function is documented. B. Controlling dysrhythmias will eliminate HF. C. Prescription for digoxin (Lanoxin) at discharge D. Prescription for angiotensin-converting enzyme (ACE) inhibitor at discharge E. Education materials about activity, medications, weight monitoring, and what to do if symptoms worsen

A, D, E. The Joint Commission has identified these three core measures for heart failure patients. Although controlling dysrhythmias will improve CO and workload, it will not eliminate HF. Prescribing digoxin for all HF patients is no longer done because there are newer effective drugs and digoxin toxicity occurs easily related to electrolyte levels and the therapeutic range must be maintained.

A70-year-old woman with chronic heart failure and atrial fibrillation asks the nurse why warfarin (Coumadin) has been prescribed for her to continue at home. Which response by the nurse is accurate? A. "The medication prevents blood clots from forming in your heart." B. "The medication dissolves clots that develop in your coronary arteries." C. "The medication reduces clotting by decreasing serum potassium levels." D. "The medication increases your heart rate so that clots do not form in your heart."

A. "The medication prevents blood clots from forming in your heart." Chronic heart failure causes enlargement of the chambers of the heart and an altered electrical pathway, especially in the atria. When numerous sites in the atria fire spontaneously and rapidly, atrial fibrillation occurs. Atrial fibrillation promotes thrombus formation within the atria with an increased risk of stroke and requires treatment with cardioversion, antidysrhythmics, and/or anticoagulants. Warfarin is an anticoagulant that interferes with hepatic synthesis of vitamin K-dependent clotting factors.

A patient with a recent diagnosis of heart failure has been prescribed furosemide (Lasix) in an effort to physiologically do what for the patient? A. Reduce preload. B. Decrease afterload. C. Increase contractility. D. Promote vasodilation.

A. Reduce preload. Diuretics such as furosemide are used in the treatment of HF to mobilize edematous fluid, reduce pulmonary venous pressure, and reduce preload. They do not directly influence afterload, contractility, or vessel tone.

The patient with chronic heart failure is being discharged from the hospital. What information should the nurse emphasize in the patient's discharge teaching to prevent progression of the disease to ADHF? A. Take medications as prescribed. B. Use oxygen when feeling short of breath. C. Only ask the physician's office questions. D. Encourage most activity in the morning when rested.

A. Take medications as prescribed. The goal for the patient with chronic HF is to avoid exacerbations and hospitalization. Taking the medications as prescribed along with nondrug therapies such as alternating activity with rest will help the patient meet this goal. If the patient needs to use oxygen at home, it will probably be used all the time or with activity to prevent respiratory acidosis. Many HF patients are monitored by a care manager or in a transitional program to assess the patient for medication effectiveness and monitor for patient deterioration and encourage the patient. This nurse manager can be asked questions or can contact the health care provider if there is evidence of worsening HF.

A. Atrial fibrillation B. Atrial Flutter

A. top B. bottom

Lisinopril, captopril, enalapril, ramipril

ACE - inhibitors

All of the following might be considered helpful in the treatment of heart failure EXCEPT for: ADH/vasopressin a Beta 1 adrenergic receptor antagonist AT II receptor antagonists an aldosterone blocking diuretic an ACE inhibitor

ADH/vasopressin

The nurse obtains a blood pressure of 176/83 mm Hg for a patient. What is the patient's mean arterial pressure (MAP)?

ANS: 114 mm Hg MAP = (SBP + 2 DBP)/3

A patient is to receive an infusion of 250 mL of platelets over 2 hours through tubing that is labeled: 1 mL equals 10 drops. How many drops per minute will the nurse infuse?

ANS: 21 To infuse 250 mL over 2 hours, the calculated drip rate is 20.8 drops/minute or 21 drops/minute

Which assessment finding in a patient admitted with acute decompensated heart failure (ADHF) requires the most immediate action by the nurse? a. O2 saturation of 88% b. Weight gain of 1 kg (2.2 lb) c. Heart rate of 106 beats/min d. Urine output of 50 mL over 2 hours

ANS: A A decrease in O2 saturation to less than 92% indicates hypoxemia, and the nurse should start supplemental O2 immediately. An increase in apical pulse rate, 1-kg weight gain, and decreases in urine output also indicate worsening heart failure and require nursing actions, but the low O2 saturation rate requires the most immediate nursing action.

Which assessment finding in a patient admitted with acute decompensated heart failure (ADHF) requires the most immediate action by the nurse? a. Oxygen saturation of 88% b. Weight gain of 1 kg (2.2 lb) c. Heart rate of 106 beats/minute d. Urine output of 50 mL over 2 hours

ANS: A A decrease in oxygen saturation to less than 92% indicates hypoxemia. The nurse should administer supplemental oxygen immediately to the patient. An increase in apical pulse rate, 1-kg weight gain, and decreases in urine output also indicate worsening heart failure and require nursing actions, but the low oxygen saturation rate requires the most immediate nursing action.

The nurse has just finished teaching a hypertensive patient about the newly prescribed ramipril (Altace). Which patient statement indicates that more teaching is needed? a. "A little swelling around my lips and face is okay." b. "The medication may not work as well if I take any aspirin." c. "The doctor may order a blood potassium level occasionally." d. "I will call the doctor if I notice that I have a frequent cough."

ANS: A Angioedema occurring with angiotensin-converting enzyme (ACE) inhibitor therapy is an indication that the ACE inhibitor should be discontinued. The patient should be taught that if any swelling of the face or oral mucosa occurs, the health care provider should be immediately notified because this could be life threatening. The other patient statements indicate that the patient has an accurate understanding of ACE inhibitor therapy

An appropriate nursing intervention for a patient with non-Hodgkin's lymphoma whose platelet count drops to 18,000/µL during chemotherapy is to a. check all stools for occult blood. b. encourage fluids to 3000 mL/day. c. provide oral hygiene every 2 hours. d. check the temperature every 4 hours.

ANS: A Because the patient is at risk for spontaneous bleeding, the nurse should check stools for occult blood. A low platelet count does not require an increased fluid intake. Oral hygiene is important, but it is not necessary to provide oral care every 2 hours. The low platelet count does not increase risk for infection, so frequent temperature monitoring is not indicated

Which menu choice indicates that the patient understands the nurse's teaching about best dietary choices for iron-deficiency anemia? a. Omelet and whole wheat toast b. Cantaloupe and cottage cheese c. Strawberry and banana fruit plate d. Cornmeal muffin and orange juice

ANS: A Eggs and whole grain breads are high in iron. The other choices are appropriate for other nutritional deficiencies but are not the best choice for a patient with iron-deficiency anemia

Which information obtained by the nurse assessing a patient admitted with multiple myeloma is most important to report to the health care provider? a. Serum calcium level is 15 mg/dL. b. Patient reports no stool for 5 days. c. Urine sample has Bence-Jones protein. d. Patient is complaining of severe back pain.

ANS: A Hypercalcemia may lead to complications such as dysrhythmias or seizures, and should be addressed quickly. The other patient findings will also be discussed with the health care provider, but are not life threatening

Which patient statement to the nurse indicates a need for additional instruction about taking oral ferrous sulfate? a. "I will call my health care provider if my stools turn black." b. "I will take a stool softener if I feel constipated occasionally." c. "I should take the iron with orange juice about an hour before eating." d. "I should increase my fluid and fiber intake while I am taking iron tablets."

ANS: A It is normal for the stools to appear black when a patient is taking iron, and the patient should not call the doctor about this. The other patient statements are correct

A patient in the intensive care unit with acute decompensated heart failure (ADHF) complains of severe dyspnea and is anxious, tachypneic, and tachycardic. All of the following medications have been ordered for the patient. The nurse's priority action will be to a. give IV morphine sulfate 4 mg. b. give IV diazepam (Valium) 2.5 mg. c. increase nitroglycerin (Tridil) infusion by 5 mcg/min. d. increase dopamine (Intropin) infusion by 2 mcg/kg/min.

ANS: A Morphine improves alveolar gas exchange, improves cardiac output by reducing ventricular preload and afterload, decreases anxiety, and assists in reducing the subjective feeling of dyspnea. Diazepam may decrease patient anxiety, but it will not improve the cardiac output or gas exchange. Increasing the dopamine may improve cardiac output, but it will also increase the heart rate and myocardial oxygen consumption. Nitroglycerin will improve cardiac output and may be appropriate for this patient, but it will not directly reduce anxiety and will not act as quickly as morphine to decrease dyspnea.

A patient in the intensive care unit with acute decompensated heart failure (ADHF) complains of severe dyspnea and is anxious, tachypneic, and tachycardic. Several drugs have been ordered for the patient. The nurse's priority action will be to a. give PRN IV morphine sulfate 4 mg. b. give PRN IV diazepam (Valium) 2.5 mg. c. increase nitroglycerin infusion by 5 mcg/min. d. increase dopamine infusion by 2 mcg/kg/min.

ANS: A Morphine improves alveolar gas exchange, improves cardiac output by reducing ventricular preload and afterload, decreases anxiety, and assists in reducing the subjective feeling of dyspnea. Diazepam may decrease patient anxiety, but it will not improve the cardiac output or gas exchange. Increasing the dopamine may improve cardiac output, but it will also increase the heart rate and myocardial oxygen consumption. Nitroglycerin will improve cardiac output and may be appropriate for this patient, but it will not directly reduce anxiety and will not act as quickly as morphine to decrease dyspnea.

Which action should the nurse include in the plan of care when caring for a patient admitted with acute decompensated heart failure (ADHF) who is receiving nesiritide (Natrecor)? a. Monitor blood pressure frequently. b. Encourage patient to ambulate in room. c. Titrate nesiritide slowly before stopping. d. Teach patient about home use of the drug.

ANS: A Nesiritide is a potent arterial and venous dilator, and the major adverse effect is hypotension. Because the patient is likely to have orthostatic hypotension, the patient should not be encouraged to ambulate. Nesiritide does not require titration and is used for ADHF but not in a home setting.

Propranolol (Inderal) is prescribed for a patient diagnosed with hypertension. The nurse should consult with the health care provider before giving this medication when the patient reveals a history of a. asthma. b. daily alcohol use. c. peptic ulcer disease. d. myocardial infarction (MI).

ANS: A Nonselective b-blockers block b1- and b2-adrenergic receptors and can cause bronchospasm, especially in patients with a history of asthma. b-Blockers will have no effect on the patient's peptic ulcer disease or alcohol use. b-Blocker therapy is recommended after MI

A patient is diagnosed with hypertension and nadolol (Corgard) is prescribed. The nurse should consult with the health care provider before giving this medication upon finding a history of a. asthma. b. peptic ulcer disease. c. alcohol dependency. d. myocardial infarction (MI).

ANS: A Nonselective β-blockers block β1- and β2-adrenergic receptors and can cause bronchospasm, especially in patients with a history of asthma. β-blockers will have no effect on the patient's peptic ulcer disease or alcohol dependency. β-blocker therapy is recommended after MI.

Which patient should the nurse assign as the roommate for a patient who has aplastic anemia? a. A patient with chronic heart failure b. A patient who has viral pneumonia c. A patient who has right leg cellulitis d. A patient with multiple abdominal drains

ANS: A Patients with aplastic anemia are at risk for infection because of the low white blood cell production associated with this type of anemia, so the nurse should avoid assigning a roommate with any possible infectious process

A 19-year-old woman with immune thrombocytopenic purpura (ITP) has an order for a platelet transfusion. Which information indicates that the nurse should consult with the health care provider before obtaining and administering platelets? a. The platelet count is 42,000/mL. b. Petechiae are present on the chest. c. Blood pressure (BP) is 94/56 mm Hg. d. Blood is oozing from the venipuncture site.

ANS: A Platelet transfusions are not usually indicated until the platelet count is below 10,000 to 20,000/mL unless the patient is actively bleeding. Therefore the nurse should clarify the order with the health care provider before giving the transfusion. The other data all indicate that bleeding caused by ITP may be occurring and that the platelet transfusion is appropriate

The charge nurse observes a new registered nurse (RN) doing discharge teaching for a patient with hypertension who has a new prescription for enalapril (Vasotec). The charge nurse will need to intervene if the new RN tells the patient to a. increase the dietary intake of high-potassium foods. b. make an appointment with the dietitian for teaching. c. check the blood pressure (BP) with a home BP monitor at least once a day. d. move slowly when moving from lying to sitting to standing.

ANS: A The ACE inhibitors cause retention of potassium by the kidney, so hyperkalemia is a possible adverse effect. The other teaching by the new RN is appropriate for a patient with newly diagnosed hypertension who has just started therapy with enalapril

The charge nurse observes a new registered nurse (RN) doing discharge teaching for a patient with hypertension who has a new prescription for enalapril (Vasotec). The charge nurse will need to intervene if the new RN tells the patient to a. increase the dietary intake of high-potassium foods. b. make an appointment with the dietitian for teaching. c. check the blood pressure (BP) at home at least once a day. d. move slowly when moving from lying to sitting to standing.

ANS: A The ACE inhibitors cause retention of potassium by the kidney, so hyperkalemia is a possible adverse effect. The other teaching by the new RN is appropriate for a patient with newly diagnosed hypertension who has just started therapy with enalapril.

The nurse is reviewing the laboratory test results for a patient who has recently been diagnosed with hypertension. Which result is most important to communicate to the health care provider? a. Serum creatinine of 2.8 mg/dL b. Serum potassium of 4.5 mEq/L c. Serum hemoglobin of 14.7 g/dL d. Blood glucose level of 96 mg/dL

ANS: A The elevated creatinine indicates renal damage caused by the hypertension. The other laboratory results are normal

The nurse is reviewing the laboratory tests for a patient who has recently been diagnosed with hypertension. Which result is most important to communicate to the health care provider? a. Serum creatinine of 2.6 mg/dL b. Serum potassium of 3.8 mEq/L c. Serum hemoglobin of 14.7 g/dL d. Blood glucose level of 98 mg/dL

ANS: A The elevated creatinine indicates renal damage caused by the hypertension. The other laboratory results are normal.

The nurse is reviewing the laboratory test results for a patient who has recently been diagnosed with hypertension. Which result is *most* important to communicate to the health care provider? a. Serum creatinine of 2.8 mg/dL c. Serum hemoglobin of 14.7 g/dL b. Serum potassium of 4.5 mEq/L d. Blood glucose level of 96 mg/dL

ANS: A The elevated serum creatinine indicates renal damage caused by the hypertension. The other laboratory results are normal.

The nurse caring for a patient with type A hemophilia being admitted to the hospital with severe pain and swelling in the right knee will a. immobilize the joint. b. apply heat to the knee. c. assist the patient with light weight bearing. d. perform passive range of motion to the knee.

ANS: A The initial action should be total rest of the knee to minimize bleeding. Ice packs are used to decrease bleeding. Range of motion (ROM) and weight-bearing exercise are contraindicated initially, but after the bleeding stops, ROM and physical therapy are started

Which nursing action should the nurse take first in order to assist a patient with newly diagnosed stage 1 hypertension in making needed dietary changes? a. Have the patient record dietary intake for 3 days. b. Give the patient a detailed list of low-sodium foods. c. Teach the patient about foods that are high in sodium. d. Help the patient make an appointment with a dietitian.

ANS: A The initial nursing action should be assessment of the patient's baseline dietary intake through a 3-day food diary. The other actions may be appropriate, but assessment of the patient's baseline should occur first.

Which nursing action should the nurse take first in order to assist a patient with newly diagnosed stage 1 hypertension in making needed dietary changes? a. Collect a detailed diet history. b. Provide a list of low-sodium foods. c. Help the patient make an appointment with a dietitian. d. Teach the patient about foods that are high in potassium.

ANS: A The initial nursing action should be assessment of the patient's baseline dietary intake through a thorough diet history. The other actions may be appropriate, but assessment of the patient's baseline should occur first

Which nursing action should the nurse take *first* to assist a patient with newly diagnosed stage 1 hypertension in making needed dietary changes? a. Collect a detailed diet history. b. Provide a list of low-sodium foods. c. Help the patient make an appointment with a dietitian. d. Teach the patient about foods that are high in potassium.

ANS: A The initial nursing action should be assessment of the patient's baseline dietary intake through a thorough diet history. The other actions may be appropriate, but assessment of the patient's baseline should occur first.

After receiving change-of-shift report on a heart failure unit, which patient should the nurse assess first? a. A patient who is cool and clammy, with new-onset confusion and restlessness b. A patient who has crackles bilaterally in the lung bases and is receiving oxygen. c. A patient who had dizziness after receiving the first dose of captopril (Capoten) d. A patient who is receiving IV nesiritide (Natrecor) and has a blood pressure of 100/62

ANS: A The patient who has "wet-cold" clinical manifestations of heart failure is perfusing inadequately and needs rapid assessment and changes in management. The other patients also should be assessed as quickly as possible but do not have indications of severe decreases in tissue perfusion.

The nurse on the intermediate care unit received change-of-shift report on four patients with hypertension. Which patient should the nurse assess *first*? a. 48-yr-old with a blood pressure of 160/92 mm Hg who reports chest pain b. 52-yr-old with a blood pressure of 198/90 mm Hg who has intermittent claudication c. 50-yr-old with a blood pressure of 190/104 mm Hg who has a creatinine of 1.7 mg/dL d. 43-yr-old with a blood pressure of 172/98 mm Hg whose urine shows microalbuminuria

ANS: A The patient with chest pain may be experiencing acute myocardial infarction and rapid assessment and intervention are needed. The symptoms of the other patients also show target organ damage but are not indicative of acute processes.

The nurse on the intermediate care unit received change-of-shift report on four patients with hypertension. Which patient should the nurse assess first? a. 43-year-old with a (blood pressure (BP) of 160/92 who is complaining of chest pain b. 52-year-old with a BP of 212/90 who has intermittent claudication c. 50-year-old with a BP of 190/104 who has a creatinine of 1.7 mg/dL d. 48-year-old with a BP of 172/98 whose urine shows microalbuminuria

ANS: A The patient with chest pain may be experiencing acute myocardial infarction, and rapid assessment and intervention are needed. The symptoms of the other patients also show target organ damage but are not indicative of acute processes

During a visit to a 78-yr-old patient with chronic heart failure, the home care nurse finds that the patient has ankle edema, a 2-kg weight gain over the past 2 days, and complains of "feeling too tired to get out of bed." Based on these data, a correct nursing diagnosis for the patient is a. activity intolerance related to fatigue. b. impaired skin integrity related to edema. c. disturbed body image related to weight gain. d. impaired gas exchange related to dyspnea on exertion.

ANS: A The patient's statement supports the diagnosis of activity intolerance. There are no data to support the other diagnoses, although the nurse will need to assess for additional patient problems.

During a visit to a 78-year-old with chronic heart failure, the home care nurse finds that the patient has ankle edema, a 2-kg weight gain over the past 2 days, and complains of "feeling too tired to get out of bed." Based on these data, the best nursing diagnosis for the patient is a. activity intolerance related to fatigue. b. disturbed body image related to weight gain. c. impaired skin integrity related to ankle edema. d. impaired gas exchange related to dyspnea on exertion.

ANS: A The patient's statement supports the diagnosis of activity intolerance. There are no data to support the other diagnoses, although the nurse will need to assess for other patient problems.

1. Based on the Joint Commission Core Measures for patients with heart failure, which topics should the nurse include in the discharge teaching plan for a patient who has been hospitalized with chronic heart failure (select all that apply)? a. How to take and record daily weight b. Importance of limiting aerobic exercise c. Date and time of follow-up appointment d. Symptoms indicating worsening heart failure e. Actions and side effects of prescribed medications

ANS: A, C, D, E The Joint Commission Core Measures state that patients should be taught about prescribed medications, follow-up appointments, weight monitoring, and actions to take for worsening symptoms. Patients with heart failure are encouraged to begin or continue aerobic exercises such as walking, while self-monitoring to avoid excessive fatigue. DIF: Cognitive Level: Apply (application) REF: 779 TOP: Nursing Process: Planning MSC: NCLEX: Physiological Integrity

Based on the Joint Commission Core Measures for patients with heart failure, which topics should the nurse include in the discharge teaching plan for a patient who has been hospitalized with chronic heart failure (select all that apply)? a. How to take and record daily weight b. Importance of limiting aerobic exercise c. Date and time of follow-up appointment d. Symptoms indicating worsening heart failure e. Actions and side effects of prescribed medications

ANS: A, C, D, E The Joint Commission Core Measures state that patients should be taught about prescribed medications, follow-up appointments, weight monitoring, and actions to take for worsening symptoms. Patients with heart failure are encouraged to begin or continue aerobic exercises such as walking, while self-monitoring to avoid excessive fatigue.

Which patient requires the most rapid assessment and care by the emergency department nurse? a. The patient with hemochromatosis who reports abdominal pain b. The patient with neutropenia who has a temperature of 101.8° F c. The patient with sickle cell anemia who has had nausea and diarrhea for 24 hours d. The patient with thrombocytopenia who has oozing after having a tooth extracted

ANS: B A neutropenic patient with a fever is assumed to have an infection and is at risk for rapidly developing sepsis. Rapid assessment, cultures, and initiation of antibiotic therapy are needed. The other patients also require rapid assessment and care but not as urgently as the neutropenic patient

The nurse has just finished teaching a hypertensive patient about the newly prescribed drug, ramipril (Altace). Which patient statement indicates that more teaching is needed? a. "The medication may not work well if I take aspirin." b. "I can expect some swelling around my lips and face." c. "The doctor may order a blood potassium level occasionally." d. "I will call the doctor if I notice that I have a frequent cough."

ANS: B Angioedema occurring with angiotensin-converting enzyme (ACE) inhibitor therapy is an indication that the ACE inhibitor should be discontinued. The patient should be taught that if any swelling of the face or oral mucosa occurs, the health care provider should be immediately notified because this could be life threatening. The other patient statements indicate that the patient has an accurate understanding of ACE inhibitor therapy.

Which laboratory result will the nurse expect to show a decreased value if a patient develops heparin-induced thrombocytopenia (HIT)? a. Prothrombin time b. Erythrocyte count c. Fibrinogen degradation products d. Activated partial thromboplastin time

ANS: D Platelet aggregation in HIT causes neutralization of heparin, so that the activated partial thromboplastin time will be shorter and more heparin will be needed to maintain therapeutic levels. The other data will not be affected by HIT

After receiving change-of-shift report for several patients with neutropenia, which patient should the nurse assess first? a. 56-year-old with frequent explosive diarrhea b. 33-year-old with a fever of 100.8° F (38.2° C) c. 66-year-old who has white pharyngeal lesions d. 23-year old who is complaining of severe fatigue

ANS: B Any fever in a neutropenic patient indicates infection and can quickly lead to sepsis and septic shock. Rapid assessment and (if prescribed) initiation of antibiotic therapy within 1 hour are needed. The other patients also need to be assessed but do not exhibit symptoms of potentially life-threatening problems

A patient with hypertension who has just started taking atenolol (Tenormin) returns to the health clinic after 2 weeks for a follow-up visit. The blood pressure (BP) is unchanged from the previous visit. Which action should the nurse take *first*? a. Tell the patient why a change in drug dosage is needed. b. Ask the patient if the medication is being taken as prescribed. c. Inform the patient that multiple drugs are often needed to treat hypertension. d. Question the patient regarding any lifestyle changes made to help control BP.

ANS: B Because nonadherence with antihypertensive therapy is common, the nurse's initial action should be to determine whether the patient is taking the atenolol as prescribed. The other actions also may be implemented, but these would be done after assessing patient adherence with the prescribed therapy.

A patient with hypertension who has just started taking atenolol (Tenormin) returns to the health clinic after 2 weeks for a follow-up visit. The blood pressure (BP) is unchanged from the previous visit. Which action should the nurse take first? a. Inform the patient about the reasons for a possible change in drug dosage. b. Question the patient about whether the medication is actually being taken. c. Inform the patient that multiple drugs are often needed to treat hypertension. d. Question the patient regarding any lifestyle changes made to help control BP.

ANS: B Because noncompliance with antihypertensive therapy is common, the nurse's initial action should be to determine whether the patient is taking the atenolol as prescribed. The other actions also may be implemented, but these would be done after assessing patient compliance with the prescribed therapy

Which collaborative problem will the nurse include in a care plan for a patient admitted to the hospital with idiopathic aplastic anemia? a. Potential complication: seizures b. Potential complication: infection c. Potential complication: neurogenic shock d. Potential complication: pulmonary edema

ANS: B Because the patient with aplastic anemia has pancytopenia, the patient is at risk for infection and bleeding. There is no increased risk for seizures, neurogenic shock, or pulmonary edema

A routine complete blood count indicates that an active 80-year-old man may have myelodysplastic syndrome. The nurse will plan to teach the patient about a. blood transfusion b. bone marrow biopsy. c. filgrastim (Neupogen) administration. d. erythropoietin (Epogen) administration.

ANS: B Bone marrow biopsy is needed to make the diagnosis and determine the specific type of myelodysplastic syndrome. The other treatments may be necessary if there is progression of the myelodysplastic syndrome, but the initial action for this asymptomatic patient will be a bone marrow biopsy.

Which information obtained by the nurse caring for a patient with thrombocytopenia should be immediately communicated to the health care provider? a. The platelet count is 52,000/µL. b. The patient is difficult to arouse. c. There are purpura on the oral mucosa. d. There are large bruises on the patient's back.

ANS: B Difficulty in arousing the patient may indicate a cerebral hemorrhage, which is life threatening and requires immediate action. The other information should be documented and reported but would not be unusual in a patient with thrombocytopenia

Which assessment finding for a patient who is receiving IV furosemide (Lasix) to treat stage 2 hypertension is most important to report to the health care provider? a. Blood glucose level of 175 mg/dL b. Blood potassium level of 3.0 mEq/L c. Most recent blood pressure (BP) reading of 168/94 mm Hg d. Orthostatic systolic BP decrease of 12 mm Hg

ANS: B Hypokalemia is a frequent adverse effect of the loop diuretics and can cause life-threatening dysrhythmias. The health care provider should be notified of the potassium level immediately and administration of potassium supplements initiated. The elevated blood glucose and BP also indicate a need for collaborative interventions but will not require action as urgently as the hypokalemia. An orthostatic drop of 12 mm Hg is common and will require intervention only if the patient is symptomatic

Which assessment finding for a patient who is receiving furosemide (Lasix) to treat stage 2 hypertension is most important to report to the health care provider? a. Blood glucose level of 180 mg/dL b. Blood potassium level of 3.0 mEq/L c. Early morning BP reading of 164/96 mm Hg d. Orthostatic systolic BP decrease of 12 mm Hg

ANS: B Hypokalemia is a frequent adverse effect of the loop diuretics and can cause life-threatening dysrhythmias. The health care provider should be notified of the potassium level immediately and administration of potassium supplements initiated. The elevated blood glucose and BP also indicate a need for collaborative interventions but will not require action as urgently as the hypokalemia. An orthostatic drop of 12 mm Hg is common and will require intervention only if the patient is symptomatic.

Which assessment finding for a patient who is receiving IV furosemide (Lasix) to treat stage 2 hypertension is *most* important to report to the health care provider? a. Blood glucose level of 175 mg/dL b. Serum potassium level of 3.0 mEq/L c. Orthostatic systolic BP decrease of 12 mm Hg d. Most recent blood pressure (BP) reading of 168/94 mm Hg

ANS: B Hypokalemia is a frequent adverse effect of the loop diuretics and can cause life-threatening dysrhythmias. The health care provider should be notified of the potassium level immediately and administration of potassium supplements initiated. The elevated blood glucose and BP also indicate a need for collaborative interventions but will not require action as urgently as the hypokalemia. An orthostatic drop of 12 mm Hg will require intervention only if the patient is symptomatic.

Which intervention will be included in the nursing care plan for a patient with immune thrombocytopenic purpura (ITP)? a. Assign the patient to a private room. b. Avoid intramuscular (IM) injections. c. Use rinses rather than a soft toothbrush for oral care. d. Restrict activity to passive and active range of motion.

ANS: B IM or subcutaneous injections should be avoided because of the risk for bleeding. A soft toothbrush can be used for oral care. There is no need to restrict activity or place the patient in a private room

An older patient has been diagnosed with possible white coat hypertension. Which planned action by the nurse *best* addresses the suspected cause of the hypertension? a. Instruct the patient about the need to decrease stress levels. b. Teach the patient how to self-monitor and record BPs at home. c. Schedule the patient for regular blood pressure (BP) checks in the clinic. d. Inform the patient and caregiver that major dietary changes will be needed.

ANS: B In the phenomenon of "white coat" hypertension, patients have elevated BP readings in a clinical setting and normal readings when BP is measured elsewhere. Having the patient self-monitor BPs at home will provide a reliable indication about whether the patient has hypertension. Regular BP checks in the clinic are likely to be high in a patient with white coat hypertension. There is no evidence that this patient has elevated stress levels or a poor diet, and those factors do not cause white coat hypertension.

A 53-year-old patient with Stage D heart failure and type 2 diabetes asks the nurse whether heart transplant is a possible therapy. Which response by the nurse is most appropriate? a. "Because you have diabetes, you would not be a candidate for a heart transplant." b. "The choice of a patient for a heart transplant depends on many different factors." c. "Your heart failure has not reached the stage in which heart transplants are needed." d. "People who have heart transplants are at risk for multiple complications after surgery."

ANS: B Indications for a heart transplant include end-stage heart failure (Stage D), but other factors such as coping skills, family support, and patient motivation to follow the rigorous posttransplant regimen are also considered. Diabetic patients who have well-controlled blood glucose levels may be candidates for heart transplant. Although heart transplants can be associated with many complications, this response does not address the patient's question.

The nurse notes scleral jaundice in a patient being admitted with hemolytic anemia. The nurse will plan to check the laboratory results for the a. Schilling test. b. bilirubin level. c. stool occult blood test. d. gastric analysis testing.

ANS: B Jaundice is caused by the elevation of bilirubin level associated with red blood cell (RBC) hemolysis. The other tests would not be helpful in monitoring or treating a hemolytic anemia

Which problem reported by a patient with hemophilia is most important for the nurse to communicate to the physician? a. Leg bruises b. Tarry stools c. Skin abrasions d. Bleeding gums

ANS: B Melena is a sign of gastrointestinal bleeding and requires collaborative actions such as checking hemoglobin and hematocrit and administration of coagulation factors. The other problems indicate a need for patient teaching about how to avoid injury, but are not indicators of possible serious blood loss

A patient who is receiving methotrexate for severe rheumatoid arthritis develops a megaloblastic anemia. The nurse will anticipate teaching the patient about increasing oral intake of a. iron. b. folic acid. c. cobalamin (vitamin B12). d. ascorbic acid (vitamin C).

ANS: B Methotrexate use can lead to folic acid deficiency. Supplementation with oral folic acid supplements is the usual treatment. The other nutrients would not correct folic acid deficiency, although they would be used to treat other types of anemia

An appropriate nursing intervention for a hospitalized patient with severe hemolytic anemia is to a. provide a diet high in vitamin K. b. alternate periods of rest and activity. c. teach the patient how to avoid injury. d. place the patient on protective isolation.

ANS: B Nursing care for patients with anemia should alternate periods of rest and activity to encourage activity without causing undue fatigue. There is no indication that the patient has a bleeding disorder, so a diet high in vitamin K or teaching about how to avoid injury is not needed. Protective isolation might be used for a patient with aplastic anemia, but it is not indicated for hemolytic anemia

It is important for the nurse providing care for a patient with sickle cell crisis to a. limit the patient's intake of oral and IV fluids. b. evaluate the effectiveness of opioid analgesics. c. encourage the patient to ambulate as much as tolerated. d. teach the patient about high-protein, high-calorie foods.

ANS: B Pain is the most common clinical manifestation of a crisis and usually requires large doses of continuous opioids for control. Fluid intake should be increased to reduce blood viscosity and improve perfusion. Rest is usually ordered to decrease metabolic requirements. Patients are instructed about the need for dietary folic acid, but high-protein, high-calorie diets are not emphasized

A patient with a history of hypertension treated with a diuretic and an angiotensin-converting enzyme (ACE) inhibitor arrives in the emergency department complaining of a severe headache and nausea and has a blood pressure (BP) of 238/118 mm Hg. Which question should the nurse ask first? a. "Did you take any acetaminophen (Tylenol) today?" b. "Have you been consistently taking your medications?" c. "Have there been any recent stressful events in your life?" d. "Have you recently taken any antihistamine medications?"

ANS: B Sudden withdrawal of antihypertensive medications can cause rebound hypertension and hypertensive crisis. Although many over-the-counter medications can cause hypertension, antihistamines and acetaminophen do not increase BP. Stressful events will increase BP but not usually to the level seen in this patient

A patient with a history of hypertension treated with a diuretic and an angiotensin-converting enzyme (ACE) inhibitor arrives in the emergency department complaining of a severe headache and nausea and has a blood pressure (BP) of 238/118 mm Hg. Which question should the nurse ask to follow up on these findings? a. "Have you recently taken any antihistamines?" b. "Have you consistently taken your medications?" c. "Did you take any acetaminophen (Tylenol) today?" d. "Have there been recent stressful events in your life?"

ANS: B Sudden withdrawal of antihypertensive medications can cause rebound hypertension and hypertensive crisis. Although many over-the-counter medications can cause hypertension, antihistamines and acetaminophen do not increase BP. Stressful events will increase BP but not usually to the level seen in this patient.

A patient with a history of a transfusion-related acute lung injury (TRALI) is to receive a transfusion of packed red blood cells (PRBCs). Which action by the nurse will decrease the risk for TRALI for this patient? a. Infuse the PRBCs slowly over 4 hours. b. Transfuse only leukocyte-reduced PRBCs. c. Administer the scheduled diuretic before the transfusion. d. Give the PRN dose of antihistamine before the transfusion.

ANS: B TRALI is caused by a reaction between the donor and the patient leukocytes that causes pulmonary inflammation and capillary leaking. The other actions may help prevent respiratory problems caused by circulatory overload or by allergic reactions, but they will not prevent TRALI

Which blood pressure (BP) finding by the nurse indicates that no changes in therapy are needed for a 48-yr-old patient with newly diagnosed hypertension? a. 98/56 mm Hg c. 128/92 mm Hg b. 128/76 mm Hg d. 142/78 mm Hg

ANS: B The 8th Joint National Committee's recommended goal for antihypertensive therapy for a 30- to 59-yr-old patient with hypertension is a BP below 140/90 mm Hg. The BP of 98/56 mm Hg may indicate overtreatment of the hypertension and an increased risk for adverse drug effects. The other two blood pressures indicate a need for modifications in the patient's treatment.

A 28-year-old man with von Willebrand disease is admitted to the hospital for minor knee surgery. The nurse will review the coagulation survey to check the a. platelet count. b. bleeding time. c. thrombin time. d. prothrombin time.

ANS: B The bleeding time is affected by von Willebrand disease. Platelet count, prothrombin time, and thrombin time are normal in von Willebrand disease

Which action will the admitting nurse include in the care plan for a 30-year old woman who is neutropenic? a. Avoid any injections. b. Check temperature every 4 hours. c. Omit fruits or vegetables from the diet. d. Place a "No Visitors" sign on the door.

ANS: B The earliest sign of infection in a neutropenic patient is an elevation in temperature. Although unpeeled fresh fruits and vegetables should be avoided, fruits and vegetables that are peeled or cooked are acceptable. Injections may be required for administration of medications such as filgrastim (Neupogen). The number of visitors may be limited and visitors with communicable diseases should be avoided, but a "no visitors" policy is not needed

Which blood pressure (BP) finding by the nurse indicates that no changes in therapy are needed for a patient with stage 1 hypertension who has a history of diabetes mellitus? a. 102/60 mm Hg b. 128/76 mm Hg c. 139/90 mm Hg d. 136/82 mm Hg

ANS: B The goal for antihypertensive therapy for a patient with hypertension and diabetes mellitus is a BP <130/80 mm Hg. The BP of 102/60 may indicate overtreatment of the hypertension and an increased risk for adverse drug effects. The other two blood pressures indicate a need for modifications in the patient's treatment

Which BP finding by the nurse indicates that no changes in therapy are needed for a patient with stage 1 hypertension who has a history of heart failure? a. 108/64 mm Hg b. 128/76 mm Hg c. 140/90 mm Hg d. 136/ 82 mm Hg

ANS: B The goal for antihypertensive therapy for a patient with hypertension and heart failure is a BP of <130/80 mm Hg. The BP of 108/64 may indicate overtreatment of the hypertension and an increased risk for adverse drug effects. The other two blood pressures indicate a need for modifications in the patient's treatment.

A 30-year-old man with acute myelogenous leukemia develops an absolute neutrophil count of 850/µL while receiving outpatient chemotherapy. Which action by the outpatient clinic nurse is most appropriate? a. Discuss the need for hospital admission to treat the neutropenia. b. Teach the patient to administer filgrastim (Neupogen) injections. c. Plan to discontinue the chemotherapy until the neutropenia resolves. d. Order a high-efficiency particulate air (HEPA) filter for the patient's home.

ANS: B The patient may be taught to self-administer filgrastim injections. Although chemotherapy may be stopped with severe neutropenia (neutrophil count less than 500/µL), administration of filgrastim usually allows the chemotherapy to continue. Patients with neutropenia are at higher risk for infection when exposed to other patients in the hospital. HEPA filters are expensive and are used in the hospital, where the number of pathogens is much higher than in the patient's home environment

Which action will the nurse in the hypertension clinic take in order to obtain an accurate baseline blood pressure (BP) for a new patient? a. Deflate the BP cuff at a rate of 5 to 10 mm Hg per second. b. Have the patient sit in a chair with the feet flat on the floor. c. Assist the patient to the supine position for BP measurements. d. Obtain two BP readings in the dominant arm and average the results.

ANS: B The patient should be seated with the feet flat on the floor. The BP is obtained in both arms, and the results of the two arms are not averaged. The patient does not need to be in the supine position. The cuff should be deflated at 2 to 3 mm Hg per second

Which action will the nurse in the hypertension clinic take to obtain an accurate baseline blood pressure (BP) for a new patient? a. Deflate the BP cuff at a rate of 5 to 10 mm Hg per second. b. Have the patient sit in a chair with the feet flat on the floor. c. Assist the patient to the supine position for BP measurements. d. Obtain two BP readings in the dominant arm and average the results.

ANS: B The patient should be seated with the feet flat on the floor. The BP is obtained in both arms, and the results of the two arms are not averaged. The patient does not need to be in the supine position. The cuff should be deflated at 2 to 3 mm Hg per second.

After receiving change-of-shift report on four patients admitted to a heart failure unit, which patient should the nurse assess first? a. A patient who reported dizziness after receiving the first dose of captopril b. A patient who is cool and clammy, with new-onset confusion and restlessness c. A patient who has crackles bilaterally in the lung bases and is receiving oxygen. d. A patient who is receiving IV nesiritide (Natrecor) and has a blood pressure of 100/62

ANS: B The patient who has "wet-cold" clinical manifestations of heart failure is perfusing inadequately and needs rapid assessment and changes in management. The other patients also should be assessed as quickly as possible but do not have indications of severe decreases in tissue perfusion.

The nurse in the emergency department received change-of-shift report on four patients with hypertension. Which patient should the nurse assess first? a. 52-year-old with a BP of 212/90 who has intermittent claudication b. 43-year-old with a BP of 190/102 who is complaining of chest pain c. 50-year-old with a BP of 210/110 who has a creatinine of 1.5 mg/dL d. 48-year-old with a BP of 200/98 whose urine shows microalbuminuria

ANS: B The patient with chest pain may be experiencing acute myocardial infarction and rapid assessment and intervention is needed. The symptoms of the other patients also show target organ damage, but are not indicative of acute processes.

Several patients call the outpatient clinic and ask to make an appointment as soon as possible. Which patient should the nurse schedule to be seen first? a. 44-year-old with sickle cell anemia who says "my eyes always look sort of yellow" b. 23-year-old with no previous health problems who has a nontender lump in the axilla c. 50-year-old with early-stage chronic lymphocytic leukemia who reports chronic fatigue d. 19-year-old with hemophilia who wants to learn to self-administer factor VII replacement

ANS: B The patient's age and presence of a nontender axillary lump suggest possible lymphoma, which needs rapid diagnosis and treatment. The other patients have questions about treatment or symptoms that are consistent with their diagnosis but do not need to be seen urgently

The nurse obtains this information from a patient with prehypertension. Which finding is most important to address with the patient? a. Low dietary fiber intake b. No regular aerobic exercise c. Weight 5 pounds above ideal weight d. Drinks wine with dinner once a week

ANS: B The recommendations for preventing hypertension include exercising aerobically for 30 minutes most days of the week. A weight that is 5 pounds over the ideal body weight is not a risk factor for hypertension. The Dietary Approaches to Stop Hypertension (DASH) diet is high in fiber, but increasing fiber alone will not prevent hypertension from developing. The patient's alcohol intake will not increase the hypertension risk.

The nurse is assessing a patient who has been admitted to the intensive care unit (ICU) with a hypertensive emergency. Which finding is most important to report to the health care provider? a. Urine output over 8 hours is 200 mL less than the fluid intake. b. The patient is unable to move the left arm and leg when asked to do so. c. Tremors are noted in the fingers when the patient extends the arms. d. The patient complains of a severe headache with pain at level 8/10 (0 to 10 scale).

ANS: B The patient's inability to move the left arm and leg indicates that a hemorrhagic stroke may be occurring and will require immediate action to prevent further neurologic damage. The other clinical manifestations also likely are caused by the hypertension and will require rapid nursing actions, but they do not require action as urgently as the neurologic changes.

The nurse is assessing a patient who has been admitted to the intensive care unit (ICU) with a hypertensive emergency. Which finding is most important to report to the health care provider? a. Urine output over 8 hours is 250 mL less than the fluid intake. b. The patient cannot move the left arm and leg when asked to do so. c. Tremors are noted in the fingers when the patient extends the arms. d. The patient complains of a headache with pain at level 8/10 (0 to 10 scale).

ANS: B The patient's inability to move the left arm and leg indicates that a hemorrhagic stroke may be occurring and will require immediate action to prevent further neurologic damage. The other clinical manifestations are also likely caused by the hypertension and will require rapid nursing actions, but they do not require action as urgently as the neurologic changes

The nurse is assessing a patient who has been admitted to the intensive care unit (ICU) with a hypertensive emergency. Which finding is *most* important to report to the health care provider? a. Urine output over 8 hours is 250 mL less than the fluid intake. b. The patient cannot move the left arm and leg when asked to do so. c. Tremors are noted in the fingers when the patient extends the arms. d. The patient complains of a headache with pain at level 7 of 10 (0 to 10 scale).

ANS: B The patient's inability to move the left arm and leg indicates that a stroke may be occurring and will require immediate action to prevent further neurologic damage. The other clinical manifestations are also likely caused by the hypertension and will require rapid nursing actions, but they do not require action as urgently as the neurologic changes.

After receiving change-of-shift report on a heart failure unit, which patient should the nurse assess first? a. Patient who is taking carvedilol (Coreg) and has a heart rate of 58 b. Patient who is taking digoxin and has a potassium level of 3.1 mEq/L c. Patient who is taking isosorbide dinitrate/hydralazine (BiDil) and has a headache d. Patient who is taking captopril (Capoten) and has a frequent nonproductive cough

ANS: B The patient's low potassium level increases the risk for digoxin toxicity and potentially fatal dysrhythmias. The nurse should assess the patient for other signs of digoxin toxicity and then notify the health care provider about the potassium level. The other patients also have side effects of their medications, but their symptoms do not indicate potentially life-threatening complications.

After receiving change-of-shift report on a heart failure unit, which patient should the nurse assess first? a. Patient who is taking carvedilol (Coreg) and has a heart rate of 58 b. Patient who is taking digoxin and has a potassium level of 3.1 mEq/L c. Patient who is taking captopril and has a frequent nonproductive cough d. Patient who is taking isosorbide dinitrate/hydralazine (BiDil) and has a headache

ANS: B The patient's low potassium level increases the risk for digoxin toxicity and potentially life-threatening dysrhythmias. The nurse should assess the patient for other signs of digoxin toxicity and then notify the health care provider about the potassium level. The other patients also have side effects of their drugs, but their symptoms do not indicate potentially life-threatening complications.

A patient with septicemia develops prolonged bleeding from venipuncture sites and blood in the stools. Which action is most important for the nurse to take? a. Avoid venipunctures. b. Notify the patient's physician. c. Apply sterile dressings to the sites. d. Give prescribed proton-pump inhibitors.

ANS: B The patient's new onset of bleeding and diagnosis of sepsis suggest that disseminated intravascular coagulation (DIC) may have developed, which will require collaborative actions such as diagnostic testing, blood product administration, and heparin administration. The other actions also are appropriate, but the most important action should be to notify the physician so that DIC treatment can be initiated rapidly

The nurse obtains the following information from a patient newly diagnosed with prehypertension. Which finding is most important to address with the patient? a. Low dietary fiber intake b. No regular aerobic exercise c. Weight 5 pounds above ideal weight d. Drinks a beer with dinner on most nights

ANS: B The recommendations for preventing hypertension include exercising aerobically for 30 minutes most days of the week. A weight that is 5 pounds over the ideal body weight is not a risk factor for hypertension. The Dietary Approaches to Stop Hypertension (DASH) diet is high in fiber, but increasing fiber alone will not prevent hypertension from developing. The patient's alcohol intake is within guidelines and will not increase the hypertension risk

The nurse obtains the following information from a patient newly diagnosed with prehypertension. Which finding is *most* important to address with the patient? a. Low dietary fiber intake b. No regular physical exercise c. Drinks a beer with dinner every night d. Weight is 5 pounds above ideal weight

ANS: B The recommendations for preventing hypertension include exercising aerobically for 30 minutes most days of the week. A weight that is 5 pounds over the ideal body weight is not a risk factor for hypertension. The Dietary Approaches to Stop Hypertension (DASH) diet is high in fiber, but increasing fiber alone will not prevent hypertension from developing. The patient's alcohol intake is within guidelines and will not increase the hypertension risk.

A 68-year-old woman with acute myelogenous leukemia (AML) asks the nurse whether the planned chemotherapy will be worth undergoing. Which response by the nurse is appropriate? a. "If you do not want to have chemotherapy, other treatment options include stem cell transplantation." b. "The side effects of chemotherapy are difficult, but AML frequently goes into remission with chemotherapy." c. "The decision about treatment is one that you and the doctor need to make rather than asking what I would do." d. "You don't need to make a decision about treatment right now because leukemias in adults tend to progress quite slowly."

ANS: B This response uses therapeutic communication by addressing the patient's question and giving accurate information. The other responses either give inaccurate information or fail to address the patient's question, which will discourage the patient from asking the nurse for information

The nurse is planning to administer a transfusion of packed red blood cells (PRBCs) to a patient with blood loss from gastrointestinal hemorrhage. Which action can the nurse delegate to unlicensed assistive personnel (UAP)? a. Verify the patient identification (ID) according to hospital policy. b. Obtain the temperature, blood pressure, and pulse before the transfusion. c. Double-check the product numbers on the PRBCs with the patient ID band. d. Monitor the patient for shortness of breath or chest pain during the transfusion.

ANS: B UAP education includes measurement of vital signs. UAP would report the vital signs to the registered nurse (RN). The other actions require more education and a larger scope of practice and should be done by licensed nursing staff members

When teaching the patient with newly diagnosed heart failure about a 2000-mg sodium diet, the nurse explains that foods to be restricted include a. canned and frozen fruits. b. yogurt and milk products. c. fresh or frozen vegetables. d. eggs and other high-protein foods.

ANS: B Yogurt and milk products (e.g., cheese) naturally contain a significant amount of sodium, and the intake of these should be limited for patients on a diet that limits sodium to 2000 mg daily. The other foods listed have minimal levels of sodium and can be eaten without restriction.

A 56-yr-old patient who has no previous history of hypertension or other health problems suddenly develops a blood pressure (BP) of 198/110 mm Hg. After reconfirming the BP, it is appropriate for the nurse to tell the patient that a. a BP recheck should be scheduled in a few weeks. b. dietary sodium and fat content should be decreased. c. diagnosis, treatment, and ongoing monitoring will be needed. d. there is an immediate danger of a stroke, requiring hospitalization.

ANS: C A sudden increase in BP in a patient older than age 50 years with no previous hypertension history or risk factors indicates that the hypertension may be secondary to some other problem. The BP will need treatment and ongoing monitoring. If the patient has no other risk factors, a stroke in the immediate future is unlikely. There is no indication that dietary salt or fat intake have contributed to this sudden increase in BP, and reducing intake of salt and fat alone will not be adequate to reduce this BP to an acceptable level.

Which action for a patient with neutropenia is appropriate for the registered nurse (RN) to delegate to a licensed practical/vocational nurse (LPN/LVN)? a. Assessing the patient for signs and symptoms of infection b. Teaching the patient the purpose of neutropenic precautions c. Administering subcutaneous filgrastim (Neupogen) injection d. Developing a discharge teaching plan for the patient and family

ANS: C Administration of subcutaneous medications is included in LPN/LVN education and scope of practice. Patient education, assessment, and developing the plan of care require RN level education and scope of practice

A patient who is receiving dobutamine for the treatment of acute decompensated heart failure (ADHF) has the following nursing interventions included in the plan of care. Which action will be most appropriate for the registered nurse (RN) to delegate to an experienced licensed practical/vocational nurse (LPN/LVN)? a. Teach the patient the reasons for remaining on bed rest. b. Change the peripheral IV site according to agency policy. c. Monitor the patient's blood pressure and heart rate every hour. d. Titrate the rate to keep the systolic blood pressure >90 mm Hg.

ANS: C An experienced LPN/LVN would be able to monitor BP and heart rate and would know to report significant changes to the RN. Teaching patients, making adjustments to the drip rate for vasoactive drugs, and inserting a new peripheral IV catheter require RN level education and scope of practice.

A patient who is receiving dobutamine (Dobutrex) for the treatment of acute decompensated heart failure (ADHF) has the following nursing interventions included in the plan of care. Which action will be most appropriate for the registered nurse (RN) to delegate to an experienced licensed practical/vocational nurse (LPN/LVN)? a. Assess the IV insertion site for signs of extravasation. b. Teach the patient the reasons for remaining on bed rest. c. Monitor the patient's blood pressure and heart rate every hour. d. Titrate the rate to keep the systolic blood pressure >90 mm Hg.

ANS: C An experienced LPN/LVN would be able to monitor BP and heart rate and would know to report significant changes to the RN. Teaching patients, making adjustments to the drip rate for vasoactive medications, and monitoring for serious complications such as extravasation require RN level education and scope of practice.

When a patient with splenomegaly is scheduled for splenectomy, which action will the nurse include in the preoperative plan of care? a. Discourage deep breathing to reduce risk for splenic rupture. b. Teach the patient to use ibuprofen (Advil) for left upper quadrant pain. c. Schedule immunization with the pneumococcal vaccine (Pneumovax). d. Avoid the use of acetaminophen (Tylenol) for 2 weeks prior to surgery.

ANS: C Asplenic patients are at high risk for infection with Pneumococcus and immunization reduces this risk. There is no need to avoid acetaminophen use before surgery, but nonsteroidal antiinflammatory drugs (NSAIDs) may increase bleeding risk and should be avoided. The enlarged spleen may decrease respiratory depth and the patient should be encouraged to take deep breaths

Which diagnostic test will be most useful to the nurse in determining whether a patient admitted with acute shortness of breath has heart failure? a. Serum troponin b. Arterial blood gases c. B-type natriuretic peptide d. 12-lead electrocardiogram

ANS: C B-type natriuretic peptide (BNP) is secreted when ventricular pressures increase, as they do with heart failure. Elevated BNP indicates a probable or very probable diagnosis of heart failure. A 12-lead electrocardiogram, arterial blood gases, and troponin may also be used in determining the causes or effects of heart failure but are not as clearly diagnostic of heart failure as BNP.

A 52-year-old patient has a new diagnosis of pernicious anemia. The nurse determines that the patient understands the teaching about the disorder when the patient states, "I a. need to start eating more red meat and liver." b. will stop having a glass of wine with dinner." c. could choose nasal spray rather than injections of vitamin B12." d. will need to take a proton pump inhibitor like omeprazole (Prilosec)."

ANS: C Because pernicious anemia prevents the absorption of vitamin B12, this patient requires injections or intranasal administration of cobalamin. Alcohol use does not cause cobalamin deficiency. Proton pump inhibitors decrease the absorption of vitamin B12. Eating more foods rich in vitamin B12 is not helpful because the lack of intrinsic factor prevents absorption of the vitamin

A patient with heart failure has a new order for captopril (Capoten) 12.5 mg PO. After administering the first dose and teaching the patient about the drug, which statement by the patient indicates that teaching has been effective? a. "I will be sure to take the medication with food." b. "I will need to eat more potassium-rich foods in my diet." c. "I will call for help when I need to get up to use the bathroom." d. "I will expect to feel more short of breath for the next few days."

ANS: C Captopril can cause hypotension, especially after the initial dose, so it is important that the patient not get up out of bed without assistance until the nurse has had a chance to evaluate the effect of the first dose. The angiotensin-converting enzyme (ACE) inhibitors are potassium sparing, and the nurse should not teach the patient to purposely increase sources of dietary potassium. Increased shortness of breath is expected with the initiation of -adrenergic blocker therapy for heart failure, not for ACE inhibitor therapy. ACE inhibitors are best absorbed when taken an hour before eating.

A patient with heart failure has a new order for captopril 12.5 mg PO. After giving the first dose and teaching the patient about the drug, which statement by the patient indicates that teaching has been effective? a. "I will be sure to take the medication with food." b. "I will need to eat more potassium-rich foods in my diet." c. "I will call for help when I need to get up to use the bathroom." d. "I will expect to feel more short of breath for the next few days."

ANS: C Captopril can cause hypotension, especially after the initial dose, so it is important that the patient not get up out of bed without assistance until the nurse has had a chance to evaluate the effect of the first dose. The angiotensin-converting enzyme (ACE) inhibitors are potassium sparing, and the nurse should not teach the patient to purposely increase sources of dietary potassium. Increased shortness of breath is expected with the initiation of -adrenergic blocker therapy for heart failure, not for ACE inhibitor therapy. ACE inhibitors are best absorbed when taken an hour before eating.

Which instruction will the nurse plan to include in discharge teaching for the patient admitted with a sickle cell crisis? a. Take a daily multivitamin with iron. b. Limit fluids to 2 to 3 quarts per day. c. Avoid exposure to crowds when possible. d. Drink only two caffeinated beverages daily.

ANS: C Exposure to crowds increases the patient's risk for infection, the most common cause of sickle cell crisis. There is no restriction on caffeine use. Iron supplementation is generally not recommended. A high-fluid intake is recommended

After the nurse teaches the patient with stage 1 hypertension about diet modifications that should be implemented, which diet choice indicates that the teaching has been *most* effective? a. The patient avoids eating nuts or nut butters. b. The patient restricts intake of chicken and fish. c. The patient drinks low-fat milk with each meal. d. The patient has two cups of coffee in the morning.

ANS: C For the prevention of hypertension, the Dietary Approaches to Stop Hypertension (DASH) recommendations include increasing the intake of calcium-rich foods. Caffeine restriction and decreased protein intake are not included in the recommendations. Nuts are high in beneficial nutrients and 4 to 5 servings weekly are recommended in the DASH diet.

Which action will be included in the plan of care when the nurse is caring for a patient who is receiving sodium nitroprusside (Nipride) to treat a hypertensive emergency? a. Organize nursing activities so that the patient has undisturbed sleep for 6 to 8 hours at night. b. Assist the patient up in the chair for meals to avoid complications associated with immobility. c. Use an automated noninvasive blood pressure machine to obtain frequent BP measurements. d. Place the patient on NPO status to prevent aspiration caused by nausea and the associated vomiting.

ANS: C Frequent monitoring of BP is needed when the patient is receiving rapid-acting IV antihypertensive medications. This can be most easily accomplished with an automated BP machine or arterial line. The patient will require frequent assessments, so allowing 6 to 8 hours of undisturbed sleep is not appropriate. When patients are receiving IV vasodilators, bed rest is maintained to prevent decreased cerebral perfusion and fainting. There is no indication that this patient is nauseated or at risk for aspiration, so an NPO status is unnecessary.

A patient has been diagnosed with possible white coat hypertension. Which action will the nurse plan to take next? a. Schedule the patient for frequent BP checks in the clinic. b. Instruct the patient about the need to decrease stress levels. c. Tell the patient how to self-monitor and record BPs at home. d. Teach the patient about ambulatory blood pressure monitoring.

ANS: C Having the patient self-monitor BPs at home will provide a reliable indication about whether the patient has hypertension. Frequent BP checks in the clinic are likely to be high in a patient with white coat hypertension. Ambulatory blood pressure monitoring may be used if the data from self-monitoring is unclear. Although elevated stress levels may contribute to hypertension, instructing the patient about this is unlikely to reduce BP.

An older patient has been diagnosed with possible white coat hypertension. Which action will the nurse plan to take next? a. Schedule the patient for regular blood pressure (BP) checks in the clinic. b. Instruct the patient about the need to decrease stress levels. c. Tell the patient how to self-monitor and record BPs at home. d. Inform the patient that ambulatory blood pressure monitoring will be needed.

ANS: C Having the patient self-monitor BPs at home will provide a reliable indication about whether the patient has hypertension. Regular BP checks in the clinic are likely to be high in a patient with white coat hypertension. Ambulatory blood pressure monitoring may be used if the data from self-monitoring are unclear. Although elevated stress levels may contribute to hypertension, instructing the patient about this is unlikely to reduce BP

Which information is *most* important for the nurse to include when teaching a patient with newly diagnosed hypertension? a. Most people are able to control BP through dietary changes. b. Annual BP checks are needed to monitor treatment effectiveness. c. Hypertension is usually asymptomatic until target organ damage occurs. d. Increasing physical activity alone controls blood pressure (BP) for most people.

ANS: C Hypertension is usually asymptomatic until target organ damage has occurred. Lifestyle changes (e.g., physical activity, dietary changes) are used to help manage BP, but drugs are needed for most patients. Home BP monitoring should be taught to the patient and findings checked by the health care provider frequently when starting treatment for hypertension and then every 3 months when stable.

Which information should the nurse include when teaching a patient with newly diagnosed hypertension? a. Dietary sodium restriction will control BP for most patients. b. Most patients are able to control BP through lifestyle changes. c. Hypertension is usually asymptomatic until significant organ damage occurs. d. Annual BP checks are needed to monitor treatment effectiveness.

ANS: C Hypertension is usually asymptomatic until target organ damage has occurred. Lifestyle changes and sodium restriction are used to help manage blood pressure, but drugs are needed for most patients. BP should be checked by the health care provider every 3 to 6 months.

A patient has recently started on digoxin (Lanoxin) in addition to furosemide (Lasix) and captopril for the management of heart failure. Which assessment finding by the home health nurse is a priority to communicate to the health care provider? a. Presence of 1+ to 2+ edema in the feet and ankles b. Palpable liver edge 2 cm below the ribs on the right side c. Serum potassium level 3.0 mEq/L after 1 week of therapy d. Weight increase from 120 pounds to 122 pounds over 3 days

ANS: C Hypokalemia can predispose the patient to life-threatening dysrhythmias (e.g., premature ventricular contractions) and potentiate the actions of digoxin. Hypokalemia also increases the risk for digoxin toxicity, which can also cause life-threatening dysrhythmias. The other data indicate that the patient's heart failure requires more effective therapies, but they do not require nursing action as rapidly as the low serum potassium level.

A 53-yr-old patient with stage D heart failure and type 2 diabetes asks the nurse whether heart transplant is a possible therapy. Which response by the nurse is most accurate? a. "Your heart failure has not reached the end stage yet." b. "You could not manage the multiple complications of that surgery." c. "The suitability of a heart transplant for you depends on many factors." d. "Because you have diabetes, you would not be a heart transplant candidate."

ANS: C Indications for a heart transplant include end-stage heart failure (stage D), but other factors such as coping skills, family support, and patient motivation to follow the rigorous posttransplant regimen are also considered. Patients with diabetes who have well-controlled blood glucose levels may be candidates for heart transplant. Although heart transplants can be associated with many complications, there are no data to suggest that the patient could not manage the care.

The RN is caring for a patient with a hypertensive crisis who is receiving sodium nitroprusside (Nipride). Which of the following nursing actions can the nurse delegate to an experienced LPN/LVN? a. Titrate nitroprusside to maintain BP at 160/100 mm Hg. b. Evaluate effectiveness of nitroprusside therapy on BP. c. Set up the automatic blood pressure machine to take BP every 15 minutes. d. Assess the patient's environment for adverse stimuli that might increase BP.

ANS: C LPN/LVN education and scope of practice include correct use of common equipment such as automatic blood pressure machines. The other actions require more nursing judgment and education and should be done by RNs.

The registered nurse (RN) is caring for a patient with a hypertensive crisis who is receiving sodium nitroprusside (Nipride). Which nursing action can the nurse delegate to an experienced licensed practical/vocational nurse (LPN/LVN)? a. Titrate nitroprusside to decrease mean arterial pressure (MAP) to 115 mm Hg. b. Evaluate effectiveness of nitroprusside therapy on blood pressure (BP). c. Set up the automatic blood pressure machine to take BP every 15 minutes. d. Assess the patient's environment for adverse stimuli that might increase BP.

ANS: C LPN/LVN education and scope of practice include the correct use of common equipment such as automatic blood pressure machines. The other actions require advanced nursing judgment and education, and should be done by RNs

Which action should the nurse take when administering the initial dose of oral labetalol (Normodyne) to a patient with hypertension? a. Encourage the use of hard candy to prevent dry mouth. b. Instruct the patient to ask for help if heart palpitations occur. c. Ask the patient to request assistance when getting out of bed. d. Teach the patient that headaches may occur with this medication.

ANS: C Labetalol decreases sympathetic nervous system activity by blocking both á- and b-adrenergic receptors, leading to vasodilation and a decrease in heart rate, which can cause severe orthostatic hypotension. Heart palpitations, dry mouth, dehydration, and headaches are possible side effects of other antihypertensives

After giving a patient the initial dose of oral labetalol (Normodyne) for treatment of hypertension, which action should the nurse take? a. Encourage oral fluids to prevent dry mouth or dehydration. b. Instruct the patient to ask for help if heart palpitations occur. c. Ask the patient to request assistance when getting out of bed. d. Teach the patient that headaches may occur with this medication.

ANS: C Labetalol decreases sympathetic nervous system activity by blocking both α- and β-adrenergic receptors, leading to vasodilation and a decrease in heart rate, which can cause severe orthostatic hypotension. Heart palpitations, dehydration, and headaches are possible side effects of other antihypertensives.

The nurse plans discharge teaching for a patient with chronic heart failure who has prescriptions for digoxin (Lanoxin) and hydrochlorothiazide. Appropriate instructions for the patient include a. limit dietary sources of potassium. b. take the hydrochlorothiazide before bedtime. c. notify the health care provider if nausea develops. d. take the digoxin if the pulse is below 60 beats/min.

ANS: C Nausea is an indication of digoxin toxicity and should be reported so that the provider can assess the patient for toxicity and adjust the digoxin dose, if necessary. The patient will need to include potassium-containing foods in the diet to avoid hypokalemia. Patients should be taught to check their pulse daily before taking the digoxin and if the pulse is less than 60 beats/min, to call their provider before taking the digoxin. Diuretics should be taken early in the day to avoid sleep disruption.

The nurse plans discharge teaching for a patient with chronic heart failure who has prescriptions for digoxin (Lanoxin) and hydrochlorothiazide (HydroDIURIL). Appropriate instructions for the patient include a. limit dietary sources of potassium. b. take the hydrochlorothiazide before bedtime. c. notify the health care provider if nausea develops. d. skip the digoxin if the pulse is below 60 beats/minute.

ANS: C Nausea is an indication of digoxin toxicity and should be reported so that the provider can assess the patient for toxicity and adjust the digoxin dose, if necessary. The patient will need to include potassium-containing foods in the diet to avoid hypokalemia. Patients should be taught to check their pulse daily before taking the digoxin and if the pulse is less than 60, to call their provider before taking the digoxin. Diuretics should be taken early in the day to avoid sleep disruption.

Propranolol (Inderal) is prescribed for a patient diagnosed with hypertension. The nurse should consult with the health care provider before giving this drug when the patient reveals a history of a. daily alcohol use. c. reactive airway disease. b. peptic ulcer disease. d. myocardial infarction (MI).

ANS: C Nonselective b-blockers block b1- and b2-adrenergic receptors and can cause bronchospasm, especially in patients with a history of asthma. b-Blockers will have no effect on the patient's peptic ulcer disease or alcohol use. b-Blocker therapy is recommended after MI.

A 54-year-old woman with acute myelogenous leukemia (AML) is considering treatment with a hematopoietic stem cell transplant (HSCT). The best approach for the nurse to assist the patient with a treatment decision is to a. emphasize the positive outcomes of a bone marrow transplant. b. discuss the need for adequate insurance to cover post-HSCT care. c. ask the patient whether there are any questions or concerns about HSCT. d. explain that a cure is not possible with any other treatment except HSCT.

ANS: C Offering the patient an opportunity to ask questions or discuss concerns about HSCT will encourage the patient to voice concerns about this treatment and also will allow the nurse to assess whether the patient needs more information about the procedure. Treatment of AML using chemotherapy is another option for the patient. It is not appropriate for the nurse to ask the patient to consider insurance needs in making this decision

A patient who has chronic heart failure tells the nurse, "I was fine when I went to bed, but I woke up in the middle of the night feeling like I was suffocating!" The nurse will document this assessment finding as a. orthopnea. b. pulsus alternans. c. paroxysmal nocturnal dyspnea. d. acute bilateral pleural effusion.

ANS: C Paroxysmal nocturnal dyspnea is caused by the reabsorption of fluid from dependent body areas when the patient is sleeping and is characterized by waking up suddenly with the feeling of suffocation. Pulsus alternans is the alteration of strong and weak peripheral pulses during palpation. Orthopnea indicates that the patient is unable to lie flat because of dyspnea. Pleural effusions develop over a longer time period.

IV sodium nitroprusside is ordered for a patient with acute pulmonary edema. During the first hours of administration, the nurse will need to titrate the nitroprusside rate down if the patient develops a. ventricular ectopy. b. a dry, hacking cough. c. a systolic BP below 90 mm Hg. d. a heart rate below 50 beats/min.

ANS: C Sodium nitroprusside is a potent vasodilator and the major adverse effect is severe hypotension. Coughing and bradycardia are not adverse effects of this medication. Nitroprusside does not cause increased ventricular ectopy.

IV sodium nitroprusside (Nipride) is ordered for a patient with acute pulmonary edema. During the first hours of administration, the nurse will need to titrate the nitroprusside rate if the patient develops a. ventricular ectopy. b. a dry, hacking cough. c. a systolic BP <90 mm Hg. d. a heart rate <50 beats/minute.

ANS: C Sodium nitroprusside is a potent vasodilator, and the major adverse effect is severe hypotension. Coughing and bradycardia are not adverse effects of this medication. Nitroprusside does not cause increased ventricular ectopy.

A patient with a history of hypertension treated with a diuretic and an angiotensin-converting enzyme (ACE) inhibitor arrives in the emergency department complaining of a severe headache and has a BP of 240/118 mm Hg. Which question should the nurse ask first? a. Did you take any acetaminophen (Tylenol) today? b. Do you have any recent stressful events in your life? c. Have you been consistently taking your medications? d. Have you recently taken any antihistamine medications?

ANS: C Sudden withdrawal of antihypertensive medications can cause rebound hypertension and hypertensive crisis. Although many over-the-counter medications can cause hypertension, antihistamines and acetaminophen do not increase BP. Stressful events will increase BP but not usually to the level seen in this patient.

The nurse working on the heart failure unit knows that teaching an older female patient with newly diagnosed heart failure is effective when the patient states that a. she will take furosemide (Lasix) every day at bedtime. b. the nitroglycerin patch is to be used when chest pain develops. c. she will call the clinic if her weight goes up 3 pounds in 1 week. d. an additional pillow can help her sleep if she is short of breath at night.

ANS: C Teaching for a patient with heart failure includes information about the need to weigh daily and notify the health care provider about an increase of 3 lb in 2 days or 3 to 5 lb in a week. Nitroglycerin patches are used primarily to reduce preload (not to prevent chest pain) in patients with heart failure and should be used daily, not on an "as needed" basis. Diuretics should be taken earlier in the day to avoid nocturia and sleep disturbance. The patient should call the clinic if increased orthopnea develops rather than just compensating by further elevating the head of the bed.

The nurse working on the heart failure unit knows that teaching an older female patient with newly diagnosed heart failure is effective when the patient states that a. she will take furosemide (Lasix) every day at bedtime. b. the nitroglycerin patch is applied when any chest pain develops. c. she will call the clinic if her weight goes from 124 to 128 pounds in a week. d. an additional pillow can help her sleep if she is feeling short of breath at night.

ANS: C Teaching for a patient with heart failure includes information about the need to weigh daily and notify the health care provider about an increase of 3 pounds in 2 days or 3 to 5 pounds in a week. Nitroglycerin patches are used primarily to reduce preload (not to prevent chest pain) in patients with heart failure and should be used daily, not on an "as needed" basis. Diuretics should be taken earlier in the day to avoid nocturia and sleep disturbance. The patient should call the clinic if increased orthopnea develops, rather than just compensating by further elevating the head of the bed.

A patient with chronic heart failure who is taking a diuretic and an angiotensin-converting enzyme (ACE) inhibitor and who is on a low-sodium diet tells the home health nurse about a 5-lb weight gain in the past 3 days. The nurse's priority action will be to a. have the patient recall the dietary intake for the past 3 days. b. ask the patient about the use of the prescribed medications. c. assess the patient for clinical manifestations of acute heart failure. d. teach the patient about the importance of restricting dietary sodium.

ANS: C The 5-lb weight gain over 3 days indicates that the patient's chronic heart failure may be worsening. It is important that the patient be assessed immediately for other clinical manifestations of decompensation, such as lung crackles. A dietary recall to detect hidden sodium in the diet, reinforcement of sodium restrictions, and assessment of medication compliance may be appropriate interventions but are not the first nursing actions indicated.

A patient with chronic heart failure who is taking a diuretic and an angiotensin-converting enzyme (ACE) inhibitor and who is on a low-sodium diet tells the home health nurse about a 5-pound weight gain in the last 3 days. The nurse's priority action will be to a. have the patient recall the dietary intake for the last 3 days. b. ask the patient about the use of the prescribed medications. c. assess the patient for clinical manifestations of acute heart failure. d. teach the patient about the importance of restricting dietary sodium.

ANS: C The 5-pound weight gain over 3 days indicates that the patient's chronic heart failure may be worsening. It is important that the patient be assessed immediately for other clinical manifestations of decompensation, such as lung crackles. A dietary recall to detect hidden sodium in the diet, reinforcement of sodium restrictions, and assessment of medication compliance may be appropriate interventions but are not the first nursing actions indicated.

The charge nurse observes a new RN doing discharge teaching for a hypertensive patient who has a new prescription for enalapril (Vasotec). The charge nurse will need to intervene if the new RN tells the patient to a. check the BP with a home BP monitor every day. b. move slowly when moving from lying to standing. c. increase the dietary intake of high-potassium foods. d. make an appointment with the dietitian for teaching.

ANS: C The ACE inhibitors cause retention of potassium by the kidney, so hyperkalemia is a possible adverse effect. The other teaching by the new RN is appropriate for a patient with newly diagnosed hypertension who has just started therapy with enalapril.

A patient has just been diagnosed with hypertension and has been started on captopril . Which information is *most* important to include when teaching the patient about this drug? a. Include high-potassium foods such as bananas in the diet. b. Increase fluid intake if dryness of the mouth is a problem. c. Change position slowly to help prevent dizziness and falls. d. Check blood pressure (BP) in both arms before taking the drug.

ANS: C The angiotensin-converting enzyme (ACE) inhibitors frequently cause orthostatic hypotension, and patients should be taught to change position slowly to allow the vascular system time to compensate for the position change. Increasing fluid intake may counteract the effect of the drug, and the patient is taught to use gum or hard candy to relieve dry mouth. The BP should be taken in the nondominant arm by newly diagnosed patients in the morning, before taking the drug, and in the evening. Because ACE inhibitors cause potassium retention, increased intake of high-potassium foods is inappropriate.

Which finding about a patient with polycythemia vera is most important for the nurse to report to the health care provider? a. Hematocrit 55% b. Presence of plethora c. Calf swelling and pain d. Platelet count 450,000/mL

ANS: C The calf swelling and pain suggest that the patient may have developed a deep vein thrombosis, which will require diagnosis and treatment to avoid complications such as pulmonary embolus. The other findings will also be reported to the health care provider but are expected in a patient with this diagnosis

Which topic will the nurse plan to include in discharge teaching for a patient with systolic heart failure and an ejection fraction of 33%? a. Need to begin an aerobic exercise program several times weekly b. Use of salt substitutes to replace table salt when cooking and at the table c. Benefits and side effects of angiotensin-converting enzyme (ACE) inhibitors d. Importance of making an annual appointment with the primary care provider

ANS: C The core measures for the treatment of heart failure established by The Joint Commission indicate that patients with an ejection fraction (EF) <40% receive an ACE inhibitor to decrease the progression of heart failure. Aerobic exercise may not be appropriate for a patient with this level of heart failure, salt substitutes are not usually recommended because of the risk of hyperkalemia, and the patient will need to see the primary care provider more frequently than annually

Which action will the nurse include in the plan of care for a patient who has thalassemia major? a. Teach the patient to use iron supplements. b. Avoid the use of intramuscular injections. c. Administer iron chelation therapy as needed. d. Notify health care provider of hemoglobin 11g/dL.

ANS: C The frequent transfusions used to treat thalassemia major lead to iron toxicity in patients unless iron chelation therapy is consistently used. Iron supplementation is avoided in patients with thalassemia. There is no need to avoid intramuscular injections. The goal for patients with thalassemia major is to maintain a hemoglobin of 10 g/dL or greater

While assessing a 68-yr-old with ascites, the nurse also notes jugular venous distention (JVD) with the head of the patient's bed elevated 45 degrees. The nurse knows this finding indicates a. decreased fluid volume. b. jugular vein atherosclerosis. c. increased right atrial pressure. d. incompetent jugular vein valves.

ANS: C The jugular veins empty into the superior vena cava and then into the right atrium, so JVD with the patient sitting at a 45-degree angle reflects increased right atrial pressure. JVD is an indicator of excessive fluid volume (increased preload), not decreased fluid volume. JVD is not caused by incompetent jugular vein valves or atherosclerosis.

During change-of-shift report, the nurse obtains the following information about a hypertensive patient who received the first dose of nadolol (Corgard) during the previous shift. Which information indicates that the patient needs immediate intervention? a. The patient's pulse has dropped from 68 to 57 beats/min. b. The patient complains that the fingers and toes feel quite cold. c. The patient has developed wheezes throughout the lung fields. d. The patient's blood pressure (BP) reading is now 158/91 mm Hg.

ANS: C The most urgent concern for this patient is the wheezes, which indicate that bronchospasm (a common adverse effect of the noncardioselective b-blockers) is occurring. The nurse should immediately obtain an O2 saturation measurement, apply supplemental O2, and notify the health care provider. The mild decrease in heart rate and complaint of cold fingers and toes are associated with b-receptor blockade but do not require any change in therapy. The BP reading may indicate that a change in medication type or dose may be indicated. However, this is not as urgently needed as addressing the bronchospasm.

During change-of-shift report, the nurse obtains the following information about a hypertensive patient who received the first dose of nadolol (Corgard) during the previous shift. Which information indicates that the patient needs immediate intervention? a. The patient's most recent blood pressure (BP) reading is 158/91 mm Hg. b. The patient's pulse has dropped from 68 to 57 beats/minute. c. The patient has developed wheezes throughout the lung fields. d. The patient complains that the fingers and toes feel quite cold.

ANS: C The most urgent concern for this patient is the wheezes, which indicate that bronchospasm (a common adverse effect of the noncardioselective b-blockers) is occurring. The nurse should immediately obtain an oxygen saturation measurement, apply supplemental oxygen, and notify the health care provider. The mild decrease in heart rate and complaint of cold fingers and toes are associated with b-receptor blockade but do not require any change in therapy. The BP reading may indicate that a change in medication type or dose may be indicated. However, this is not as urgently needed as addressing the bronchospasm

During change-of-shift report, the nurse obtains this information about a hypertensive patient who received the first dose of propranolol (Inderal) during the previous shift. Which information indicates that the patient needs immediate intervention? a. The patient's most recent BP reading is 156/94 mm Hg. b. The patient's pulse has dropped from 64 to 58 beats/minute. c. The patient has developed wheezes throughout the lung fields. d. The patient complains that the fingers and toes feel quite cold.

ANS: C The most urgent concern for this patient is the wheezes, which indicate that bronchospasm (a common adverse effect of the noncardioselective β-blockers) is occurring. The nurse should immediately obtain an oxygen saturation measurement, apply supplemental oxygen, and notify the health care provider. The mild decrease in heart rate and complaint of cold fingers and toes are associated with β-receptor blockade but do not require any change in therapy. The BP reading may indicate that a change in medication type or dose may be indicated; however, this is not as urgently needed as addressing the bronchospasm.

Which action will the nurse in the hypertension clinic take in order to obtain an accurate baseline blood pressure (BP) for a new patient? a. Obtain a BP reading in each arm and average the results. b. Deflate the BP cuff at a rate of 5 to 10 mm Hg per second. c. Have the patient sit in a chair with the feet flat on the floor. d. Assist the patient to the supine position for BP measurements.

ANS: C The patient should be seated with the feet flat on the floor. The BP is obtained in both arms, but the results of the two arms are not averaged. The patient does not need to be in the supine position. The cuff should be deflated at 2 to 3 mm Hg per second.

An outpatient who has chronic heart failure returns to the clinic after 2 weeks of therapy with metoprolol (Toprol XL). Which assessment finding is most important for the nurse to report to the health care provider? a. 2+ pedal edema b. Heart rate of 56 beats/minute c. Blood pressure (BP) of 88/42 mm Hg d. Complaints of fatigue

ANS: C The patient's BP indicates that the dose of metoprolol may need to be decreased because of hypotension. Bradycardia is a frequent adverse effect of -adrenergic blockade, but the rate of 56 is not unusual with â-adrenergic blocker therapy. -Adrenergic blockade initially will worsen symptoms of heart failure in many patients, and patients should be taught that some increase in symptoms, such as fatigue and edema, is expected during the initiation of therapy with this class of drugs

A patient with possible disseminated intravascular coagulation arrives in the emergency department with a blood pressure of 82/40, temperature 102° F (38.9° C), and severe back pain. Which physician order will the nurse implement first? a. Administer morphine sulfate 4 mg IV. b. Give acetaminophen (Tylenol) 650 mg. c. Infuse normal saline 500 mL over 30 minutes. d. Schedule complete blood count and coagulation studies.

ANS: C The patient's blood pressure indicates hypovolemia caused by blood loss and should be addressed immediately to improve perfusion to vital organs. The other actions also are appropriate and should be rapidly implemented, but improving perfusion is the priority for this patient

A postoperative patient receiving a transfusion of packed red blood cells develops chills, fever, headache, and anxiety 35 minutes after the transfusion is started. After stopping the transfusion, what action should the nurse take? a. Draw blood for a new crossmatch. b. Send a urine specimen to the laboratory. c. Administer PRN acetaminophen (Tylenol). d. Give the PRN diphenhydramine (Benadryl).

ANS: C The patient's clinical manifestations are consistent with a febrile, nonhemolytic transfusion reaction. The transfusion should be stopped and antipyretics administered for the fever as ordered. A urine specimen is needed if an acute hemolytic reaction is suspected. Diphenhydramine (Benadryl) is used for allergic reactions. This type of reaction does not indicate incorrect crossmatching

A patient with a history of chronic heart failure is admitted to the emergency department (ED) with severe dyspnea and a dry, hacking cough. Which action should the nurse do first? a. Auscultate the abdomen. b. Check the capillary refill. c. Auscultate the breath sounds. d. Assess the level of orientation.

ANS: C This patient's severe dyspnea and cough indicate that acute decompensated heart failure (ADHF) is occurring. ADHF usually manifests as pulmonary edema, which should be detected and treated immediately to prevent ongoing hypoxemia and cardiac/respiratory arrest. The other assessments will provide useful data about the patient's volume status and also should be accomplished rapidly, but detection (and treatment) of pulmonary complications is the priority.

A patient with a history of chronic heart failure is admitted to the emergency department with severe dyspnea and a dry, hacking cough. Which action should the nurse do first? a. Auscultate the abdomen. b. Check the capillary refill. c. Auscultate the breath sounds. d. Ask about the patient's allergies.

ANS: C This patient's severe dyspnea and cough indicate that acute decompensated heart failure (ADHF) is occurring. ADHF usually manifests as pulmonary edema, which should be detected and treated immediately to prevent ongoing hypoxemia and cardiac/respiratory arrest. The other assessments will provide useful data about the patient's volume status and also should be accomplished rapidly, but detection (and treatment) of pulmonary complications is the priority.

A 52-year-old patient who has no previous history of hypertension or other health problems suddenly develops a BP of 188/106 mm Hg. After reconfirming the BP, it is appropriate for the nurse to tell the patient that a. a BP recheck should be scheduled in a few weeks. b. the dietary sodium and fat content should be decreased. c. there is an immediate danger of a stroke and hospitalization will be required. d. more diagnostic testing may be needed to determine the cause of the hypertension.

ANS: D A sudden increase in BP in a patient over age 50 with no previous hypertension history or risk factors indicates that the hypertension may be secondary to some other problem. The BP will need rapid treatment and ongoing monitoring. If the patient has no other risk factors, a stroke in the immediate future is unlikely. There is no indication that dietary salt or fat intake have contributed to this sudden increase in BP, and reducing intake of salt and fat alone will not be adequate to reduce this BP to an acceptable level.

A 56-year-old patient who has no previous history of hypertension or other health problems suddenly develops a blood pressure (BP) of 198/110 mm Hg. After reconfirming the BP, it is appropriate for the nurse to tell the patient that a. a BP recheck should be scheduled in a few weeks. b. dietary sodium and fat content should be decreased. c. there is an immediate danger of a stroke and hospitalization will be required. d. diagnosis of a possible cause, treatment, and ongoing monitoring will be needed.

ANS: D A sudden increase in BP in a patient over age 50 with no previous hypertension history or risk factors indicates that the hypertension may be secondary to some other problem. The BP will need treatment and ongoing monitoring. If the patient has no other risk factors, a stroke in the immediate future is unlikely. There is no indication that dietary salt or fat intake have contributed to this sudden increase in BP, and reducing intake of salt and fat alone will not be adequate to reduce this BP to an acceptable level

Following an acute myocardial infarction, a previously healthy 63-yr-old develops clinical manifestations of heart failure. The nurse anticipates discharge teaching will include information about a. -Adrenergic blockers. b. calcium channel blockers. c. digitalis and potassium therapy regimens. d. angiotensin-converting enzyme (ACE) inhibitors.

ANS: D ACE inhibitor therapy is currently recommended to prevent the development of heart failure in patients who have had a myocardial infarction and as a first-line therapy for patients with chronic heart failure. Digoxin therapy for heart failure is no longer considered a first-line measure, and digoxin is added to the treatment protocol when therapy with other drugs such as ACE-inhibitors, diuretics, and -adrenergic blockers is insufficient. Calcium channel blockers are not generally used in the treatment of heart failure. The -adrenergic blockers are not used as initial therapy for new onset heart failure.

Following an acute myocardial infarction, a previously healthy 63-year-old develops clinical manifestations of heart failure. The nurse anticipates discharge teaching will include information about a. digitalis preparations. b. -adrenergic blockers. c. calcium channel blockers. d. angiotensin-converting enzyme (ACE) inhibitors.

ANS: D ACE inhibitor therapy is currently recommended to prevent the development of heart failure in patients who have had a myocardial infarction and as a first-line therapy for patients with chronic heart failure. Digoxin therapy for heart failure is no longer considered a first-line measure, and digoxin is added to the treatment protocol when therapy with other medications such as ACE-inhibitors, diuretics, and -adrenergic blockers is insufficient. Calcium channel blockers are not generally used in the treatment of heart failure. The -adrenergic blockers are not used as initial therapy for new onset heart failure.

A patient who has been receiving a heparin infusion and warfarin (Coumadin) for a deep vein thrombosis (DVT) is diagnosed with heparin-induced thrombocytopenia (HIT) when her platelet level drops to 110,000/µL. Which action will the nurse include in the plan of care? a. Use low-molecular-weight heparin (LMWH) only. b. Administer the warfarin (Coumadin) at the scheduled time. c. Teach the patient about the purpose of platelet transfusions. d. Discontinue heparin and flush intermittent IV lines using normal saline.

ANS: D All heparin is discontinued when the HIT is diagnosed. The patient should be instructed to never receive heparin or LMWH. Warfarin is usually not given until the platelet count has returned to 150,000/µL. The platelet count does not drop low enough in HIT for a platelet transfusion, and platelet transfusions increase the risk for thrombosis

A patient who has just been admitted with pulmonary edema is scheduled to receive the following medications. Which medication should the nurse question before giving? a. captopril 25 mg b. furosemide (Lasix) 60 mg c. digoxin (Lanoxin) 0.125 mg d. carvedilol (Coreg) 3.125 mg

ANS: D Although carvedilol is appropriate for the treatment of chronic heart failure, it is not used for patients with acute decompensated heart failure (ADHF) because of the risk of worsening the heart failure. The other drugs are appropriate for the patient with ADHF.

A patient who has just been admitted with pulmonary edema is scheduled to receive the following medications. Which medication should the nurse question before giving? a. Furosemide (Lasix) 60 mg b. Captopril (Capoten) 25 mg c. Digoxin (Lanoxin) 0.125 mg d. Carvedilol (Coreg) 3.125 mg

ANS: D Although carvedilol is appropriate for the treatment of chronic heart failure, it is not used for patients with acute decompensated heart failure (ADHF) because of the risk of worsening the heart failure. The other medications are appropriate for the patient with ADHF.

The nurse has just finished teaching a hypertensive patient about the newly prescribed quinapril (Accupril). Which patient statement indicates that more teaching is needed? a. "The medication may not work as well if I take any aspirin." b. "The doctor may order a blood potassium level occasionally." c. "I will call the doctor if I notice that I have a frequent cough." d. "I won't worry if I have a little swelling around my lips and face."

ANS: D Angioedema occurring with angiotensin-converting enzyme (ACE) inhibitor therapy is an indication that the ACE inhibitor should be discontinued. The patient should be taught that if any swelling of the face or oral mucosa occurs, the health care provider should be immediately notified because this could be life threatening. The other patient statements indicate that the patient has an accurate understanding of ACE inhibitor therapy.

Which statement by a patient indicates good understanding of the nurse's teaching about prevention of sickle cell crisis? a. "Home oxygen therapy is frequently used to decrease sickling." b. "There are no effective medications that can help prevent sickling." c. "Routine continuous dosage narcotics are prescribed to prevent a crisis." d. "Risk for a crisis is decreased by having an annual influenza vaccination."

ANS: D Because infection is the most common cause of a sickle cell crisis, influenza, Haemophilus influenzae, pneumococcal pneumonia, and hepatitis immunizations should be administered. Although continuous dose opioids and oxygen may be administered during a crisis, patients do not receive these therapies to prevent crisis. Hydroxyurea (Hydrea) is a medication used to decrease the number of sickle cell crises

Which patient information is most important for the nurse to monitor when evaluating the effectiveness of deferoxamine (Desferal) for a patient with hemochromatosis? a. Skin color b. Hematocrit c. Liver function d. Serum iron level

ANS: D Because iron chelating agents are used to lower serum iron levels, the most useful information will be the patient's iron level. The other parameters will also be monitored, but are not the most important to monitor when determining the effectiveness of deferoxamine

The nurse is caring for a patient who is receiving IV furosemide (Lasix) and morphine for the treatment of acute decompensated heart failure (ADHF) with severe orthopnea. Which clinical finding is the best indicator that the treatment has been effective? a. Weight loss of 2 pounds in 24 hours b. Hourly urine output greater than 60 mL c. Reduction in patient complaints of chest pain d. Reduced dyspnea with the head of bed at 30 degrees

ANS: D Because the patient's major clinical manifestation of ADHF is orthopnea (caused by the presence of fluid in the alveoli), the best indicator that the medications are effective is a decrease in dyspnea with the head of the bed at 30 degrees. The other assessment data also may indicate that diuresis or improvement in cardiac output has occurred, but are not as specific to evaluating this patient's response.

The nurse is caring for a patient who is receiving IV furosemide (Lasix) and morphine for the treatment of acute decompensated heart failure (ADHF) with severe orthopnea. Which clinical finding is the best indicator that the treatment has been effective? a. Weight loss of 2 lb in 24 hours b. Hourly urine output greater than 60 mL c. Reduction in patient complaints of chest pain d. Reduced dyspnea with the head of bed at 30 degrees

ANS: D Because the patient's major clinical manifestation of ADHF is orthopnea (caused by the presence of fluid in the alveoli), the best indicator that the medications are effective is a decrease in dyspnea with the head of the bed at 30 degrees. The other assessment data may also indicate that diuresis or improvement in cardiac output has occurred but are not as specific to evaluating this patient's response.

The nurse is caring for a 70-year-old who uses hydrochlorothiazide (HydroDIURIL) and enalapril (Norvasc), but whose self-monitored blood pressure (BP) continues to be elevated. Which patient information may indicate a need for a change? a. Patient takes a daily multivitamin tablet. b. Patient checks BP daily just after getting up. c. Patient drinks wine three to four times a week. d. Patient uses ibuprofen (Motrin) daily to treat osteoarthritis.

ANS: D Because use of nonsteroidal antiinflammatory drugs (NSAIDs) can prevent adequate BP control, the patient may need to avoid the use of ibuprofen. A multivitamin tablet will help supply vitamin D, which may help lower BP. BP decreases while sleeping, so self-monitoring early in the morning will result in obtaining pressures that are at their lowest. The patient's alcohol intake is not excessive

The nurse is caring for a 70-yr-old patient who uses hydrochlorothiazide and enalapril (Norvasc) but whose self-monitored blood pressure (BP) continues to be elevated. Which patient information may indicate a need for a change? a. Patient takes a daily multivitamin tablet. b. Patient checks BP daily just after getting up. c. Patient drinks wine three to four times a week. d. Patient uses ibuprofen (Motrin) treat osteoarthritis.

ANS: D Because use of nonsteroidal antiinflammatory drugs (NSAIDs) can prevent adequate BP control, the patient may need to avoid the use of ibuprofen. A multivitamin tablet will help supply vitamin D, which may help lower BP. BP decreases while sleeping, so self-monitoring early in the morning will result in obtaining pressures that are at their lowest. The patient's alcohol intake is not excessive.

Which laboratory test will the nurse use to determine whether filgrastim (Neupogen) is effective for a patient with acute lymphocytic leukemia who is receiving chemotherapy? a. Platelet count b. Reticulocyte count c. Total lymphocyte count d. Absolute neutrophil count

ANS: D Filgrastim increases the neutrophil count and function in neutropenic patients. Although total lymphocyte, platelet, and reticulocyte counts also are important to monitor in this patient, the absolute neutrophil count is used to evaluate the effects of filgrastim

After the nurse teaches the patient with stage 1 hypertension about diet modifications that should be implemented, which diet choice indicates that the teaching has been effective? a. The patient avoids eating nuts or nut butters. b. The patient restricts intake of chicken and fish. c. The patient has two cups of coffee in the morning. d. The patient has a glass of low-fat milk with each meal.

ANS: D For the prevention of hypertension, the Dietary Approaches to Stop Hypertension (DASH) recommendations include increasing the intake of calcium-rich foods. Caffeine restriction and decreased protein intake are not included in the recommendations. Nuts are high in beneficial nutrients and 4 to 5 servings weekly are recommended in the DASH diet.

Which action will be included in the plan of care when the nurse is caring for a patient who is receiving nicardipine (Cardene) to treat a hypertensive emergency? a. Keep the patient NPO to prevent aspiration caused by nausea and possible vomiting. b. Organize nursing activities so that the patient has undisturbed sleep for 6 to 8 hours at night. c. Assist the patient up in the chair for meals to avoid complications associated with immobility. d. Use an automated noninvasive blood pressure machine to obtain frequent blood pressure (BP) measurements.

ANS: D Frequent monitoring of BP is needed when the patient is receiving rapid-acting IV antihypertensive medications. This can be most easily accomplished with an automated BP machine or arterial line. The patient will require frequent assessments, so allowing 6 to 8 hours of undisturbed sleep is not appropriate. When patients are receiving IV vasodilators, bed rest is maintained to prevent decreased cerebral perfusion and fainting. There is no indication that this patient is nauseated or at risk for aspiration, so an NPO status is unnecessary

Which action will be included in the plan of care when the nurse is caring for a patient who is receiving nicardipine (Cardene) to treat a hypertensive emergency? a. Organize nursing activities so that the patient has undisturbed sleep for 8 hours at night. b. Keep the patient NPO to prevent aspiration caused by nausea and possible vomiting. c. Assist the patient up in the chair for meals to avoid complications associated with immobility. d. Use an automated noninvasive blood pressure machine to obtain frequent measurements.

ANS: D Frequent monitoring of BP is needed when the patient is receiving rapid-acting IV antihypertensive medications. This can be most easily accomplished with an automated BP machine or arterial line. The patient will require frequent assessments, so allowing 8 hours of undisturbed sleep is not reasonable. When patients are receiving IV vasodilators, bed rest is maintained to prevent decreased cerebral perfusion and fainting. There is no indication that this patient is nauseated or at risk for aspiration, so an NPO status is unnecessary.

Which information should the nurse include when teaching a patient with newly diagnosed hypertension? a. Increasing physical activity will control blood pressure (BP) for most patients. b. Most patients are able to control BP through dietary changes. c. Annual BP checks are needed to monitor treatment effectiveness. d. Hypertension is usually asymptomatic until target organ damage occurs.

ANS: D Hypertension is usually asymptomatic until target organ damage has occurred. Lifestyle changes (e.g., physical activity, dietary changes) are used to help manage blood pressure, but drugs are needed for most patients. Home BP monitoring should be taught to the patient and findings checked by the health care provider frequently when starting treatment for hypertension and then every 3 months once stable

Which action should the nurse take when giving the initial dose of oral labetalol to a patient with hypertension? a. Encourage the use of hard candy to prevent dry mouth. b. Teach the patient that headaches often occur with this drug. c. Instruct the patient to call for help if heart palpitations occur. d. Ask the patient to request assistance before getting out of bed.

ANS: D Labetalol decreases sympathetic nervous system activity by blocking both a and b adrenergic receptors, leading to vasodilation and a decrease in heart rate, which can cause severe orthostatic hypotension. Heart palpitations, dry mouth, dehydration, and headaches are possible side effects of other antihypertensives.

A patient who has non-Hodgkin's lymphoma is receiving combination treatment with rituximab (Rituxan) and chemotherapy. Which patient assessment finding requires the most rapid action by the nurse? a. Anorexia b. Vomiting c. Oral ulcers d. Lip swelling

ANS: D Lip swelling in angioedema may indicate a hypersensitivity reaction to the rituximab. The nurse should stop the infusion and further assess for anaphylaxis. The other findings may occur with chemotherapy, but are not immediately life threatening

A critical action by the nurse caring for a patient with an acute exacerbation of polycythemia vera is to a. place the patient on bed rest. b. administer iron supplements. c. avoid use of aspirin products. d. monitor fluid intake and output.

ANS: D Monitoring hydration status is important during an acute exacerbation because the patient is at risk for fluid overload or underhydration. Aspirin therapy is used to decrease risk for thrombosis. The patient should be encouraged to ambulate to prevent deep vein thrombosis (DVT). Iron is contraindicated in patients with polycythemia vera

When a patient with hypertension who has a new prescription for atenolol (Tenormin) returns to the health clinic after 2 weeks for a follow-up visit, the BP is unchanged from the previous visit. Which action should the nurse take first? a. Provide information about the use of multiple drugs to treat hypertension. b. Teach the patient about the reasons for a possible change in drug therapy. c. Remind the patient that lifestyle changes also are important in BP control. d. Question the patient about whether the medication is actually being taken.

ANS: D Since noncompliance with antihypertensive therapy is common, the nurse's initial action should be to determine whether the patient is taking the atenolol as prescribed. The other actions also may be implemented, but these would be done after assessing patient compliance with the prescribed therapy.

After the nurse teaches the patient with stage 1 hypertension about diet modifications that should be implemented, which diet choice indicates that the teaching has been effective? a. The patient avoids eating nuts or nut butters. b. The patient restricts intake of dietary protein. c. The patient has only one cup of coffee in the morning. d. The patient has a glass of low-fat milk with each meal.

ANS: D The Dietary Approaches to Stop Hypertension (DASH) recommendations for prevention of hypertension include increasing the intake of calcium-rich foods. Caffeine restriction and decreased protein intake are not included in the recommendations. Nuts are high in beneficial nutrients and 4 to 5 servings weekly are recommended in the DASH diet.

A patient has just been diagnosed with hypertension and has a new prescription for captopril (Capoten). Which information is important to include when teaching the patient? a. Check BP daily before taking the medication. b. Increase fluid intake if dryness of the mouth is a problem. c. Include high-potassium foods such as bananas in the diet. d. Change position slowly to help prevent dizziness and falls.

ANS: D The angiotensin-converting enzyme (ACE) inhibitors frequently cause orthostatic hypotension, and patients should be taught to change position slowly to allow the vascular system time to compensate for the position change. Increasing fluid intake may counteract the effect of the medication, and the patient is taught to use gum or hard candy to relieve dry mouth. The BP does not need to be checked at home by the patient before taking the medication. Because ACE inhibitors cause potassium retention, increased intake of high-potassium foods is inappropriate.

A patient has just been diagnosed with hypertension and has been started on captopril (Capoten). Which information is important to include when teaching the patient about this medication? a. Check blood pressure (BP) in both arms before taking the medication. b. Increase fluid intake if dryness of the mouth is a problem. c. Include high-potassium foods such as bananas in the diet. d. Change position slowly to help prevent dizziness and falls.

ANS: D The angiotensin-converting enzyme (ACE) inhibitors frequently cause orthostatic hypotension, and patients should be taught to change position slowly to allow the vascular system time to compensate for the position change. Increasing fluid intake may counteract the effect of the medication, and the patient is taught to use gum or hard candy to relieve dry mouth. The BP should be taken in the nondominant arm by newly diagnosed patients in the morning, before taking the medication, and in the evening. Because ACE inhibitors cause potassium retention, increased intake of high-potassium foods is inappropriate

Following successful treatment of Hodgkin's lymphoma for a 55-year-old woman, which topic will the nurse include in patient teaching? a. Potential impact of chemotherapy treatment on fertility b. Application of soothing lotions to treat residual pruritus c. Use of maintenance chemotherapy to maintain remission d. Need for follow-up appointments to screen for malignancy

ANS: D The chemotherapy used in treating Hodgkin's lymphoma results in a high incidence of secondary malignancies; follow-up screening is needed. The fertility of a 55-year-old woman will not be impacted by chemotherapy. Maintenance chemotherapy is not used for Hodgkin's lymphoma. Pruritus is a clinical manifestation of lymphoma, but should not be a concern after treatment

Which topic will the nurse plan to include in discharge teaching for a patient with heart failure with reduced ejection fraction (HFrEF)? a. Need to begin an aerobic exercise program several times weekly b. Use of salt substitutes to replace table salt when cooking and at the table c. Importance of making an annual appointment with the health care provider d. Benefits and side effects of angiotensin-converting enzyme (ACE) inhibitors

ANS: D The core measures for the treatment of heart failure established by The Joint Commission indicate that patients with an ejection fraction below 40% should receive an ACE inhibitor to decrease the progression of heart failure. Aerobic exercise may not be appropriate for a patient with this level of heart failure, salt substitutes are not usually recommended because of the risk of hyperkalemia, and the patient will need to see the primary care provider more frequently than annually.

While admitting an 82-yr-old patient with acute decompensated heart failure to the hospital, the nurse learns that the patient lives alone and sometimes confuses the "water pill" with the "heart pill." When planning for the patient's discharge the nurse will facilitate a a. plan for around-the-clock care. b. consultation with a psychologist. c. transfer to a long-term care facility. d. referral to a home health care agency.

ANS: D The data about the patient suggest that assistance in developing a system for taking medications correctly at home is needed. A home health nurse will assess the patient's home situation and help the patient develop a method for taking the two medications as directed. There is no evidence that the patient requires services such as a psychologist consult, long-term care, or around-the-clock home care.

An outpatient who has chronic heart failure returns to the clinic after 2 weeks of therapy with metoprolol (Toprol XL). Which assessment finding is most important for the nurse to report to the health care provider? a. 2+ bilateral pedal edema b. Heart rate of 56 beats/min c. Complaints of increased fatigue d. Blood pressure (BP) of 88/42 mm Hg

ANS: D The patient's BP indicates that the dose of metoprolol may need to be decreased because of hypotension. Bradycardia is a frequent adverse effect of -adrenergic blockade, but the rate of 56 is not unusual though it may need to be monitored. -Adrenergic blockade initially will worsen symptoms of heart failure in many patients and patients should be taught that some increase in symptoms, such as fatigue and edema, is expected during the initiation of therapy with this class of drugs.

A 62-year old man with chronic anemia is experiencing increased fatigue and occasional palpitations at rest. The nurse would expect the patient's laboratory findings to include a. a hematocrit (Hct) of 38%. b. an RBC count of 4,500,000/mL. c. normal red blood cell (RBC) indices. d. a hemoglobin (Hgb) of 8.6 g/dL (86 g/L).

ANS: D The patient's clinical manifestations indicate moderate anemia, which is consistent with a Hgb of 6 to 10 g/dL. The other values are all within the range of normal

A patient in the emergency department complains of back pain and difficulty breathing 15 minutes after a transfusion of packed red blood cells is started. The nurse's first action should be to a. administer oxygen therapy at a high flow rate. b. obtain a urine specimen to send to the laboratory. c. notify the health care provider about the symptoms. d. disconnect the transfusion and infuse normal saline.

ANS: D The patient's symptoms indicate a possible acute hemolytic reaction caused by the transfusion. The first action should be to disconnect the transfusion and infuse normal saline. The other actions also are needed but are not the highest priority

A client is transported to the emergency department in respiratory distress after eating peanuts. The following interventions are ordered by the health care provider. Which intervention should the nurse complete first?

Administer epinephrine (adrenaline)

A client is transported to the emergency department in respiratory distress after eating peanuts. The following interventions are ordered by the health care provider. Which intervention should the nurse complete first?

Administer epinephrine (adrenaline).

A patient admitted with heart failure is anxious and reports shortness of breath. Which nursing actions would be appropriate to alleviate this patient's anxiety (select all that apply.)? Administer ordered morphine sulfate. Position patient in a semi-Fowler's position. Position patient on left side with head of bed flat. Instruct patient on the use of relaxation techniques. Use a calm, reassuring approach while talking to patient.

Administer ordered morphine sulfate. -Morphine sulfate reduces anxiety and may assist in reducing dyspnea. The patient should be positioned in semi-Fowler's position to improve ventilation that will reduce anxiety. Relaxation techniques and a calm reassuring approach will also serve to reduce anxiety. Position patient in a semi-Fowler's position. -Morphine sulfate reduces anxiety and may assist in reducing dyspnea. The patient should be positioned in semi-Fowler's position to improve ventilation that will reduce anxiety. Relaxation techniques and a calm reassuring approach will also serve to reduce anxiety Instruct patient on the use of relaxation techniques. -Morphine sulfate reduces anxiety and may assist in reducing dyspnea. The patient should be positioned in semi-Fowler's position to improve ventilation that will reduce anxiety. Relaxation techniques and a calm reassuring approach will also serve to reduce anxiety Use a calm, reassuring approach while talking to patient. -Morphine sulfate reduces anxiety and may assist in reducing dyspnea. The patient should be positioned in semi-Fowler's position to improve ventilation that will reduce anxiety. Relaxation techniques and a calm reassuring approach will also serve to reduce anxiety.

A client is rushed to the emergency department with symptoms of urticaria, wheezing, chest tightness, and difficulty in breathing. The client is most likely experiencing which type of shock?

Anaphylactic

A patient admitted with heart failure is anxious and reports shortness of breath. Which nursing actions would be appropriate to alleviate (giảm bớt) this patient's anxiety (select all that apply.)? Administer ordered morphine sulfate. Position patient in a semi-Fowler's position. Position patient on left side with head of bed flat. Instruct patient on the use of relaxation techniques. Use a calm, reassuring approach while talking to patient.

Administer ordered morphine sulfate. Position patient in a semi-Fowler's position. Instruct patient on the use of relaxation techniques. Use a calm, reassuring approach while talking to patient. Morphine sulfate reduces anxiety and may assist in reducing dyspnea. The patient should be positioned in semi-Fowler's position to improve ventilation that will reduce anxiety. Relaxation techniques and a calm reassuring approach will also serve to reduce anxiety.

there is electrical activity with no mechanical response

After defibrillation, the advanced cardiac life support nurse says that the patient has pulseless electrical activity, what is most important for the nurse to understand about this rhythm?

A 20-year-old male client is experiencing a severe immunologically mediated reaction in which histamines have been released into the blood. Select the type of reaction most likely occurring with this client.

Anaphylatic shock

D ) The catheter used to place the stent is usually inserted in the femoral artery. The affected leg is maintained in extension to prevent bleeding and hematoma formation. Chest tubes are not needed for this surgery and would not be secured to bed linens in any case. IV lines would be maintained as a saline lock in the event a line is needed for IV medications. Because the stent re-establishes blood flow to the myocardium, narcotic analgesics are rarely needed.

An African American male client with a history of diabetes and stroke returns from coronary angioplasty with stent placement. Which priority action will the nurse perform at this time? A) Discontinuing intravenous lines when taking oral fluids B) Securing chest tubes to the bedding C) Treating chest pain with intravenous morphine as needed D) Maintaining leg extension on the affected side

Candesartan, Ibesartan, Losartan, valsartan

Angiotension Receptor blockers (ARBS)

The nurse is administering a dose of digoxin to a patient with heart failure (HF). The nurse would become concerned with the possibility of digitalis toxicity if the patient reported which symptom? Muscle aches Constipation Pounding headache Anorexia and nausea (Biếng ăn và buồn nôn)

Anorexia (ˌanəˈreksēə) and nausea Anorexia, nausea, vomiting, blurred or yellow vision, and cardiac dysrhythmias are all signs of digitalis toxicity. The nurse would become concerned and notify the health care provider if the patient exhibited any of these symptoms.

The nurse is administering a dose of digoxin to a patient with heart failure (HF). The nurse would become concerned with the possibility of digitalis toxicity if the patient reported which symptom? Muscle aches Constipation Pounding headache Anorexia and nausea

Anorexia and nausea Anorexia, nausea, vomiting, blurred or yellow vision, and cardiac dysrhythmias are all signs of digitalis toxicity. The nurse would become concerned and notify the health care provider if the patient exhibited any of these symptoms.

Digitalis Toxicity

Anorexia, nausea, visual changes, confusion, bradycardia

Other medications for heart failure

Anticoagulants, antiarrhythmic drugs, statins Contraindicated: NSAIDs (ibuprofen, Motrin)

Regarding heart failure, the nurse knows that which of the following statements are correct? Select all that apply.

Aortic stenosis can cause left-sided failure. Cardiomyopathy is a common cause of heart failure. Compensated heart failure may be clinically asymptomatic.

Indicated for diastolic dysfunction

Beta-blockers

A patient who had bladder surgery 2 days ago develops acute decompensated heart failure (ADHF) with severe dyspnea. Which action by the nurse would be indicated first? Perform a bladder scan to assess for urinary retention. Restrict the patient's oral fluid intake to 500 mL per day. Assist the patient to a sitting position with arms on the overbed table. Instruct the patient to use pursed-lip breathing until the dyspnea subsides.

Assist the patient to a sitting position with arms on the overbed table The nurse should place the patient with ADHF in a high Fowler's position with the feet horizontal in the bed or dangling at the bedside. This position helps decrease venous return because of the pooling of blood in the extremities. This position also increases the thoracic capacity, allowing for improved ventilation. Pursed-lip breathing helps with obstructive air trapping but not with acute pulmonary edema. Restricting fluids takes considerable time to have an effect.

A patient who had bladder surgery 2 days ago develops acute decompensated heart failure (ADHF) with severe dyspnea. Which action by the nurse would be indicated first? Perform a bladder scan to assess for urinary retention. Restrict the patient's oral fluid intake to 500 mL per day. Assist the patient to a sitting position with arms on the overbed table. Instruct the patient to use pursed-lip breathing until the dyspnea subsides.

Assist the patient to a sitting position with arms on the overbed table. The nurse should place the patient with ADHF in a high Fowler's position with the feet horizontal in the bed or dangling at the bedside. This position helps decrease venous return because of the pooling of blood in the extremities. This position also increases the thoracic capacity, allowing for improved ventilation. Pursed-lip breathing helps with obstructive air trapping but not with acute pulmonary edema. Restricting fluids takes considerable time to have an effect.

Dysrhythmias can occur in patients with heart failure. The dysrhythmia that occurs most frequently in heart failure is which of the following?

Atrial fibrillation

The nurse is watching the cardiac monitor, and a patient's rhythm suddenly changes. There are no P waves. Instead there are fine, wavy lines between the QRS complexes. The QRS complexes measure 0.08 sec (narrow), but they occur irregularly with a rate of 120 beats/min. The nurse correctly interprets this rhythm as what? Sinus tachycardia Atrial fibrillation Ventricular fibrillation Ventricular tachycardia

Atrial fibrillation Atrial fibrillation is represented on the cardiac monitor by irregular R-R intervals and small fibrillatory (f) waves. There are no normal P waves because the atria are not truly contracting, just fibrillating. Sinus tachycardia is a sinus rate above 100 beats/minute with normal P waves. Ventricular fibrillation is seen on the ECG without a visible P wave; an unmeasurable heart rate, PR or QRS; and the rhythm is irregular and chaotic. Ventricular tachycardia is seen as three or more premature ventricular contractions (PVCs) that have distorted QRS complexes with regular or irregular rhythm, and the P wave is usually buried in the QRS complex without a measurable PR interval.

Atrial Flutter

Atrial rate 240-360 BPM (usually 300); Ventricular rate can vary from 60-150 BPM Regular or irregular rhythm Identical jagged, saw tooth repetitive waves; "F" or flutter waves; picket fence in appearance. One flutter wave is usually buried in the QRS. No PR interval; P:QRS conduction ratio may be 2:1, 3:1, 4:1 QRS complex usually narrow and normal

Atrial Fibrillation

Atrial rate 350-600 BPM; Ventricular rate 60-100 (controlled) > 100 (uncontrolled) Irregularly irregular ventricular rhythm Wavy undulating baseline; "f" or fibrillation waves that are irregularly shaped, rounded, or pointed and dissimilar. Coarse if "f" waves are large (greater than or equal to 1 mm). Fine if "f" waves are less than or equal to 1 mm. PR interval absent QRS complex usually normal

The nurse is preparing to administer a nitroglycerin patch to a patient. When providing instructions regarding the use of the patch, what should the nurse include in the teaching? Avoid high-potassium foods Avoid drugs to treat erectile dysfunction Avoid over-the-counter H2-receptor blockers Avoid nonsteroidal antiinflammatory drugs (NSAIDS)

Avoid drugs to treat erectile dysfunction The use of erectile drugs concurrent with nitrates creates a risk of severe hypotension and possibly death. High-potassium foods, NSAIDs, and H2-receptor blockers do not pose a risk in combination with nitrates.

The nurse is preparing to administer a nitroglycerin patch to a patient. When providing instructions regarding the use of the patch, what should the nurse include in the teaching? Avoid high-potassium foods Avoid drugs to treat erectile dysfunction Avoid over-the-counter H2-receptor blockers Avoid nonsteroidal antiinflammatory drugs (NSAIDS)

Avoid drugs to treat erectile dysfunction The use of erectile drugs concurrent with nitrates creates a risk of severe hypotension and possibly death. High-potassium foods, NSAIDs, and H2-receptor blockers do not pose a risk in combination with nitrates.

Physiological Integrity 21. A 19-year-old student comes to the student health center at the end of the semester complaining that, "My heart is skipping beats." An electrocardiogram (ECG) shows occasional premature ventricular contractions (PVCs). What action should the nurse take next? a. Start supplemental O2 at 2 to 3 L/min via nasal cannula. b. Ask the patient about current stress level and caffeine use. c. Ask the patient about any history of coronary artery disease. d. Have the patient taken to the hospital emergency department (ED).

B In a patient with a normal heart, occasional PVCs are a benign finding. The timing of the PVCs suggests stress or caffeine as possible etiologic factors. It is unlikely that the patient has coronary artery disease, and this should not be the first question the nurse asks. The patient is hemodynamically stable, so there is no indication that the patient needs to be seen in the ED or that oxygen needs to be administered. DIF: Cognitive Level: Apply (application) REF: 799 OBJ: Special Questions: Prioritization TOP: Nursing Process: Implementation MSC:

Physiological Integrity 16. A patient has ST segment changes that support an acute inferior wall myocardial infarction. Which lead would be best for monitoring the patient? a. I b. II c. V2 d. V6

B Leads II, III, and AVF reflect the inferior area of the heart and the ST segment changes. Lead II will best capture any electrocardiographic (ECG) changes that indicate further damage to the myocardium. The other leads do not reflect the inferior part of the myocardial wall and will not provide data about further ischemic changes in that area. DIF: Cognitive Level: Analyze (analysis) REF: 806 TOP: Nursing Process: Implementation MSC:

14. The nurse identifies the collaborative problem of potential complication: pulmonary edema for a patient in ADHF. When assessing the patient, the nurse will be most concerned about a. an apical pulse rate of 106 beats/min. b. an oxygen saturation of 88% on room air. c. weight gain of 1 kg (2.2 lb) over 24 hours. d. decreased hourly patient urinary output.

B Rationale: A decrease in oxygen saturation to less than 92% indicates hypoxemia. The nurse should administer supplemental oxygen immediately to the patient. An increase in apical pulse rate, 1-kg weight gain, and decreases in urine output also indicate worsening heart failure and require rapid nursing actions, but the low oxygen saturation rate requires the most immediate nursing action. Cognitive Level: Analysis Text Reference: pp. 829-830 Nursing Process: Assessment NCLEX: Physiological Integrity

22. A patient who is receiving dobutamine (Dobutrex) for the treatment of ADHF has all of the following nursing actions included in the plan of care. Which action will be best for the RN to delegate to an experienced LPN/LVN? a. Teach the patient the reasons for remaining on bed rest. b. Monitor the patient's BP every hour. c. Adjust the drip rate to keep the systolic BP >90 mm Hg. d. Call the health care provider about a decrease in urine output.

B Rationale: An experienced LPN/LVN would be able to monitor BP and would know to report significant changes to the RN. Teaching patients and making adjustments to the drip rate for vasoactive medications are RN-level skills. Because the health care provider may order changes in therapy based on the decrease in urine output, the RN should call the health care provider about the decreased urine output. Cognitive Level: Application Text Reference: pp. 827-829 Nursing Process: Planning NCLEX: Safe and Effective Care Environment

5. When the nurse is developing a teaching plan to prevent the development of heart failure in a patient with stage 1 hypertension, the information that is most likely to improve compliance with antihypertensive therapy is that a. hypertensive crisis may lead to development of acute heart failure in some patients. b. hypertension eventually will lead to heart failure by overworking the heart muscle. c. high BP increases risk for rheumatic heart disease. d. high systemic pressure precipitates papillary muscle rupture.

B Rationale: Hypertension is a primary cause of heart failure because the increase in ventricular afterload leads to ventricular hypertrophy and dilation. Hypertensive crisis may precipitate acute heart failure is some patients, but this patient with stage 1 hypertension may not be concerned about a crisis that happens only to some patients. Hypertension does not directly cause rheumatic heart disease (which is precipitated by infection with group A -hemolytic streptococcus) or papillary muscle rupture (which is caused by myocardial infarction/necrosis of the papillary muscle). Cognitive Level: Application Text Reference: p. 822 Nursing Process: Planning NCLEX: Health Promotion and Maintenance

Physiological Integrity 24. A patient whose heart monitor shows sinus tachycardia, rate 132, is apneic and has no palpable pulses. What is the first action that the nurse should take? a. Perform synchronized cardioversion. b. Start cardiopulmonary resuscitation (CPR). c. Administer atropine per agency dysrhythmia protocol. d. Provide supplemental oxygen via non-rebreather mask.

B The patient's clinical manifestations indicate pulseless electrical activity and the nurse should immediately start CPR. The other actions would not be of benefit to this patient. DIF: Cognitive Level: Apply (application) REF: 800 OBJ: Special Questions: Prioritization TOP: Nursing Process: Implementation MSC:

8. A patient admitted to the hospital with an exacerbation of chronic heart failure tells the nurse, "I felt fine when I went to bed, but I woke up in the middle of the night feeling like I was suffocating!" The nurse can best document this assessment information as a. pulsus alternans. b. paroxysmal nocturnal dyspnea. c. two-pillow orthopnea. d. acute bilateral pleural effusion.

B Rationale: Paroxysmal nocturnal dyspnea is caused by the reabsorption of fluid from dependent body areas when the patient is sleeping and is characterized by waking up suddenly with the feeling of suffocation. Pulsus alternans is the alternation of strong and weak peripheral pulses during palpation. Orthopnea indicates that the patient is unable to lie flat because of dyspnea. Pleural effusions develop over a longer time period. Cognitive Level: Comprehension Text Reference: p. 825 Nursing Process: Assessment NCLEX: Physiological Integrity

11. When developing a plan to decrease preload in the patient with heart failure, the nurse will include actions such as a. administering sedatives to promote rest and decrease myocardial oxygen demand. b. positioning the patient in a high-Fowler's position with the feet horizontal in the bed. c. administering oxygen per mask or nasal cannula. d. encouraging leg exercises to improve venous return.

B Rationale: Positioning the patient in a high-Fowler's position with the legs dependent will reduce preload by decreasing venous return to the right atrium. The other interventions may also be appropriate for patients with heart failure but will not help in decreasing preload. Cognitive Level: Application Text Reference: pp. 827-828 Nursing Process: Planning NCLEX: Physiological Integrity

10. The nurse working in the heart failure clinic will know that teaching for a 74-year-old patient with newly diagnosed heart failure has been effective when the patient a. says that the nitroglycerin patch will be used for any chest pain that develops. b. calls when the weight increases from 124 to 130 pounds in a week. c. tells the home care nurse that furosemide (Lasix) is taken daily at bedtime. d. makes an appointment to see the doctor at least once yearly.

B Rationale: Teaching for a patient with heart failure includes information about the need to weigh daily and notify the health care provider about an increase of 3 pounds in 2 days or 5 pounds in a week. Nitroglycerin patches are used primarily to reduce preload (not to prevent chest pain) in patients with heart failure and should be used daily, not on an "as necessary" basis. Diuretics should be taken earlier in the day to avoid nocturia and sleep disturbance. Heart failure is a chronic condition that will require frequent follow-up rather than an annual health care provider examination. Cognitive Level: Application Text Reference: pp. 826, 833-834, 838 Nursing Process: Evaluation NCLEX: Health Promotion and Maintenance

15. While admitting an 80-year-old patient with heart failure to the medical unit, the nurse obtains the information that the patient lives alone and sometimes confuses the "water pill" with the "heart pill." The nurse makes a note that discharge planning for the patient will need to include a. transfer to a dementia care service. b. referral to a home health care agency. c. placement in a long-term-care facility. d. arrangements for around-the-clock care.

B Rationale: The data about the patient suggest that assistance in developing a system for taking medications correctly at home is needed. A home health nurse will assess the patient's home situation and help the patient to develop a method for taking the two medications as directed. There is no evidence that the patient requires services such as dementia care, long-term-care, or around-the-clock home care. Cognitive Level: Application Text Reference: pp. 836-837 Nursing Process: Assessment NCLEX: Health Promotion and Maintenance

Physiological Integrity 5. The nurse notes that a patient's cardiac monitor shows that every other beat is earlier than expected, has no visible P wave, and has a QRS complex that is wide and bizarre in shape. How will the nurse document the rhythm? a. Ventricular couplets b. Ventricular bigeminy c. Ventricular R-on-T phenomenon d. Multifocal premature ventricular contractions

B Ventricular bigeminy describes a rhythm in which every other QRS complex is wide and bizarre looking. Pairs of wide QRS complexes are described as ventricular couplets. There is no indication that the premature ventricular contractions (PVCs) are multifocal or that the R-on-T phenomenon is occurring. DIF: Cognitive Level: Apply (application) REF: 799 TOP: Nursing Process: Assessment MSC:

Safe and Effective Care Environment 13. Which action should the nurse perform when preparing a patient with supraventricular tachycardia for cardioversion who is alert and has a blood pressure of 110/66 mm Hg? a. Turn the synchronizer switch to the "off" position. b. Give a sedative before cardioversion is implemented. c. Set the defibrillator/cardioverter energy to 360 joules. d. Provide assisted ventilations with a bag-valve-mask device.

B When a patient has a nonemergency cardioversion, sedation is used just before the procedure. The synchronizer switch is turned "on" for cardioversion. The initial level of joules for cardioversion is low (e.g., 50). Assisted ventilations are not indicated for this patient. DIF: Cognitive Level: Apply (application) REF: 802 TOP: Nursing Process: Implementation MSC:

When obtaining a health history of a patient admitted with a diagnosis of heart failure, which statement made by the patient supports the diagnosis of heart failure? A "I get hot and break out in a sweat during the night." B "I get out of breath when I go up a flight of stairs." C "I sometimes feel pain in the middle of my chest during exercise." D "I often feel pain in my lower legs when I take my walk."

B "I get out of breath when I go up a flight of stairs."

A patient diagnosed with mild heart failure is prescribed hydrochlorothiazide (Microzide). The healthcare provider should determine the teaching about the medication has been successful if the patient makes which of these statements? A "I might experience swelling in my legs when taking this medication." B "It is important for me to change positions slowly because I might become dizzy." C "This medication might cause me to have a decrease in my appetite." D "I should not worry if I experience a dry cough when taking this medication."

B "It is important for me to change positions slowly because I might become dizzy."

The laboratory results of a patient diagnosed with heart failure shows a serum digoxin (Lanoxin) level of 2.1 ng/mL. Which medication is appropriate to administer at this time? A Furosemide (Lasix) B Digoxin immune fab (DigiFab) C An increased dose of digoxin (Lanoxin) D Potassium chloride (K-tab)

B Digoxin immune fab (DigiFab)

A patient with a diagnosis of heart failure has been started on a nitroglycerin patch by his primary care provider. What should this patient be taught to avoid? A High-potassium foods B Drugs to treat erectile dysfunction C Nonsteroidal antiinflammatory drugs D Over-the-counter H2 -receptor blockers

B Drugs to treat erectile dysfunction The use of erectile drugs concurrent with nitrates creates a risk of severe hypotension and possibly death. High-potassium foods, NSAIDs, and H2-receptor blockers do not pose a risk in combination with nitrates.

What should the nurse recognize as an indication for the use of dopamine (Intropin) in the care of a patient with heart failure? A Acute anxiety B Hypotension and tachycardia C Peripheral edema and weight gain D Paroxysmal nocturnal dyspnea (PND)

B Hypotension and tachycardia Dopamine is a β-adrenergic agonist whose inotropic action is used for treatment of severe heart failure accompanied by hemodynamic instability. Such a state may be indicated by tachycardia accompanied by hypotension. PND, anxiety, edema, and weight gain are common signs and symptoms of heart failure, but these do not necessarily warrant the use of dopamine.

A patient is being assessed for possible heart failure. Which of these laboratory results will provide support this diagnosis? A Decreased serum sodium B Increased brain natriuretic peptide (BNP) C Decreased C-reactive protein D Increased creatine kinase

B Increased brain natriuretic peptide (BNP)

When assessing a patient with chronic heart failure, the healthcare provider would expect to identify which of these clinical manifestations? A Expiratory wheezing B Inspiratory crackles C Subcutaneous crepitus D Asymmetrical chest expansion

B Inspiratory crackles

The nurse is preparing to administer digoxin to a patient with heart failure. In preparation, laboratory results are reviewed with the following findings: sodium 139 mEq/L, potassium 5.6 mEq/L, chloride 103 mEq/L, and glucose 106 mg/dL. What should the nurse do next? A Withhold the daily dose until the following day. B Withhold the dose and report the potassium level. C Give the digoxin with a salty snack, such as crackers. D Give the digoxin with extra fluids to dilute the sodium level.

B Withhold the dose and report the potassium level The normal potassium level is 3.5 to 5.0 mEq/L. The patient is hyperkalemic, which makes the patient more prone to digoxin toxicity. For this reason, the nurse should withhold the dose and report the potassium level. The physician may order the digoxin to be given once the potassium level has been treated and decreases to within normal range.

A patient with a diagnosis of heart failure has been started on a nitroglycerin patch by his primary care provider. What should this patient be taught to avoid? A. High-potassium foods B. Drugs to treat erectile dysfunction C. Nonsteroidal antiinflammatory drugs D. Over-the-counter H2-receptor blockers

B. Drugs to treat erectile dysfunction The use of erectile drugs concurrent with nitrates creates a risk of severe hypotension and possibly death. High-potassium foods, NSAIDs, and H2-receptor blockers do not pose a risk in combination with nitrates.

What should the nurse recognize as an indication for the use of dopamine (Intropin) in the care of a patient with heart failure? A. Acute anxiety B. Hypotension and tachycardia C. Peripheral edema and weight gain D. Paroxysmal nocturnal dyspnea (PND)

B. Hypotension and tachycardia Dopamine is a β-adrenergic agonist whose inotropic action is used for treatment of severe heart failure accompanied by hemodynamic instability. Such a state may be indicated by tachycardia accompanied by hypotension. PND, anxiety, edema, and weight gain are common signs and symptoms of heart failure, but these do not necessarily warrant the use of dopamine.

The home care nurse visits a 73-year-old Hispanic woman with chronic heart failure. Which clinical manifestations, if assessed by the nurse, would indicate acute decompensated heart failure (pulmonary edema)? A. Fatigue, orthopnea, and dependent edema B. Severe dyspnea and blood-streaked, frothy sputum C. Temperature is 100.4o F and pulse is 102 beats/minute D. Respirations 26 breaths/minute despite oxygen by nasal cannula

B. Severe dyspnea and blood-streaked, frothy sputum Clinical manifestations of pulmonary edema include anxiety, pallor, cyanosis, clammy and cold skin, severe dyspnea, use of accessory muscles of respiration, a respiratory rate > 30 breaths per minute, orthopnea, wheezing, and coughing with the production of frothy, blood-tinged sputum. Auscultation of the lungs may reveal crackles, wheezes, and rhonchi throughout the lungs. The heart rate is rapid, and blood pressure may be elevated or decreased.

The nurse obtains a 6-second rhythm strip and charts the following analysis: Tab 1 Atrial data Rate: 70, regular Variable PR interval Independent beats Tab 2 Ventricular data Rate: 40, regular Isolated escape beats Tab 3 Additional data QRS: 0.04 sec P wave and QRS complexes unrelated What is the correct interpretation of this rhythm strip? A. Sinus arrhythmias B. Third-degree heart block C. Wenckebach phenomenon D. Premature ventricular contractions

B. Third-degree heart block Third-degree heart block represents a loss of communication between the atrium and ventricles from AV node dissociation. This is depicted on the rhythm strip as no relationship between the P waves (representing atrial contraction) and QRS complexes (representing ventricular contraction). The atria are beating totally on their own at 70 beats/min, whereas the ventricles are pacing themselves at 40 beats/min. Sinus dysrhythmia is seen with a slower heart rate with exhalation and an increased heart rate with inhalation. In Wenckebach heart block, there is a gradual lengthening of the PR interval until an atrial impulse is nonconducted and a QRS complex is blocked or missing. Premature ventricular contractions (PVCs) are the early occurrence of a wide, distorted QRS complex.

The nurse is preparing to administer digoxin to a patient with heart failure. In preparation, laboratory results are reviewed with the following findings: sodium 139 mEq/L, potassium 5.6 mEq/L, chloride 103 mEq/L, and glucose 106 mg/dL. What should the nurse do next? A. Withhold the daily dose until the following day. B. Withhold the dose and report the potassium level. C. Give the digoxin with a salty snack, such as crackers. D. Give the digoxin with extra fluids to dilute the sodium level.

B. Withhold the dose and report the potassium level. The normal potassium level is 3.5 to 5.0 mEq/L. The patient is hyperkalemic, which makes the patient more prone to digoxin toxicity. For this reason, the nurse should withhold the dose and report the potassium level. The physician may order the digoxin to be given once the potassium level has been treated and decreases to within normal range.

Which diagnostic test is most useful in differentiating dyspnea related to pulmonary effects of heart failure from dyspnea related to pulmonary disease?

BNP levels

Carvedilol (Coreg)

Beta Blocker Used for HR and chest pain Takes about 3 weeks to get into system

Carvedilol, Metoprolol

Beta blockers

What is the priority assessment by the nurse caring for a patient receiving IV nesiritide to treat heart failure? Urine output Lung sounds Blood pressure Respiratory rate

Blood pressure Although all identified assessments are appropriate for a patient receiving IV nesiritide, the priority assessment would be monitoring for hypotension, the main adverse effect of nesiritide.

An asymptomatic patient with acute decompensated heart failure (ADHF) suddenly becomes dyspneic. Before dangling the patient on the bedside, what should the nurse assess first? Urine output Heart rhythm Breath sounds Blood pressure

Blood pressure The nurse should evaluate the blood pressure before dangling the patient on the bedside because the blood pressure can decrease as blood pools in the periphery and preload decreases. If the patient's blood pressure is low or marginal, the nurse should put the patient in the semi-Fowler's position and use other measures to improve gas exchange.

An asymptomatic patient with acute decompensated heart failure (ADHF) suddenly becomes dyspneic. Before dangling the patient on the bedside, what should the nurse assess first? Urine output Heart rhythm Breath sounds Blood pressure

Blood pressure The nurse should evaluate the blood pressure before dangling the patient on the bedside because the blood pressure can decrease as blood pools in the periphery and preload decreases. If the patient's blood pressure is low or marginal, the nurse should put the patient in the semi-Fowler's position and use other measures to improve gas exchange.

What is the priority assessment by the nurse caring for a patient receiving IV nesiritide to treat heart failure? Urine output Lung sounds Blood pressure Respiratory rate

Blood pressure. Although all identified assessments are appropriate for a patient receiving IV nesiritide, the priority assessment would be monitoring for hypotension, the main adverse effect of nesiritide.

Which initial physical assessment finding would the nurse expect to be present in a patient with acute left-sided heart failure

Bubbling crackles and tachycardia

2. A patient with chronic heart failure who has been following a low-sodium diet tells the nurse at the clinic about a 5-pound weight gain in the last 3 days. The nurse's first action will be to a. ask the patient to recall the dietary intake for the last 3 days because there may be hidden sources of sodium in the patient's diet. b. instruct the patient in a low-calorie, low-fat diet because the weight gain has likely been caused by excessive intake of inappropriate foods. c. assess the patient for clinical manifestations of acute heart failure because an exacerbation of the chronic heart failure may be occurring. d. educate the patient about the use of diuretic therapy because it is likely that the patient will need medications to reduce the hypervolemia.

C Rationale: The 5-pound weight gain over 3 days indicates that the patient's chronic heart failure may be worsening; it is important that the patient be immediately assessed for other clinical manifestations of decompensation, such as lung crackles. A dietary recall to detect hidden sodium in the diet and teaching about diuretic therapy are appropriate interventions but are not the first nursing actions indicated. There is no evidence that the patient's weight gain is caused by excessive dietary intake of fat or calories, so the answer beginning "instruct the patient in a low-calorie, low-fat diet" describes an inappropriate action. Cognitive Level: Application Text Reference: p. 826 Nursing Process: Assessment NCLEX: Physiological Integrity

Safe and Effective Care Environment 23. A patient who is on the progressive care unit develops atrial flutter, rate 150, with associated dyspnea and chest pain. Which action that is included in the hospital dysrhythmia protocol should the nurse do first? a. Obtain a 12-lead electrocardiogram (ECG). b. Notify the health care provider of the change in rhythm. c. Give supplemental O2 at 2 to 3 L/min via nasal cannula. d. Assess the patient's vital signs including oxygen saturation.

C Because this patient has dyspnea and chest pain in association with the new rhythm, the nurse's initial actions should be to address the patient's airway, breathing, and circulation (ABC) by starting with oxygen administration. The other actions also are important and should be implemented rapidly. DIF: Cognitive Level: Apply (application) REF: 793 OBJ: Special Questions: Prioritization TOP: Nursing Process: Implementation MSC:

Physiological Integrity 3. A patient has a junctional escape rhythm on the monitor. The nurse will expect the patient to have a heart rate of _____ beats/minute. a. 15 to 20 b. 20 to 40 c. 40 to 60 d. 60 to 100

C If the sinoatrial (SA) node fails to discharge, the atrioventricular (AV) node will automatically discharge at the normal rate of 40 to 60 beats/minute. The slower rates are typical of the bundle of His and the Purkinje system and may be seen with failure of both the SA and AV node to discharge. The normal SA node rate is 60 to 100 beats/minute. DIF: Cognitive Level: Understand (comprehension) REF: 797 TOP: Nursing Process: Assessment MSC:

Physiological Integrity 10. Which information will the nurse include when teaching a patient who is scheduled for a radiofrequency catheter ablation for treatment of atrial flutter? a. The procedure will prevent or minimize the risk for sudden cardiac death. b. The procedure will use cold therapy to stop the formation of the flutter waves. c. The procedure will use electrical energy to destroy areas of the conduction system. d. The procedure will stimulate the growth of new conduction pathways between the atria.

C Radiofrequency catheter ablation therapy uses electrical energy to "burn" or ablate areas of the conduction system as definitive treatment of atrial flutter (i.e., restore normal sinus rhythm) and tachydysrhythmias. All other statements regarding the procedure are incorrect. DIF: Cognitive Level: Apply (application) REF: 805 TOP: Nursing Process: Implementation MSC:

20. An elderly patient with a 40-pack-year history of smoking and a recent myocardial infarction is admitted to the medical unit with acute shortness of breath; the nurse need to rule out pneumonia versus heart failure. The diagnostic test that the nurse will monitor to help in determining whether the patient has heart failure is a. 12-lead electrocardiogram (ECG). b. arterial blood gases (ABGs). c. B-type natriuretic peptide (BNP). d. serum creatine kinase (CK).

C Rationale: BNP is secreted when ventricular pressures increase, as with heart failure, and elevated BNP indicates a probable or very probable diagnosis of heart failure. 12-lead ECGs, ABGs, and CK may also be used in determining the causes or effects of heart failure but are not as clearly diagnostic of heart failure as BNP. Cognitive Level: Application Text Reference: p. 827 Nursing Process: Assessment NCLEX: Physiological Integrity

16. A home health care patient has recently started taking oral digoxin (Lanoxin) and furosemide (Lasix) for control of heart failure. The patient data that will require the most immediate action by the nurse is if the patient's a. weight increases from 120 pounds to 122 pounds over 3 days. b. liver is palpable 2 cm below the ribs on the right side. c. serum potassium level is 3.0 mEq/L after 1 week of therapy. d. has 1 to 2+ edema in the feet and ankles in the morning.

C Rationale: Hypokalemia potentiates the actions of digoxin and increases the risk for digoxin toxicity, which can cause life-threatening dysrhythmias. The other data indicate that the patient's heart failure requires more effective therapies, but they do not require nursing action as rapidly as the low serum potassium level. Cognitive Level: Application Text Reference: pp. 832-833 Nursing Process: Assessment NCLEX: Physiological Integrity

3. During assessment of a 72-year-old with ankle swelling, the nurse notes jugular venous distention (JVD) with the head of the patient's bed elevated 45 degrees. The nurse knows this finding indicates a. decreased fluid volume. b. incompetent jugular vein valves. c. elevated right atrial pressure. d. jugular vein atherosclerosis.

C Rationale: The jugular veins empty into the superior vena cava and then into the right atrium, so JVD with the patient sitting at a 45-degree angle reflects elevated right atrial pressure. JVD is an indicator of excessive fluid volume (increased preload), not decreased fluid volume; it is not caused by incompetent jugular vein valves or atherosclerosis. Cognitive Level: Comprehension Text Reference: p. 825 Nursing Process: Assessment NCLEX: Physiological Integrity

9. During a visit to an elderly patient with chronic heart failure, the home care nurse finds that the patient has severe dependent edema and that the legs appear to be weeping serous fluid. Based on these data, the best nursing diagnosis for the patient is a. activity intolerance related to venous congestion. b. disturbed body image related to massive leg swelling. c. impaired skin integrity related to peripheral edema. d. impaired gas exchange related to chronic heart failure.

C Rationale: The patient's findings of severe dependent edema and weeping serous fluid from the legs support the nursing diagnosis of impaired skin integrity. There is less evidence for the nursing diagnoses of activity intolerance, disturbed body image, and impaired gas exchange, although the nurse will further assess the patient to determine whether there are other clinical manifestations of heart failure to indicate that these diagnoses are appropriate. Cognitive Level: Application Text Reference: p. 836 Nursing Process: Diagnosis NCLEX: Physiological Integrity

Safe and Effective Care Environment 28. A patient reports dizziness and shortness of breath for several days. During cardiac monitoring in the emergency department (ED), the nurse obtains the following electrocardiographic (ECG) tracing. The nurse interprets this heart rhythm as a. junctional escape rhythm. b. accelerated idioventricular rhythm. c. third-degree atrioventricular (AV) block. d. sinus rhythm with premature atrial contractions (PACs).

C The inconsistency between the atrial and ventricular rates and the variable P-R interval indicate that the rhythm is third-degree AV block. Sinus rhythm with PACs will have a normal rate and consistent P-R intervals with occasional PACs. An accelerated idioventricular rhythm will not have visible P waves. DIF: Cognitive Level: Apply (application) REF: 798 TOP: Nursing Process: Assessment MSC:

Physiological Integrity 29. A patient who is complaining of a "racing" heart and feeling "anxious" comes to the emergency department. The nurse places the patient on a heart monitor and obtains the following electrocardiographic (ECG) tracing. Which action should the nurse take next? a. Prepare to perform electrical cardioversion. b. Have the patient perform the Valsalva maneuver. c. Obtain the patient's vital signs including oxygen saturation. d. Prepare to give a -blocker medication to slow the heart rate.

C The patient has sinus tachycardia, which may have multiple etiologies such as pain, dehydration, anxiety, and myocardial ischemia. Further assessment is needed before determining the treatment. Vagal stimulation or -blockade may be used after further assessment of the patient. Electrical cardioversion is used for some tachydysrhythmias, but would not be used for sinus tachycardia. DIF: Cognitive Level: Analyze (analysis) REF: 793 OBJ: Special Questions: Prioritization TOP: Nursing Process: Implementation MSC:

Physiological Integrity 26. Which nursing action can the registered nurse (RN) delegate to experienced unlicensed assistive personnel (UAP) working as a telemetry technician on the cardiac care unit? a. Decide whether a patient's heart rate of 116 requires urgent treatment. b. Monitor a patient's level of consciousness during synchronized cardioversion. c. Observe cardiac rhythms for multiple patients who have telemetry monitoring. d. Select the best lead for monitoring a patient admitted with acute coronary syndrome.

C UAP serving as telemetry technicians can monitor cardiac rhythms for individuals or groups of patients. Nursing actions such as assessment and choice of the most appropriate lead based on ST segment elevation location require RN-level education and scope of practice. DIF: Cognitive Level: Analyze (analysis) REF: 15 OBJ: Special Questions: Delegation TOP: Nursing Process: Planning MSC:

A 54-year-old male patient who had bladder surgery 2 days ago develops acute decompensated heart failure (ADHF) with severe dyspnea. Which action by the nurse would be indicated first? A Perform a bladder scan to assess for urinary retention. B Restrict the patient's oral fluid intake to 500 mL per day. C Assist the patient to a sitting position with arms on the overbed table. D Instruct the patient to use pursed-lip breathing until the dyspnea subsides.

C Assist the patient to a sitting position with arms on the overbed table. The nurse should place the patient with ADHF in a high Fowler's position with the feet horizontal in the bed or dangling at the bedside. This position helps decrease venous return because of the pooling of blood in the extremities. This position also increases the thoracic capacity, allowing for improved ventilation. Pursed-lip breathing helps with obstructive air trapping but not with acute pulmonary edema. Restricting fluids takes considerable time to have an effect.

What is the priority assessment by the nurse caring for a patient receiving IV nesiritide (Natrecor) to treat heart failure? A Urine output B Lung sounds C Blood pressure D Respiratory rate

C Blood pressure Although all identified assessments are appropriate for a patient receiving IV nesiritide, the priority assessment would be monitoring for hypotension, the main adverse effect of nesiritide.

A patient who has a history of pulmonary valve stenosis tells the healthcare provider, "I don't have a lot of energy anymore, and both of my feet get swollen in the late afternoon." Which of these problems does the healthcare provider conclude is the likely cause of these clinical findings? A Acute pericarditis B Peripheral artery disease C Right ventricular failure D Deep vein thrombosis (DVT)

C Right ventricular failure

The nurse prepares to administer digoxin (Lanoxin) 0.125 mg to an 82-year-old man admitted with influenza and a history of chronic heart failure. What should the nurse assess before giving the medication? A Prothrombin time B Urine specific gravity C Serum potassium level D Hemoglobin and hematocrit

C Serum potassium level Serum potassium should be monitored because hypokalemia increases the risk for digoxin toxicity. Changes in prothrombin time, urine specific gravity, and hemoglobin or hematocrit would not require holding the digoxin dose.

A patient has been diagnosed with early left ventricular heart failure (HF). The nurse knows that the following changes occur as the disease progresses. In which order do the changes involved in the development of dyspnea associated with left ventricular HF occur? (Answer with a letter followed by a comma and a space (e.g. A, B, C, D).) A. Inadequate alveolar gas exchange B. Elevated pressure in the left atrium C. Ineffective ventricular contractility D. Fluid leaking into interstitial spaces

C, B, D, A In left ventricular HF, ineffective ventricular contractions impair the normal forward flow of blood to the body. As a result, fluid returning to the heart from the lungs backs up in the heart and increases the pressure in the left atrium. If contractility continues to falter, the blood continues to back up into the pulmonary vasculature and eventually can result in fluid leaking into the interstitial spaces and alveoli of the lungs. This abnormal fluid in the parenchyma and alveoli impairs gas exchange, which causes dyspnea.

The nurse is caring for a patient who is 24 hours postpacemaker insertion. Which nursing intervention is most appropriate at this time? A. Reinforcing the pressure dressing as needed B. Encouraging range-of-motion exercises of the involved arm C. Assessing the incision for any redness, swelling, or discharge D. Applying wet-to-dry dressings every 4 hours to the insertion site

C. Assessing the incision for any redness, swelling, or discharge After pacemaker insertion, it is important for the nurse to observe signs of infection by assessing for any redness, swelling, or discharge from the incision site. The nonpressure dressing is kept dry until removed, usually 24 hours postoperatively. It is important for the patient to limit activity of the involved arm to minimize pacemaker lead displacement.

A 54-year-old male patient who had bladder surgery 2 days ago develops acute decompensated heart failure (ADHF) with severe dyspnea. Which action by the nurse would be indicated first? A. Perform a bladder scan to assess for urinary retention. B. Restrict the patient's oral fluid intake to 500 mL per day. C. Assist the patient to a sitting position with arms on the overbed table. D. Instruct the patient to use pursed-lip breathing until the dyspnea subsides.

C. Assist the patient to a sitting position with arms on the overbed table. The nurse should place the patient with ADHF in a high Fowler's position with the feet horizontal in the bed or dangling at the bedside. This position helps decrease venous return because of the pooling of blood in the extremities. This position also increases the thoracic capacity, allowing for improved ventilation. Pursed-lip breathing helps with obstructive air trapping but not with acute pulmonary edema. Restricting fluids takes considerable time to have an effect.

What is the priority assessment by the nurse caring for a patient receiving IV nesiritide (Natrecor) to treat heart failure? A. Urine output B. Lung sounds C. Blood pressure D. Respiratory rate

C. Blood pressure Although all identified assessments are appropriate for a patient receiving IV nesiritide, the priority assessment would be monitoring for hypotension, the main adverse effect of nesiritide.

Which statement best describes the electrical activity of the heart represented by measuring the PR interval on the ECG? A. The length of time it takes to depolarize the atrium B. The length of time it takes for the atria to depolarize and repolarize C. The length of time for the electrical impulse to travel from the SA node to the Purkinje fibers D. The length of time it takes for the electrical impulse to travel from the SA node to the AV node

C. The length of time for the electrical impulse to travel from the SA node to the Purkinje fibers The electrical impulse in the heart must travel from the SA node through the AV node and into the Purkinje fibers in order for synchronous atrial and ventricular contraction to occur. When measuring the PR interval (the time from the beginning of the P wave to the beginning of the QRS), the nurse is identifying the length of time it takes for the electrical impulse to travel from the SA node to the Purkinje fibers. The P wave represents the length of time it takes for the impulse to travel from the SA node through the atrium causing depolarization of the atria (atrial contraction). Atrial repolarization occurs during ventricular depolarization and is hidden by the QRS complex. The length of time it takes for the electrical impulse to travel from the SA node to the AV node is the flat line between the end of the P wave and the beginning of the Q wave on the ECG and is not usually measured.

Blood pressure = ______ x _______

CO x SVR

The health care provider is reviewing diagnostic tests that were ordered for a client diagnosed with heart failure. Select the test that would provide information about the client's ejection fraction and ventricular preload.

Cardiac computerized axial tomography (CCT)

A patient is scheduled for a heart transplant. Beyond the first year after a heart transplant, the nurse knows that what is a major cause of death? Infection Acute rejection Immunosuppression Cardiac vasculopathy

Cardiac vasculopathy Beyond the first year after a heart transplant, malignancy (especially lymphoma) and cardiac vasculopathy (accelerated coronary artery disease) are the major causes of death. During the first year after transplant, infection and acute rejection are the major causes of death. Immunosuppressive therapy will be used for posttransplant management to prevent rejection and increases the patient's risk of an infection.

A patient is scheduled for a heart transplant. Beyond the first year after a heart transplant, the nurse knows that what is a major cause of death? Infection Acute rejection Immunosuppression Cardiac vasculopathy

Cardiac vasculopathy Beyond the first year after a heart transplant, malignancy (ác tính) (especially lymphoma) and cardiac vasculopathy (accelerated coronary artery disease) are the major causes of death. During the first year after transplant, infection and acute rejection are the major causes of death. Immunosuppressive therapy will be used for posttransplant management to prevent rejection and increases the

The nurse is caring for a client who has just experienced an acute myocardial infarction and is diagnosed with "pump failure." The nurse is aware that the client is experiencing which type of shock?

Cardiogenic

ACE inhibitors

Cause agitative cough=non compliant Cannot give to asthma pt causes bronchospasm Treats mild HTN Teach pt to change position slowly

beta blockers (lol)

Check BP and HR first Make pt feel disconnected, hard to get use to Drop BP and HR to orthostatic point Erectile disfunction, depression on men

A client with congestive heart failure, CHF, is prescribed digoxin (Lanoxin) and furosemide (Lasix). Nursing interventions will include: (Select all that apply.) Encourage intake of water and fruit juices. Restrict intake of green, leafy vegetables. Checking apical pulse before administering medication. Monitor hemoglobin and hematocrit levels. Monitor serum electrolytes.

Checking apical pulse before administering medication. Monitor serum electrolytes. Rationale: Digoxin is a cardiac glycoside. which can slow heart rate, and an apical heart rate is checked prior to administration. Lasix is a loop diuretic used in treatment of CHF, which promotes not only water loss, but also loss of electrolytes. A low potassium level increases risk of digoxin toxicity. Fluids are often restricted with CHF. H and H level do not need to be checked, and green, leafy vegetables would not need to be restricted.

A patient with a long-standing history of heart failure recently qualified for hospice care. What measure should the nurse now prioritize when providing care for this patient? Taper the patient off his current medications. Continue education for the patient and his family. Pursue experimental therapies or surgical options. Choose interventions to promote comfort and prevent suffering.

Choose interventions to promote comfort and prevent suffering. The central focus of hospice care is the promotion of comfort and the prevention of suffering. Patient education should continue, but providing comfort is paramount. Medications should be continued unless they are not tolerated. Experimental therapies and surgeries are not commonly used in the care of hospice patients.

A patient with a long-standing history of heart failure recently qualified for hospice care. What measure should the nurse now prioritize when providing care for this patient? Taper the patient off his current medications. Continue education for the patient and his family. Pursue experimental therapies or surgical options. Choose interventions to promote comfort and prevent suffering.

Choose interventions to promote comfort and prevent suffering. The central focus of hospice care is the promotion of comfort and the prevention of suffering. Patient education should continue, but providing comfort is paramount. Medications should be continued unless they are not tolerated. Experimental therapies and surgeries are not commonly used in the care of hospice patients.

Nitrates (isosorbide dinitrate) & hydralazine

Combo is an alternative for HF treatment--preferred for African Americans (ACE inhibitors not always as effective) Nitrates: vasodilation Hydralazine: lowers vascular resistance & afterload Monitor: hypotension

Which of the following statements regarding heart failure is true?

Compensated congestive heart failure may be clinically asymptomatic.

Chronic HF - Collab Care 1

Correct Na and H2O retention, volume overload and preload diuretics (thiazide, loop, K sparing): - Hydrochlorothiazide (HCTZ) - it's not as potent as Lasix, inhibits Na reabsorption in the distal tubules monitor for K+ - Lasix - Spironolactone (K+ sparing) - will help retain potassium ACE inhibitors - Block angiotensin - vasodilation and will block aldosterone to lower BP FIRST LINE DRUG - Side effects - dry and tickling cough (some people can't tolerate), may switch to an ARB Can drop BP - monitor! Nitrates With high BP Reduction of cardiac workload - Beta Blockers - will block the SNS to slow down the heart - Reduces O2 consumption and workload of the heart

The nurse must achieve which of the following objectives for a patient in cardiogenic shock? Select all that apply.

Correct pulmonary edema Increase coronary perfusion Improve cardiac output Regulate blood volume

Which manifestation of left-sided heart failure can be diagnosed by examination of the lips and mucous membranes?

Cyanosis

Physiological Integrity 15. When analyzing the rhythm of a patient's electrocardiogram (ECG), the nurse will need to investigate further upon finding a(n) a. isoelectric ST segment. b. P-R interval of 0.18 second. c. Q-T interval of 0.38 second. d. QRS interval of 0.14 second.

D Because the normal QRS interval is 0.04 to 0.10 seconds, the patient's QRS interval of 0.14 seconds indicates that the conduction through the ventricular conduction system is prolonged. The P-R interval and Q-T interval are within normal range, and ST segment should be isoelectric (flat). DIF: Cognitive Level: Apply (application) REF: 791 TOP: Nursing Process: Assessment MSC:

Physiological Integrity 6. A patient has a normal cardiac rhythm and a heart rate of 72 beats/minute. The nurse determines that the P-R interval is 0.24 seconds. The most appropriate intervention by the nurse would be to a. notify the health care provider immediately. b. give atropine per agency dysrhythmia protocol. c. prepare the patient for temporary pacemaker insertion. d. document the finding and continue to monitor the patient.

D First-degree atrioventricular (AV) block is asymptomatic and requires ongoing monitoring because it may progress to more serious forms of heart block. The rate is normal, so there is no indication that atropine is needed. Immediate notification of the health care provider about an asymptomatic rhythm is not necessary. DIF: Cognitive Level: Apply (application) REF: 798 TOP: Nursing Process: Implementation MSC:

Physiological Integrity 17. Which laboratory result for a patient with multifocal premature ventricular contractions (PVCs) is most important for the nurse to communicate to the health care provider? a. Blood glucose 243 mg/dL b. Serum chloride 92 mEq/L c. Serum sodium 134 mEq/L d. Serum potassium 2.9 mEq/L

D Hypokalemia increases the risk for ventricular dysrhythmias such as PVCs, ventricular tachycardia, and ventricular fibrillation. The health care provider will need to prescribe a potassium infusion to correct this abnormality. Although the other laboratory values also are abnormal, they are not likely to be the etiology of the patient's PVCs and do not require immediate correction. DIF: Cognitive Level: Apply (application) REF: 799 OBJ: Special Questions: Prioritization TOP: Nursing Process: Assessment MSC:

Physiological Integrity 4. The nurse obtains a rhythm strip on a patient who has had a myocardial infarction and makes the following analysis: no visible P waves, P-R interval not measurable, ventricular rate 162, R-R interval regular, and QRS complex wide and distorted, QRS duration 0.18 second. The nurse interprets the patient's cardiac rhythm as a. atrial flutter. b. sinus tachycardia. c. ventricular fibrillation. d. ventricular tachycardia.

D The absence of P waves, wide QRS, rate >150 beats/minute, and the regularity of the rhythm indicate ventricular tachycardia. Atrial flutter is usually regular, has a narrow QRS configuration, and has flutter waves present representing atrial activity. Sinus tachycardia has P waves. Ventricular fibrillation is irregular and does not have a consistent QRS duration. DIF: Cognitive Level: Apply (application) REF: 794 TOP: Nursing Process: Assessment MSC:

17. Following an acute myocardial infarction, a previously healthy 67-year-old patient develops clinical manifestations of heart failure. The nurse anticipates discharge teaching will include information about a. digitalis preparations, such as digoxin (Lanoxin). b. calcium-channel blockers, such as diltiazem (Cardizem). c. -adrenergic agonists, such as dobutamine (Dobutrex). d. angiotensin-converting enzyme (ACE) inhibitors, such as captopril (Capoten).

D Rationale: ACE-inhibitor therapy is currently recommended to prevent the development of heart failure in patients who have had a myocardial infarction and as a first-line therapy for patients with chronic heart failure. Digoxin therapy for heart failure is no longer considered a first-line measure, and digoxin is added to the treatment protocol when therapy with other medications such as ACE-inhibitors, diuretics, and -adrenergic blockers is insufficient. Calcium-channel blockers are not generally used in the treatment of heart failure. The -adrenergic agonists such as dobutamine are administered through the IV route and are not used as initial therapy for heart failure. Cognitive Level: Application Text Reference: p. 832 Nursing Process: Implementation NCLEX: Physiological Integrity

23. A hospitalized patient with heart failure has a new order for captopril (Capoten) 12.5 mg PO. After administering the first dose and teaching the patient about captopril, which statement by the patient indicates that teaching has been effective? a. "I will need to include more high-potassium foods in my diet." b. "I will expect to feel more short of breath for the next few days." c. "I will be sure to take the medication after eating something." d. "I will call for help when I need to get up to the bathroom."

D Rationale: Captopril can cause hypotension, especially after the initial dose, so it is important that the patient not get up out of bed without assistance until the nurse has had a chance to evaluate the effect of the first dose. The ACE inhibitors are potassium sparring, and the nurse should not teach the patient to increase sources of dietary potassium. Increased shortness of breath is expected with initiation of -blocker therapy for heart failure, not for ACE-inhibitor therapy. ACE inhibitors are best absorbed when taken an hour before eating. Cognitive Level: Application Text Reference: p. 832 Nursing Process: Evaluation NCLEX: Physiological Integrity

13. The nurse plans discharge teaching for a patient with chronic heart failure who has prescriptions for digoxin (Lanoxin), hydrochlorothiazide (HydroDIURIL), and a potassium supplement. Appropriate instructions for the patient include a. avoid dietary sources of potassium because too much can cause digitalis toxicity. b. take the pulse rate daily and never take digoxin if the pulse is below 60 beats/min. c. take the hydrochlorothiazide before bedtime to maximize activity level during the day. d. notify the health care provider immediately if nausea or difficulty breathing occurs.

D Rationale: Difficulty breathing is an indication of acute decompensated heart failure and suggests that the medications are not achieving the desired effect. Nausea is an indication of digoxin toxicity and should be reported so that the provider can assess the patient for toxicity and adjust the digoxin dose, if necessary. Digoxin toxicity is potentiated by hypokalemia, rather than hyperkalemia. Patients should be taught to check their pulse daily before taking the digoxin and, if the pulse is less than 60, to call their provider before taking the digoxin. Diuretics should be taken early in the day to avoid sleep disruption. Cognitive Level: Application Text Reference: p. 835 Nursing Process: Implementation NCLEX: Health Promotion and Maintenance

18. A 55-year-old patient with inoperable coronary artery disease and end-stage heart failure asks the nurse whether heart transplant is a possible therapy. The nurse's response to the patient will be based on the knowledge that a. heart transplants are experimental surgeries that are not covered by most insurance. b. the patient is too old to be placed on the transplant list. c. the diagnoses and symptoms indicate that the patient is not an appropriate candidate. d. candidacy for heart transplant depends on many factors.

D Rationale: Indications for a heart transplant include inoperable coronary artery disease and refractory end-stage heart failure, but other factors such as coping skills, family support, and patient motivation to follow the rigorous post-transplant regimen are also considered. Heart transplants are not considered experimental; rather, transplantation has become the treatment of choice for patients who meet the criteria. The patient is not too old for a transplant. The patient's diagnoses and symptoms indicate that the patient may be an appropriate candidate for a heart transplant. Cognitive Level: Comprehension Text Reference: p. 837 Nursing Process: Planning NCLEX: Health Promotion and Maintenance

12. When teaching the patient with heart failure about a 2000-mg sodium diet, the nurse explains that foods to be restricted include a. eggs and other high-cholesterol foods. b. canned and frozen fruits. c. fresh or frozen vegetables. d. milk, yogurt, and other milk products.

D Rationale: Milk and yogurt naturally contain a significant amount of sodium, and intake of these should be limited for patients on a diet that limits sodium to 2000 mg daily. Other milk products, such as processed cheeses, have very high levels of sodium and are not appropriate for a 2000-mg sodium diet. The other foods listed have minimal levels of sodium and can be eaten without restriction. Cognitive Level: Application Text Reference: p. 833 Nursing Process: Implementation NCLEX: Health Promotion and Maintenance

Physiological Integrity 7. A patient who was admitted with a myocardial infarction experiences a 45-second episode of ventricular tachycardia, then converts to sinus rhythm with a heart rate of 98 beats/minute. Which of the following actions should the nurse take next? a. Immediately notify the health care provider. b. Document the rhythm and continue to monitor the patient. c. Perform synchronized cardioversion per agency dysrhythmia protocol. d. Prepare to give IV amiodarone (Cordarone) per agency dysrhythmia protocol.

D The burst of sustained ventricular tachycardia indicates that the patient has significant ventricular irritability, and antidysrhythmic medication administration is needed to prevent further episodes. The nurse should notify the health care provider after the medication is started. Defibrillation is not indicated given that the patient is currently in a sinus rhythm. Documentation and continued monitoring are not adequate responses to this situation. DIF: Cognitive Level: Apply (application) REF: 799 TOP: Nursing Process: Implementation MSC:

Physiological Integrity 22. The nurse has received change-of-shift report about the following patients on the progressive care unit. Which patient should the nurse see first? a. A patient who is in a sinus rhythm, rate 98, after having electrical cardioversion 2 hours ago b. A patient with new onset atrial fibrillation, rate 88, who has a first dose of warfarin (Coumadin) due c. A patient with second-degree atrioventricular (AV) block, type 1, rate 60, who is dizzy when ambulating d. A patient whose implantable cardioverter-defibrillator (ICD) fired two times today who has a dose of amiodarone (Cordarone) due

D The frequent firing of the ICD indicates that the patient's ventricles are very irritable, and the priority is to assess the patient and administer the amiodarone. The other patients may be seen after the amiodarone is administered. DIF: Cognitive Level: Analyze (analysis) REF: 803 OBJ: Special Questions: Prioritization; Multiple Patients TOP: Nursing Process: Implementation MSC:

Physiological Integrity 19. A patient's cardiac monitor shows sinus rhythm, rate 64. The P-R interval is 0.18 seconds at 1:00 AM, 0.22 seconds at 2:30 PM, and 0.28 seconds at 4:00 PM. Which action should the nurse take next? a. Place the transcutaneous pacemaker pads on the patient. b. Administer atropine sulfate 1 mg IV per agency dysrhythmia protocol. c. Document the patient's rhythm and assess the patient's response to the rhythm. d. Call the health care provider before giving the next dose of metoprolol (Lopressor).

D The patient has progressive first-degree atrioventricular (AV) block, and the -blocker should be held until discussing the medication with the health care provider. Documentation and assessment are appropriate but not fully adequate responses. The patient with first-degree AV block usually is asymptomatic, and a pacemaker is not indicated. Atropine is sometimes used for symptomatic bradycardia, but there is no indication that this patient is symptomatic. DIF: Cognitive Level: Apply (application) REF: 798 OBJ: Special Questions: Prioritization TOP: Nursing Process: Implementation MSC:

Physiological Integrity 20. A patient develops sinus bradycardia at a rate of 32 beats/minute, has a blood pressure (BP) of 80/42 mm Hg, and is complaining of feeling faint. Which actions should the nurse take next? a. Recheck the heart rhythm and BP in 5 minutes. b. Have the patient perform the Valsalva maneuver. c. Give the scheduled dose of diltiazem (Cardizem). d. Apply the transcutaneous pacemaker (TCP) pads.

D The patient is experiencing symptomatic bradycardia, and treatment with TCP is appropriate. Continued monitoring of the rhythm and BP is an inadequate response. Calcium channel blockers will further decrease the heart rate, and the diltiazem should be held. The Valsalva maneuver will further decrease the rate. DIF: Cognitive Level: Apply (application) REF: 804 OBJ: Special Questions: Prioritization TOP: Nursing Process: Implementation MSC:

Psychosocial Integrity 11. After providing a patient with discharge instructions on the management of a new permanent pacemaker, the nurse knows that teaching has been effective when the patient states a. "I will avoid cooking with a microwave oven or being near one in use." b. "It will be 1 month before I can take a bath or return to my usual activities." c. "I will notify the airlines when I make a reservation that I have a pacemaker." d. "I won't lift the arm on the pacemaker side up very high until I see the doctor."

D The patient is instructed to avoid lifting the arm on the pacemaker side above the shoulder to avoid displacing the pacemaker leads. The patient should notify airport security about the presence of a pacemaker before going through the metal detector, but there is no need to notify the airlines when making a reservation. Microwave oven use does not affect the pacemaker. The insertion procedure involves minor surgery that will have a short recovery period. DIF: Cognitive Level: Apply (application) REF: 805 TOP: Nursing Process: Evaluation MSC:

Physiological Integrity 2. The nurse needs to quickly estimate the heart rate for a patient with a regular heart rhythm. Which method will be best to use? a. Count the number of large squares in the R-R interval and divide by 300. b. Print a 1-minute electrocardiogram (ECG) strip and count the number of QRS complexes. c. Calculate the number of small squares between one QRS complex and the next and divide into 1500. d. Use the 3-second markers to count the number of QRS complexes in 6 seconds and multiply by 10.

D This is the quickest way to determine the ventricular rate for a patient with a regular rhythm. All the other methods are accurate, but take longer. DIF: Cognitive Level: Apply (application) REF: 789-790 TOP: Nursing Process: Assessment MSC:

The nurse is administering a dose of digoxin (Lanoxin) to a patient with heart failure (HF). The nurse would become concerned with the possibility of digitalis toxicity if the patient reported which symptom(s)? A Muscle aches B Constipation C Pounding headache D Anorexia and nausea

D Anorexia and nausea Anorexia, nausea, vomiting, blurred or yellow vision, and cardiac dysrhythmias are all signs of digitalis toxicity. The nurse would become concerned and notify the health care provider if the patient exhibited any of these symptoms.

A stable patient with acute decompensated heart failure (ADHF) suddenly becomes dyspneic. Before positioning the patient on the bedside, what should the nurse assess first? A Urine output B Heart rhythm C Breath sounds D Blood pressure

D Blood pressure The nurse should evaluate the blood pressure before dangling the patient on the bedside because the blood pressure can decrease as blood pools in the periphery and preload decreases. If the patient's blood pressure is low or marginal, the nurse should put the patient in the semi-Fowler's position and use other measures to improve gas exchange.

Beyond the first year after a heart transplant, the nurse knows that what is a major cause of death? A Infection B Acute rejection C Immunosuppression D Cardiac vasculopathy

D Cardiac vasculopathy Beyond the first year after a heart transplant, malignancy (especially lymphoma) and cardiac vasculopathy (accelerated CAD) are the major causes of death. During the first year after transplant, infection and acute rejection are the major causes of death. Immunosuppressive therapy will be used for posttransplant management to prevent rejection and increases the patient's risk of an infection.

A male patient with a long-standing history of heart failure has recently qualified for hospice care. What measure should the nurse now prioritize when providing care for this patient? A Taper the patient off his current medications. B Continue education for the patient and his family. C Pursue experimental therapies or surgical options. D Choose interventions to promote comfort and prevent suffering.

D Choose interventions to promote comfort and prevent suffering. The central focus of hospice care is the promotion of comfort and the prevention of suffering. Patient education should continue, but providing comfort is paramount. Medications should be continued unless they are not tolerated. Experimental therapies and surgeries are not commonly used in the care of hospice patients.

When considering physiological changes common in geriatric patients, the healthcare provider understands that which of these factors may increase the risk of heart failure in older patients? A Decreased sympathetic activity B Increased stroke volume C Increased myocardial contractility D Impaired diastolic filling

D Impaired diastolic filling

A patient has been admitted to the cardiac unit with a diagnosis of right ventricular failure. Which of the following assessment findings is most likely to be observed by the healthcare provider? A Bradycardia and circumoral cyanosis B Fatigue and hemoptysis C Dyspnea and pulmonary crackles D Peripheral edema and jugular vein distension

D Peripheral edema and jugular vein distension

After having an MI, the nurse notes the patient has jugular venous distention, gained weight, developed peripheral edema, and has a heart rate of 108/minute. What should the nurse suspect is happening? A ADHF B Chronic HF C Left-sided HF D Right-sided HF

D Right-sided HF An MI is a primary cause of heart failure. The jugular venous distention, weight gain, peripheral edema, and increased heart rate are manifestations of right-sided heart failure.

A patient is diagnosed with heart failure and is prescribed digoxin (Lanoxin) and furosemide (Lasix). Before administering the furosemide to the patient, which laboratory result should the healthcare provider to review? A Blood urea nitrogen (BUN) B Serum sodium C Serum troponin D Serum potassium

D Serum potassium

The nurse is administering a dose of digoxin (Lanoxin) to a patient with heart failure (HF). The nurse would become concerned with the possibility of digitalis toxicity if the patient reported which symptom(s)? A. Muscle aches B. Constipation C. Pounding headache D. Anorexia and nausea

D. Anorexia and nausea Anorexia, nausea, vomiting, blurred or yellow vision, and cardiac dysrhythmias are all signs of digitalis toxicity. The nurse would become concerned and notify the health care provider if the patient exhibited any of these symptoms.

A stable patient with acute decompensated heart failure (ADHF) suddenly becomes dyspneic. Before positioning the patient on the bedside, what should the nurse assess first? A. Urine output B. Heart rhythm C. Breath sounds D. Blood pressure

D. Blood pressure The nurse should evaluate the blood pressure before dangling the patient on the bedside because the blood pressure can decrease as blood pools in the periphery and preload decreases. If the patient's blood pressure is low or marginal, the nurse should put the patient in the semi-Fowler's position and use other measures to improve gas exchange.

Beyond the first year after a heart transplant, the nurse knows that what is a major cause of death? A. Infection B. Acute rejection C. Immunosuppression D. Cardiac vasculopathy

D. Cardiac vasculopathy Beyond the first year after a heart transplant, malignancy (especially lymphoma) and cardiac vasculopathy (accelerated CAD) are the major causes of death. During the first year after transplant, infection and acute rejection are the major causes of death. Immunosuppressive therapy will be used for posttransplant management to prevent rejection and increases the patient's risk of an infection.

A male patient with a long-standing history of heart failure has recently qualified for hospice care. What measure should the nurse now prioritize when providing care for this patient? A. Taper the patient off his current medications. B. Continue education for the patient and his family. C. Pursue experimental therapies or surgical options. D. Choose interventions to promote comfort and prevent suffering.

D. Choose interventions to promote comfort and prevent suffering. The central focus of hospice care is the promotion of comfort and the prevention of suffering. Patient education should continue, but providing comfort is paramount. Medications should be continued unless they are not tolerated. Experimental therapies and surgeries are not commonly used in the care of hospice patients.

After having an MI, the nurse notes the patient has jugular venous distention, gained weight, developed peripheral edema, and has a heart rate of 108/minute. What should the nurse suspect is happening? A. ADHF B. Chronic HF C. Left-sided HF D. Right-sided HF

D. Right-sided HF An MI is a primary cause of heart failure. The jugular venous distention, weight gain, peripheral edema, and increased heart rate are manifestations of right-sided heart failure.

The diagnosis is septic shock. The nurse knows that administration of insulin to this patient will provide which of the following beneficial effects? Select all that apply.

Decreased risk of infection Better wound healing

A client has developed constrictive pericarditis and myocardial hypertrophy. Select the most likely cause.

Diastolic dysfunction

Lanoxin

Digoxin

The client's serum digoxin level is 2.2 ng/dL and the heart rate is 120 and irregular. The nurse expects to administer which of the following drugs? Digoxin immune Fab (Digibind) Furosemide (Lasix) 60 mg I.V. Digoxin 0.5 mg bolus I.V. Potassium 40 mEq added to I.V. fluids

Digoxin immune Fab (Digibind) Rationale: Digibind binds and removes digoxin from the body and prevents toxic effects of digoxin overdose. A serum level of 2.2 is elevated, and the client is exhibiting signs of digoxin toxicity. The question does not indicate that the potassium level is low. Giving additional digoxin would exacerbate the toxicity. Giving Lasix may reduce potassium levels and contribute to increased toxicity.

A 70-year-old woman presents at a hospital after fainting at the funeral of a loved one. She is diagnosed as being in shock. Which signs and symptoms is she most likely to display? Select all that apply.

Dry, warm skin Bradycardia

Pulseless Electrical Activity

ECG shows a normal heart rhythm that does not produce a pulse

At the cellular level, cardiac muscle cells respond to an increase in ventricular volume to the point of overload by: Select all that apply.

Elongating the cardiac muscle cells Decreasing the ventricular wall thickness

A patient in asystole is likely to receive which drug treatment? Epinephrine and atropine Lidocaine and amiodarone Digoxin and procainamide β-adrenergic blockers and dopamine

Epinephrine and atropine Normally the patient in asystole cannot be successfully resuscitated. However, administration of epinephrine and atropine may prompt the return of depolarization and ventricular contraction. Lidocaine and amiodarone are used for PVCs. Digoxin and procainamide are used for ventricular rate control. β-adrenergic blockers are used to slow heart rate, and dopamine is used to increase heart rate.

The nurse will monitor which of the following respiratory manifestations of heart failure? Select all that apply.

Exertional dyspnea Orthopnea Cheyne-Stokes respiration Chronic dry cough

A patient with Class I heart failure is likely to need an immediate heart transplant.

FALSE. A patient with Class IV heart failure is likely to need an immediate heart transplant

Diastolic dysfunction occurs due to a decrease in contractility leading to decreased stroke volume.

FALSE. Diastolic dysfunction occurs due to a decrease in preload leading to decreased stroke volume.

Myocardial hypertrophy is a useful compensatory mechanism in the later stages of HF.

FALSE. Myocardial hypertrophy is a useful compensatory mechanism in the later stages of HF. It unfortunately increases the oxygen demand of the heart.

Septic shock involves inappropriate vasodilation due to a histamine response.

FALSE. Septic shock involves inappropriate vasodilation due to inflammatory cytokines released in response to gram-negative bacterial endotoxins.

The Renin-angiotensin-aldosterone mechanism results in increased preload and increased afterload, which are helpful during end stage heart failure.

FALSE. The Renin-angiotensin-aldosterone mechanism results in increased preload and increased afterload, which are NOT helpful during end stage heart failure. Not helpful! The heart can't pump against a high afterload, and the heart is already too full of fluid, so more preload will worsen the HF.

The acronym FACES is used to help educate patients to identify symptoms of heart failure. what does this acronym mean?

Fatigue Limitation of activities chest congestion/Cough edema SOB

The patient has hypokalemia, and the nurse obtains the following measurements on the rhythm strip: Heart rate of 86 with a regular rhythm; the P wave is 0.06 seconds (sec) and normal shape; the PR interval is 0.24 sec; the QRS is 0.09 sec. How should the nurse document this rhythm? First-degree AV block Second-degree AV block Premature atrial contraction (PAC) Premature ventricular contraction (PVC)

First-degree AV block In first-degree AV block there is prolonged duration of AV conduction that lengthens the PR interval above 0.20 sec. In type I second-degree AV block the PR interval continues to increase in duration until a QRS complex is blocked. In Type II the PR interval may be normal or prolonged, the ventricular rhythm may be irregular, and the QRS is usually greater than 0.12 sec. PACs cause an irregular rhythm with a different-shaped P wave than the rest of the beats, and the PR interval may be shorter or longer. PVCs cause an irregular rhythm, and the QRS complex is wide and distorted in shape.

Dopamine

For acute pain pt Excess preload Doesn't affect BP

Dobutrex (dobutamine)

For acute pain pt Excess preload Good for BP

Primacor and Inocor

For pt who are chronic Pt will get an infusion for a couple hours a couple days a week Palliative care Breath better and feel better

Natrecor

For pt who have >900 BMP Aren't going to get any better, just used to keep them alive

sinus tachycardia

HR above 100

sinus bradycardia

HR below 40

Digoxin toxicity

Halos, ringing in the ear, n/v Give digibind

Anaphylactic shock causes severe hypoxia very quickly because of which of the following reasons?

Histamine release causes massive vasodilation

IV Infusions

IV inotropes (milrinone--Primacor, dobutamine--Dobutrex) Acute decompensated HF Increase force of myocardial contraction IV vasodilators (nitropusside--Nipride, nitroglycerin, nesitiride--Natrecor)

Apresoline

Hydralazine

Lisinopril (Prinivil) is part of the treatment regimen for a client with HF. The nurse monitors the client for which electrolyte imbalance of this drug? Hyponatremia Hyperkalemia Hypokalemia Hypernatremia

Hyperkalemia Rationale: ACE inhibitors block aldosterone secretion, which results in sodium loss and potassium retention. Hyperkalemia may occur, especially when the drug is taken concurrently with potassium-sparing diuretics.

Increased cardiac workload with left heart failure can result in which of the following change to the myocardial cells?

Hypertrophy

What should the nurse recognize as an indication for the use of dopamine in the care of a patient with heart failure? Acute anxiety Hypotension and tachycardia Peripheral edema and weight gain Paroxysmal nocturnal dyspnea (PND)

Hypotension and tachycardia Dopamine is a β-adrenergic agonist whose inotropic action is used for treatment of severe heart failure accompanied by hemodynamic instability. Such a state may be indicated by tachycardia accompanied by hypotension. PND, anxiety, edema, and weight gain are common signs and symptoms of heart failure, but these do not necessarily warrant the use of dopamine.

What should the nurse recognize as an indication for the use of dopamine in the care of a patient with heart failure? Acute anxiety Hypotension and tachycardia Peripheral edema and weight gain Paroxysmal nocturnal dyspnea (PND)

Hypotension and tachycardia Dopamine is a β-adrenergic agonist whose inotropic action is used for treatment of severe heart failure accompanied by hemodynamic instability. Such a state may be indicated by tachycardia accompanied by hypotension. PND, anxiety, edema, and weight gain are common signs and symptoms of heart failure, but these do not necessarily warrant the use of dopamine.

A 20-year-old college student has a pelvic fracture and a severed leg from a motorcycle accident. She lost several units of blood. When the student arrived in the emergency department, her blood pressure was very low, her pulse was high, and her skin was pale. The nurse knows that this patient has developed which of the following types of shock?

Hypovolemic

A client has just returned from his surgical procedure. During initial vital sign measurements, the nurse notes that the client's heart rate is 111 beats/minute and the BP is 100/78 (borderline low). In this early postoperative period, the nurse should be diligently monitoring the client for the development of:

Hypovolemic shock due to acute intravascular volume loss

A 2400 mg sodium diet is prescribed for a patient with chronic heart failure, the nurse recognizes that additional teaching is necessary when the patient makes which statement?

I can eat most foods as long as I do not add salt when cooking or at the table?

which statement by the patient with chronic heart failure should cause the nurse to determine that additional discharge teaching is needed?

I should weigh myself every morning and go on a diet if I gain more than 2 or 3 pounds in 2 days.

The shortness of breath and cyanosis that occur in clients experiencing acute heart failure syndrome are primarily caused by: Select all that apply.

Impaired gas exchange Accumulation of fluid in the alveoli and airways Lung stiffness

the procedure will destroy area of the conduction system that are causing rapid heart rhythms

Important teaching for the patient scheduled for a radiofrequency catheter ablation procedure includes explaining that

Chronic HF - Collab Care 2

Improve Cardiac Contractility - cardiac glycosides - DIGOXIN - increase force of contraction and slows the heart down - improved cardiac output - kidneys work better to clean out body - monitor for DIGOXIN TOXICITY - halos around lights - brady arrythmias - anorexia and N/V **Also used for Atrail Fib (will also be on blood thinners to reduce chance of having a stroke)**

Acute HF Collab Care

Improve Cardiac Function positive inotropic drugs (dobutamine) - increases force of contraction Dopamine (raises HR) vs dobutamine (does NOT raise HR) If on dobutamine you need to be on BP lowering drugs (because dobutamine raises BP) Improve gas exchange and oxygenation Probably 100% O2 They usually vomit and can't breath well - INTUBATE (gets pulled out 24 hr later) Reducing Anxiety Explain everything you're doing to help calm the pt

The health care team is developing a plan of care for a client diagnosed with congestive heart failure (CHF). The primary treatment goal would be:

Improving quality of life by relieving symptoms

A Interdisciplinary goals for the client with heart failure are to reduce cardiac workload and improve pump effectiveness. Loss of fluid, as indicated by weight loss, reduces cardiac workload. Decrease in heart rate and reduced pulmonary congestion indicate improved pump effectiveness. The client's condition has improved and there is not a need for more aggressive treatment. However, the heart rate remains higher than normal, and there are still crackles in the lungs, indicating the need for continued treatment.

In assessing a client admitted 24 hours ago with heart failure, the nurse notes that the client has lost 2.5 pounds, heart rate is down from 105 to 88, and there are fine crackles only in the bases of the lungs. The nurse correctly interprets these data as indicating: A) The treatment regimen is achieving the desired effect. B)The client's condition is unchanged. C)A need for more aggressive treatment. D) Heart failure has resolved.

A) Normal ejection fraction is 60%; 25% ejection indicates severe dysfunction. The percentage represents the amount of blood ejected from the ventricle, not the amount retained. Cardiac output is decreased in heart failure.

In reviewing the physician's admitting notes for a client with heart failure, the nurse notes that the client has an ejection fraction of 25%. What is the appropriate interpretation of the nurse's findings? A) Ventricular function is severely impaired. B) Cardiac output is greater than normal. C) The amount of blood ejected from the ventricles is within normal limits. D) 25% of the blood in the ventricle remains after systole.

D The most accurate area on the body to assess dependent edema in a bed-ridden client is the sacral area. Sacral, or dependent, edema is secondary to right-sided heart failure.

In which of the following disorders would the nurse expect to assess sacral edema in a bedridden client? A Diabetes B Pulmonary emboli C Renal failure D Right-sided heart failure

The nurse developing a teaching plan for a client receiving thiazide diuretics should include the following. Teaching the client to take apical pulse. Decreasing potassium-rich foods in the diet. Including citrus fruits, melons, and vegetables in the diet. Teaching the client to check blood pressure t.i.d.

Including citrus fruits, melons, and vegetables in the diet. Rationale: Thiazide diuretics are potassium wasting, and levels should be closely monitored. Encouraging foods rich in potassium could help maintain potassium levels. Taking an apical pulse is indicated before administering cardiac glycosides and beta blockers. It would not be necessary to check blood pressure TID unless the client was experiencing hypotension.

An 86-year-old male client is disappointed to learn that he has class II heart failure despite a lifelong commitment to exercise and healthy eating. Which of the following age-related changes predisposes older adults to developing heart failure?

Increased vascular stiffness

A patient diagnosed with diastolic heart failure asks the nurse to explain why this has developed. The nurse knows that which of the following are often associated with diastolic failure? Select all that apply.

Ischemic heart disease Constrictive pericarditis Myocardial hypertrophy

Isordil

Isosorbide Dinitrate

Imdur

Isosorbide Mononitrate

Spironolactone

K sparing diuretic

Nitrate Teaching

Keep meds in dark area Replace every 3-6 months Check BP first Take 2-3 before calling 911 Causes GI upset

Anaphylactic shock is the most severe form of systemic allergic reaction. Immunologically medicated substances are released into the blood, causing vasodilation and an increase in capillary permeability. What physiologic response often accompanies the vascular response in anaphylaxis?

Laryngeal edema

The patient has heart failure (HF) with an ejection fraction of less than 40%. What core measures should the nurse expect to include in the plan of care for this patient (select all that apply.)? Left ventricular function is documented. Controlling dysrhythmias will eliminate HF. Prescription for digoxin (Lanoxin) at discharge Prescription for angiotensin-converting enzyme inhibitor at discharge Education materials about activity, medications, weight monitoring, and what to do if symptoms worsen

Left ventricular function is documented. -The Joint Commission has identified these three core measures for heart failure patients. Although controlling dysrhythmias will improve CO and workload, it will not eliminate HF. Prescribing digoxin for all HF patients is no longer done because there are newer effective drugs and digoxin toxicity occurs easily related to electrolyte levels and the therapeutic range must be maintained. Prescription for angiotensin-converting enzyme inhibitor at discharge -The Joint Commission has identified these three core measures for heart failure patients. Although controlling dysrhythmias will improve CO and workload, it will not eliminate HF. Prescribing digoxin for all HF patients is no longer done because there are newer effective drugs and digoxin toxicity occurs easily related to electrolyte levels and the therapeutic range must be maintained. Education materials about activity, medications, weight monitoring, and what to do if symptoms worsen -The Joint Commission has identified these three core measures for heart failure patients. Although controlling dysrhythmias will improve CO and workload, it will not eliminate HF. Prescribing digoxin for all HF patients is no longer done because there are newer effective drugs and digoxin toxicity occurs easily related to electrolyte levels and the therapeutic range must be maintained.

The patient has heart failure (HF) with an ejection fraction of less than 40%. What core measures should the nurse expect to include in the plan of care for this patient (select all that apply.)? Left ventricular function is documented. Controlling dysrhythmias will eliminate HF. Prescription for digoxin (Lanoxin) at discharge Prescription for angiotensin-converting enzyme inhibitor at discharge Education materials about activity, medications, weight monitoring, and what to do if symptoms worsen

Left ventricular function is documented. Prescription for angiotensin-converting enzyme inhibitor at discharge. Education materials about activity, medications, weight monitoring, and what to do if symptoms worsen. The Joint Commission has identified these three core measures for heart failure patients. Although controlling dysrhythmias will improve CO and workload, it will not eliminate HF. Prescribing digoxin for all HF patients is no longer done because there are newer effective drugs and digoxin toxicity occurs easily related to electrolyte levels and the therapeutic range must be maintained.

best antibiotic for pneumonia

Levoquin

Bumex (bumetanide)

Loop diuretic Cerebral edema

A 30-year-old male brought to the emergency department has the following admission data: Blood pressure 50/30, pulse 100, respiratory rate 12, temperature 101°F. The nurse does not know the exact cause but does know that patient is in shock because of which of the following readings?

Low blood pressure

The client is prescribed captopril (Capoten) for treatment of HF. The nurse teaches that the primary action of the drug is to Prevent influx of calcium. Lower peripheral resistance and reduce blood volume. Increase strength of ventricular contractions. Increase heart rate.

Lower peripheral resistance and reduce blood volume. Rationale: ACE inhibitors decrease peripheral resistance and reduce blood volume by enhancing the excretion of sodium and water. This results in decreased afterload and increased cardiac output.

Zaroxylyn

Mild diuretic Does not have a lot of K+ change

When caring for a client receiving triamterene (Dyrenium), the nurse places priority on which nursing intervention? Monitor electrolytes for hyperkalemia. Monitor electrolytes for hypernatremia. Monitor heart rate closely. Monitor blood pressure closely.

Monitor electrolytes for hyperkalemia. Rationale: Dyrenium is a potassium-sparing diuretic. The client should be monitored for signs of hyperkalemia. Hyponatremia, not hypernatremia may occur. BP and heart rate should be monitored, but are not priority with this drug.

A client is being started on lisinopril (Zestril). Nursing interventions during initial therapy with this medication must include Monitoring blood pressure. Monitoring intake and output. Monitoring EKG. Monitoring serum levels.

Monitoring blood pressure. Rationale: Lisinopril is an ACE inhibitor, which can cause severe hypotension with initial doses. The nurse should monitor the client closely for several hours.

Ventricular Tachycardia

Monomorphic P wave - not usually visible PR interval - not measurable QRS - wide and distorted

Valsartan (Diovan)

Most common ARB Given to pt who can't have ACE Used for HF and HTN Pt take huge amounts, causes a decrease in BP

Metoprolol (Lopressor)

Most common beta blocker -check BP and HR

Lasix (furosemide)

Most common loop diuretic Depletes K+

Spironolactone

Most common potassium-sparing Treatment of edema and HTN

Nitrostat

Nitroglycerin

Nitrates

Nitroglycerin -Always check BP before giving it -Decreases BP

Which of the following health problems can contribute to diastolic dysfunction heart failure?

Myocardial hypertrophy

A patient has developed cardiogenic shock. The most frequent cause of this type of shock is which of the following?

Myocardial infarction

A patient has developed left heart failure. Which of the following symptoms might have precipitated this condition?

Myocardial infarction

The patient is admitted with acute coronary syndrome (ACS). The ECG shows ST-segment depression and T-wave inversion. What should the nurse know that this indicates? Myocardia injury Myocardial ischemia Myocardial infarction A pacemaker is present.

Myocardial ischemia The ST depression and T wave inversion on the ECG of a patient diagnosed with ACS indicate myocardial ischemia from inadequate supply of blood and oxygen to the heart. Myocardial injury is identified with ST-segment elevation. Myocardial infarction is identified with ST-segment elevation and a widened and deep Q wave. A pacemaker's presence is evident on the ECG by a spike leading to depolarization and contraction.

Third degree block

No communication between the atria and ventricles; P will fire different from QRS

Natrecor

Nesiritide

A compensatory mechanism involved in HF that leads to inappropriate fluid retention and additional workload of the heart is

Neurohormonal response

Vasodilators

Nitric Oxide (fluffs alveoli) Flolan (pulmonary htn) Flolan is used with Viagra with cystic fibrosis kids -monitor for hypotension

Ventricular Fibrillation

No rate Grossly irregular rhythm P wave absent PR interval absent QRS complex irregularly shaped, rounded or pointed and markedly dissimilar fibrillation (f) waves. May be coarse >3mm or fine if <3mm. Absence of pulse is best way to distinguish from artifact.

Asystole

Nothing

A client who developed a deep vein thrombosis during a prolonged period of bed rest has deteriorated as the clot has dislodged and resulted in a pulmonary embolism. Which of the following types of shock is this client at risk of experiencing?

Obstructive shock

Paroxysmal superventricular tachycardia

P wave - abnormal shape, may be hidden in preceding T wave PR interval - Normal or shortened QRS complex - normal

Ventricular Fibrillation

P wave - absent PR interval - not measurable QRS - not measurable

Atrial Fibrillation

P wave - fibrillatory waves PR interval - not measurable QRS - normal

Atrial Flutter

P wave - flutter waves sawtooth pattern; more flutter waves than QRS complexes; occur in 2:1, 3:1, 4:1 PR interval - not measurable QRS - normal

First-Degree AV Block

P wave - normal PR interval - >.20 secs QRS - normal

is an emergency, no shocking

PEA

Second Degree AV block: Type 1

PR interval gets progressively longer until a p-wave is blocked completely cyclic lengthening no ventricular stimulation- there is a dropped beat symptoms: lightheadedness, dizziness, syncope

First degree block

PR interval greater than .20 seconds; toxicity in digoxin is most common reason

first degree AV block

PR interval is longer than 0.2 second, p wave is separated from QRS complex

Hypokalemia can cause these heart rhythms

PVCs, bradycardia, blocks, ventricular tachycardia, inverted T waves and ST depression

Hyperkalemia can cause these heart rhythms

PVCs, vfib, peaked T waves, widened QRS

A client with heart failure tells the nurse that he is frustrated and is unable to get "a good night's rest." The client relates that he falls asleep and is suddenly awakened and feels as though he is having a hard time breathing and is suffocating. The nurse recognizes this assessment as:

Paroxysmal nocturnal dyspnea

On a holiday trip home, the nurse's mother states that the nurse's father was diagnosed with right heart failure. Which of the following manifestations exhibited by the father does the nurse know might have preceded this diagnosis?

Peripheral edema weight gain

Milrinone

Phosphodiesterase inhibitor

Torsades de Pointes

Polymorphic VT P wave - not usually visible PR interval - not measurable QRS - wide and distorted, alternates size and shape, prolonged QT interbal

when caring for the patient with heart failure is treated with hydrochlorothiazide, digoxin and lisinopril. To prevent the risk of digitalis toxicity with these drugs, what is most important that the nurse monitor for this patient?

Potassium levles

PAC- premature atrial beat

Pre-cursor to A Fib; Cardioversion: controlled; shock to regulate the rate; Cardizem

A patient has sought care following a syncopal episode of unknown etiology. Which nursing action should the nurse prioritize in the patient's subsequent diagnostic workup? Preparing to assist with a head-up tilt-test Preparing an IV dose of a β-adrenergic blocker Assessing the patient's knowledge of pacemakers Teaching the patient about the role of antiplatelet aggregators

Preparing to assist with a head-up tilt-test In patients without structural heart disease, the head-up tilt-test is a common component of the diagnostic workup following episodes of syncope. IV β-blockers are not indicated although an IV infusion of low-dose isoproterenol may be started in an attempt to provoke a response if the head-up tilt-test did not have a response. Addressing pacemakers is premature and inappropriate at this stage of diagnosis. Patient teaching surrounding antiplatelet aggregators is not directly relevant to the patient's syncope at this time.

Heart failure can have which of the following consequences? Select all that apply.

Pulmonary congestion Systemic congestion Low cardiac output

wenckebachs phenomena

QRS complex goes missing , or spaced out.

Atrial flutter

Rate and rhythm- Atrial 200-350 bpm and regular, Ventricular> or < 100 bpm regular or irregular. P Wave- Flutter (F) waves (sawtooth pattern) more flutter waves than QRS complexes; may occur in a 2:1, 3:1, 4:1 pattern. PR interval- Not measurable QRS complex- Normal usually

Atrial fibrillation

Rate and rhythm- Atrial 350-600 bpm and regular. P Wave- Fibrillatory (f) waves PR interval- not measurable QRS complex- Normal (usually)

Third-degree AV block

Rate and rhythm- Atrial: Regular but may appear irregular due to P waves hidden in QRS complexes, Ventricular: 20-60 beats/min and regular. P Wave- Normal, but no connection with QRS complex. PR interval- Variable. QRS complex- Normal or widened, no relationship with P waves.

Second-degree AV block Type I

Rate and rhythm- Atrial: normal and regular, Ventricular: slower and regular. P Wave- Normal PR interval-Progressive lengthening QRS complex- Normal QRS width, with pattern of one nonconducted (blocked) QRS complex

Second-degree AV block Type II

Rate and rhythm- Atrial: usually normal and regular, Ventricular: slower and regular or irregular. P Wave- More P waves than QRS complexes. PR interval- Normal or prolonged QRS complex- Widened QRS, preceded by >= P waves, with nonconducted (blocked) QRS complex.

First-degree AV block

Rate and rhythm- Normal and regular P Wave- Normal PR interval- >0.20 sec QRS complex- Normal

Ventricular fibrillation

Rate and rhythm- Not measurable and irregular. P Wave- Absent. PR interval- Not measurable. QRS complex- Not measurable.

Premature ventricular contraction

Rate and rhythm- Underlying rhythm can be any rate, regular or irregular rhythm, PVCs occur at variable rates. P Wave- Not usually visible, hidden in the PVC. PR interval- Not measurable QRS complex- Wide and distorted.

Premature atrial contraction

Rate and rhythm- Usually 60-100 bpm and irregular. P Wave- Abnormal shape PR interval- Normal QRS complex- Normal (usually)

A nurse preceptor is evaluating the skills of a new registered nurse (RN) caring for clients experiencing shock. Which action by the new RN indicates a need for more education?

Raising the head of the bed to a high Fowler's position

Severe stenosis of the pulmonary semilunar valve can cause right-sided HF.

TRUE

Premature Ventricular Contraction (PVC)

Rate of underlying rhythm Irregular rhythm when early PVCs interrupt underlying rhythm P wave absent or appearing as notches after QRS in ST segment or T wave. Compensatory pause usually present. PR interval absent QRS complex > 0.12; wide with bizarre shape; determine if PVCs are identical they are called uniform; PVCs with different shape are called multiform

Which ECG characteristic is consistent with a diagnosis of ventricular tachycardia (VT)? Unmeasurable rate and rhythm Rate 150 beats/min; inverted P wave Rate 200 beats/min; P wave not visible Rate 125 beats/min; normal QRS complex

Rate 200 beats/min; P wave not visible VT is associated with a rate of 150 to 250 beats/min; the P wave is not normally visible. Rate and rhythm are not measurable in ventricular fibrillation. P wave inversion and a normal QRS complex are not associated with VT.

Sinus Tachycardia

Rate and rhythm- 101-200 bpm and regular P Wave- Normal PR interval- Normal QRS complex- Normal

Paroxysmal supraventricular tachycardia

Rate and rhythm- 150-220 bpm regular P Wave- Abnormal shape, may be hidden in the preceding T wave. PR interval- Normal or shortened QRS complex- Normal (usually)

Ventricular tachycardia

Rate and rhythm- 150-250 bpm and regular or irregular. P Wave- Not usually visible. PR interval- Not measurable QRS complex- Wide and distorted.

Accelerated idioventricular rhythm

Rate and rhythm- 40-100 bpm and regular. P Wave- not usually visible. PR interval- Not measurable. QRS complex- Wide and distorted.

Junctional dysrhythmias

Rate and rhythm- 40-180 bpm and regular P Wave- Inverted, may be hidden in QRS complex. PR interval- Variable QRS complex- Normal (usually)

Normal Sinus Rhythm

Rate and rhythm- 60-100 bpm P Wave- Normal PR interval- Normal QRS Complex- Normal

Sinus Bradycardia

Rate and rhythm- <60 bpm regular P Wave- Normal PR interval- Normal QRS complex- Normal

The pulmonary capillary wedge pressure represents the left ventricular pressures.

TRUE

You might see hyperglycemia and DIC in septic shock.

TRUE

The nurse knows that the basic pathophysiology of heart failure is best described as which of the following?

Reduced ventricular efficiency

A patient with a recent diagnosis of heart failure has been prescribed furosemide. What outcome does the nurse anticipate will occur that demonstrates medication effectiveness? Promote vasodilation. Reduction of preload. Decrease in afterload. Increase in contractility.

Reduction of preload. Diuretics such as furosemide are used in the treatment of heart failure to mobilize edematous fluid, reduce pulmonary venous pressure, and reduce preload. They do not directly influence afterload, contractility, or vessel tone.

A patient with a recent diagnosis of heart failure has been prescribed furosemide. What outcome does the nurse anticipate will occur that demonstrates medication effectiveness? Promote vasodilation. Reduction of preload. Decrease in afterload. Increase in contractility.

Reduction of preload. Diuretics such as furosemide are used in the treatment of heart failure to mobilize edematous fluid, reduce pulmonary venous pressure, and reduce preload. They do not directly influence afterload, contractility, or vessel tone.

Diuretics

Remove excess extracellular fluid Different types differ in site of action All should be admin. in the morning to avoid disruption of sleep cycle Need may be avoided by diet & fluid intake Low Na diet (<2 g/day) & fluid intake (<2 qt/day)

After having a myocardial infarction (MI), the nurse notes the patient has jugular venous distention, gained weight, developed peripheral edema, and has a heart rate of 108 beats/min. What should the nurse suspect is happening? ADHF Chronic HF Left-sided HF Right-sided HF

Right-sided HF An MI is a primary cause of heart failure. The jugular venous distention, weight gain, peripheral edema, and increased heart rate are manifestations of right-sided heart failure.

After having a myocardial infarction (MI), the nurse notes the patient has jugular venous distention, gained weight, developed peripheral edema, and has a heart rate of 108 beats/min. What should the nurse suspect is happening? ADHF Chronic HF Left-sided HF Right-sided HF

Right-sided HF An MI is a primary cause of heart failure. The jugular venous distention, weight gain, peripheral edema, and increased heart rate are manifestations of right-sided heart failure.

Assessment of an elderly female client reveals the presence of bilateral pitting edema of the client's feet and ankles and pedal pulses that are difficult to palpate. Auscultation of the client's lungs reveals clear air entry to bases, and the client's oxygen saturation level is 93%, and vital signs are within reference ranges. What is this client's most likely health problem?

Right-sided heart failure

SVT

SA node uncontrolled firing, HR greater than 150, pwave present

CO = __________ x _________

SV x HR

A 20-year-old college student being treated for a kidney infection developed a temperature of 104ºF in spite of treatment with antibiotics. Her pulse was high, her blood pressure was low, and her skin was hot, dry, and flushed. The nurse knows that this patient most likely is experiencing which of the following types of shock?

Septic

The nurse prepares to administer digoxin 0.125 mg to a patient admitted with influenza and a history of chronic heart failure. What should the nurse assess before giving the medication? Prothrombin time Urine specific gravity Serum potassium level Hemoglobin and hematocrit

Serum potassium level Serum potassium should be monitored because hypokalemia increases the risk for digoxin toxicity. Changes in prothrombin time, urine specific gravity, and hemoglobin or hematocrit would not require holding the digoxin dose.

The nurse prepares to administer digoxin 0.125 mg to a patient admitted with influenza and a history of chronic heart failure. What should the nurse assess before giving the medication? Prothrombin time Urine specific gravity Serum potassium level Hemoglobin and hematocrit

Serum potassium level Serum potassium should be monitored because hypokalemia increases the risk for digoxin toxicity. Changes in prothrombin time, urine specific gravity, and hemoglobin or hematocrit would not require holding the digoxin dose.

The nurse reviews lab studies of a client receiving digoxin (Lanoxin). Intervention by the nurse is required if the results include a Serum sodium level of 140 mEq/L. Serum digoxin level of 1.2 ng/dL. Serum potassium level of 3.0 mEq/L. Hemoglobin 14.4 g/dL.

Serum potassium level of 3.0 mEq/L. Rationale: Normal serum potassium level is 3.5-5.0 mEq/L. Hypokalemia may predispose the client to digitalis toxicity. The other lab values are WNL.

The home care nurse visits a patient with chronic heart failure. Which clinical manifestations, assessed by the nurse, would indicate acute decompensated heart failure (pulmonary edema)? Fatigue, orthopnea, and dependent edema Severe dyspnea and blood-streaked, frothy sputum Temperature is 100.4oF and pulse is 102 beats/min Respirations 26 breaths/min despite oxygen by nasal cannula

Severe dyspnea and blood-streaked, frothy sputum Clinical manifestations of pulmonary edema include anxiety, pallor, cyanosis, clammy and cold skin, severe dyspnea, use of accessory muscles of respiration, a respiratory rate greater than 30 breaths/min, orthopnea, wheezing, and coughing with the production of frothy, blood-tinged sputum. Auscultation of the lungs may reveal crackles, wheezes, and rhonchi throughout the lungs. The heart rate is rapid, and blood pressure may be elevated or decreased.

The home care nurse visits a patient with chronic heart failure. Which clinical manifestations, assessed by the nurse, would indicate acute decompensated heart failure (pulmonary edema)? Fatigue, orthopnea, and dependent edema Severe dyspnea and blood-streaked, frothy sputum Temperature is 100.4oF and pulse is 102 beats/min Respirations 26 breaths/min despite oxygen by nasal cannula

Severe dyspnea and blood-streaked, frothy sputum (Khó thở nặng và có đốm máu, đờm bọt) Clinical manifestations of pulmonary edema include anxiety, pallor, cyanosis, clammy and cold skin, severe dyspnea, use of accessory muscles of respiration, a respiratory rate greater than 30 breaths/min, orthopnea, wheezing, and coughing with the production of frothy, blood-tinged sputum. Auscultation of the lungs may reveal crackles, wheezes, and rhonchi throughout the lungs. The heart rate is rapid, and blood pressure may be elevated or decreased.

Which of the following does the nurse know can lead to right-sided heart failure?

Severe pneumonia

An important factor in the mortality of severe shock is acute renal failure. What impacts the extent of renal damage in shock?

Severity and duration of shock

Sally's height is 70 inches, and her weight is 145 lb. Her resting HR is 110 bpm and her SV is 35 mL/beat. Which of the following is NOT true about Sally? Her EF is probably low (but you can't calculate it with this information) She is bradycardic Her CO is low, which will result in a low BP and tissue hypoxia Her CO is 3.85 L/min

She is bradycardic

The client is prescribed a beta-blocker as adjunct therapy to treatment of heart failure. The nurse recognizes that beta blockers act by Increasing contractility and cardiac output. Decreasing preload. Slowing the heart and decreasing afterload. Decreasing peripheral resistance

Slowing the heart and decreasing afterload. Rationale: Beta-blockers improve symptoms of HF by slowing heart rate and decreasing blood pressure. The decreased afterload causes decreased workload on the heart.

Corticosteroids

Solumedrol/Solucortef Prednisone -decreases agitation and calms cilia -makes pt gain weight, agitated, immunocompromised -make sure pt doesn't develop thrush

The nurse reviews the following vital signs recorded by an unlicensed assistive personnel (UAP) on a patient with acute decompensated heart failure: BP 98/60, HR 102 bpm, Temp 98.2 F, SpO2 84 % on 2 L/min via nasal cannula. what is the highest priority? what should the nurse due next?

SpO2 Place patient in high fowlers

Aldosterone antagonists (K-sparing diuretics)

Spironolactone (Aldactone) Blocks aldosterone in distal tubule & collecting duct Reduces mortality & morbidity in HF Monitor: hyperkalemia, hyponatremia

Digoxin (Lanoxin)

Strengthens ventricular pump Causes BP to drop, hold <60 Give IV if HR is really high Labs drawn q3months

What is the primary cause of heart failure in infants and children?

Structural heart defects

The patient has atrial fibrillation with a rapid ventricular response. The nurse knows to prepare for which treatment if an electrical treatment is planned for this patient? Defibrillation Synchronized cardioversion Automatic external defibrillator (AED) Implantable cardioverter-defibrillator (ICD)

Synchronized cardioversion Synchronized cardioversion is planned for a patient with supraventricular tachydysrhythmias (atrial fibrillation with a rapid ventricular response). Defibrillation or AEDs are the treatment of choice to end ventricular fibrillation and pulseless ventricular tachycardia (VT). An ICD is used with patients who have survived sudden cardiac death (SCD), have spontaneous sustained VT, and are at high risk for future life-threatening dysrhythmias.

A common finding in all types of circulatory shock is tissue hypoxia.

TRUE

An increased serum lactate level (lactic acidosis) is commonly used as a marker of anaerobic metabolism.

TRUE

Heart failure patients are at risk for atrial fibrillation.

TRUE

In systolic dysfunction, there is a decreased ejection fraction.

TRUE

Left-sided HF may lead to right-sided HF owing to increased right ventricular afterload.

TRUE

Patients with right-sided HF may present with jugular venous distention and peripheral edema.

TRUE

Severe HTN (which increases afterload) is a primary cause of left-sided heart failure.

TRUE

The patient with chronic heart failure is being discharged from the hospital. What information should the nurse emphasize in the patient's discharge teaching to prevent progression of the disease to acute decompensated heart failure (ADHF)? Take medications as prescribed. Use oxygen when feeling short of breath. Only ask the physician's office questions. Encourage most activity in the morning when rested.

Take medications as prescribed. The goal for the patient with chronic HF is to avoid exacerbations and hospitalization. Taking the medications as prescribed along with nondrug therapies such as alternating activity with rest will help the patient meet this goal. If the patient needs to use oxygen at home, it will probably be used all the time or with activity to prevent respiratory acidosis. Many HF patients are monitored by a care manager or in a transitional program to assess the patient for medication effectiveness and monitor for patient deterioration and encourage the patient. This nurse manager can be asked questions or can contact the health care provider if there is evidence of worsening HF.

The patient with chronic heart failure is being discharged from the hospital. What information should the nurse emphasize in the patient's discharge teaching to prevent progression of the disease to acute decompensated heart failure (ADHF)? Take medications as prescribed. Use oxygen when feeling short of breath. Only ask the physician's office questions. Encourage most activity in the morning when rested.

Take medications as prescribed. The goal for the patient with chronic HF is to avoid exacerbations and hospitalization. Taking the medications as prescribed along with nondrug therapies such as alternating activity with rest will help the patient meet this goal. If the patient needs to use oxygen at home, it will probably be used all the time or with activity to prevent respiratory acidosis. Many HF patients are monitored by a care manager or in a transitional program to assess the patient for medication effectiveness and monitor for patient deterioration and encourage the patient. This nurse manager can be asked questions or can contact the health care provider if there is evidence of worsening HF.

assessing the patient response to the dysrhythmia

The ECG monitor of a patient in the cardiac care unit after an MI indicates ventricular bigeminy with a rate of 50 beats/min. The nurse would anticipate?

Sinus rhythm with a depressed ST segment

The nurse is monitoring the ECG of a patient admitted with ACS, which ECG characteristics would be most suggestive of Myocardial ischemia

The health care provider is reviewing lab results of a client diagnosed with heart failure. The provider notes that the client's ANP and BNP levels have been increasing and remain significantly elevated. These results would be interpreted as:

The condition is getting progressively worse

A, B, C, D, F (HDL is good)

The healthcare provider is teaching a group of senior citizens about risk factors for heart failure. Which of these factors will the healthcare provider include in the teaching? Select all that apply. A) High sodium intake B) History of preeclampsia C) Obesity D) Sleep apnea E) Increased high density lipoproteins (HDL) F) Hypertension

A) Stopping smoking can decrease the risk of CAD by 50%. While obesity, diet, and, stress impact the risk for CAD, stopping smoking has the greatest impact.

The nurse determines that teaching has been effective when a client with coronary artery disease (CAD) identifies which priority modifiable risk factor? A) Smoking B) Obesity C) Stress D) Diet

A, E The client's edema is unlikely to be due to fluid retention if daily weights have been stable, so the nurse's interventions are aimed at promoting venous return to the heart by having the client elevate the legs and applying antiembolism stockings. While reviewing the diet and lab values is appropriate, it is unlikely the client is experiencing fluid retention if daily weights are not increasing. There is no need to increase the client's diuretic dosage.

The nurse in a long-term care facility is talking with the family of a client diagnosed with heart failure, diabetes, hypertension, and chronic renal failure. The nurse notes mild edema of the ankles while the client is sitting in the chair. Breath sounds are clear, equal, and with good chest excursion, and the client denies any feeling of shortness of breath. The nurse reviews the medical record and sees no significant change in the client's daily weights over the last week. What are the nurse's priority interventions for this client? (Select all that apply.) A) Encourage the client to elevate feet when sitting. B) Review the client's BUN and creatinine. C) Call the doctor for an order to increase the client's diuretic. D) Review the client's diet to determine sodium intake. E) Apply antiembolism stockings.

B Rationale: Stable angina is predictable and is associated with increased activity, and is relieved by rest and nitrates. ECG changes, nocturnal pain, and weak peripheral pulses are not findings associated with stable angina.

The nurse in the clinic assesses a client with stable angina. What expectations does the nurse have for this client? A) Increasing nocturnal pain B) Correlation between activity level and pain C) Weak peripheral pulses D) Persistent ECG changes

A) Peptic ulcer disease does not exacerbate heart failure. Respiratory infections, nutritional anemia, atrial fibrillation, stress, pregnancy, and thyroid disorders are among disorders and conditions that will exacerbate heart failure.

The nurse is examining a client in the clinic for follow-up care for heart failure. Which factor, if reported by the client, would not be associated with exacerbating heart failure? A) Peptic ulcer disease B) Nutritional anemia C) Recent upper respiratory infection D) Atrial fibrillation

Patients should be sedated if cardioversion is done on a non emergency basis

The nurse prepares a patient for synchronized cardioverion knowing that cardioversion differs from defibrillation in that

B, C, E Rationale: The client undergoing angiography has a large-bore catheter inserted through the femoral artery, so the priority of care is to monitor and prevent bleeding. The client will lie flat for several hours and the groin will be checked regularly. A sandbag may be placed to maintain constant pressure on the arterial puncture site. Elevating the foot of the bed would increase pressure and blood flow to the groin and increase the risk of bleeding, as would applying heat to the leg.

The nurse receives an 82-year-old client with a history of A-fib from the angiography department. What priority actions will the nurse do next? (Select all that apply.) A) Elevate the foot of the bed or place pillows under the legs. B) Check the groin for bleeding or hematoma. C) Place the client supine for several hours. D) Apply heat to the calf of the leg. E) Place a sandbag on the femoral site.

tachydysrhythmias

The use of catheter ablation therapy to "burn" areas of the cardiac conduction system is indicated for the treatment of

The nurse assess the patient with chronic biventricular heart failure for paroxysmal nocturnal dyspnea (PND) by questioning the patient regarding?

The use of two or more pillows to help breathing during sleep.

Methylxanthines

Theophylline (level: 5-15) decreases inflammation, use with smokers Side effects: increases heart rate (30-40 beats)

Heart failure and circulatory shock are both conditions of circulatory system failure. Which of the following statements regarding these conditions is correct?

They have the same compensatory mechanisms

Heart failure and circulatory shock are both conditions of circulatory system failure. Which of the following statements regarding these conditions is correct?

They have the same compensatory mechanisms.

Hydrochlorothiazide, metolozone

Thiazide and thiazide like duiretics

The nurse is seeing artifact on the telemetry monitor. Which factors could contribute to this artifact? Disabled automaticity Electrodes in the wrong lead Too much hair under the electrodes Stimulation of the vagus nerve fibers

Too much hair under the electrodes Artifact is caused by muscle activity, electrical interference, or insecure leads and electrodes that could be caused by excessive chest wall hair. Disabled automaticity would cause an atrial dysrhythmia. Electrodes in the wrong lead will measure electricity in a different plane of the heart and may have a different wave form than expected. Stimulation of the vagus nerve fibers causes a decrease in heart rate, not artifact.

Anti-cholinergics

Uses: COPD Atrovent Spiriva Side effects: dry mouth*, stomatitis, dizziness -cannot use with peanut allergy

Beta agonists

Uses: COPD, asthma, chronic gas pts Proventil - short-acting Xopenox - kids with cystic fibrosis; breathing treatment Serevent - long-acting, most common Side effects: tachycardia*, shakes, tremors *make sure pt is better after treatment*

A patient is told that she has cardiac valve leaflets, or cusps, that are floppy and fail to shut completely, permitting blood flow even when the valve should be completely closed. The nurse knows that this condition can lead to heart failure and is referred to as which of the following?

Valvular regurgitation

An elderly client asks the nurse why so many older people develop heart failure. The best response would be increased:

Vascular stiffness

Levels of endothelins may be increased in clients with heart failure. Which of the following is the primary action of endothelins?

Vasoconstriction

A client who has progressed to late-stage heart failure is experiencing sympathetic stimulation. The nurse would assess the client for: Select all that apply.

Vasoconstriction Arrhythmias Increased oxygen demand

In hypovolemic shock, the main purpose of treatment is correcting or controlling the underlying cause of the hypovolemia and improving the perfusion of the tissues and organs of the body. Which of the following treatments is not a primary form of therapy for hypovolemic shock?

Vasoconstrictive drugs

What describes the action of the natrieuretic peptides and nitric oxide in thier counterregulatory processes in response to heart failure?

Vasodilation and decreased blood pressure

Reduce Afterload

Vasodilator (IV Nitroprusside) Nitroprusside - IV continuous - arterial vasodilator They HAVE to have a high BP to be on this Morphine Help with the breathlessness Helps with respiratory problems

Hydralazine , Isosorbide dinitrate

Vasodilators

For which dysrhythmia is defibrillation primarily indicated? Ventricular fibrillation Third-degree AV block Uncontrolled atrial fibrillation Ventricular tachycardia with a pulse

Ventricular fibrillation Defibrillation is always indicated in the treatment of ventricular fibrillation. Drug treatments are normally used in the treatment of uncontrolled atrial fibrillation and for ventricular tachycardia with a pulse (if the patient is stable). Otherwise, synchronized cardioversion is used (as long as the patient has a pulse). Pacemakers are the treatment of choice for third-degree heart block.

Calcium channel blockers

Verapamil, Pricardia, Diltiazem Used for very high HR and BP Pt go into heart block Get a headache

While teaching a client with new-onset right-sided heart failure, the nurse should educate the client to monitor for fluid accumulation by:

Weighing every day at the same time with same type of clothing

A patient is diagnosed with right-sided heart failure. The nurse knows that a frequent sign of this type of failure is peripheral edema, evidenced by which of the following?

Weight gain

Lead placement for V1 includes one lead each for right arm, right leg, left arm, and left leg with the 5th lead on the 4th intercoastal space of the right of the sternal border.

What accurately describes ECG monitoring?

automaticity

What describes the SA node ability to discharge an electrical impulses spontaneously

applying gel pads to the patient chest

What is included in the nurse's responsibilities in preparing to administer defibrillation?

B Left-sided failure results in backflow of blood from the pulmonary system resulting in pulmonary edema and shortness of breath. Edema of the feet and ankles and liver enlargement would be seen in clients with right-sided failure, due to backup of blood return from the body. Abdominal distention is not usually a symptom of heart failure.

When assessing a client diagnosed with left-sided heart failure, the nurse anticipates which finding? A) Abdominal distention B) Shortness of breath C) Liver enlargement D) Edema of the feet and ankles

A) Pulmonary Edema is a manifestation of left sided heart failure

When assessing a patient with chronic heart failure, the healthcare provider would expect to identify which of these clinical manifestations? A) Inspiratory crackles B) Asymmetrical Chest Expansion C) Expiratory Wheezing D) Subcutaneous crepitus

D) ("Generally, left heart failure causes you to be short of breath (breathlessness). This is generally worse when you exert yourself (for example, walking up hill) or when you are lying flat in bed")

When obtaining a health history of a patient admitted with a diagnosis of heart failure, which statement made by the patient supports the diagnosis of heart failure? A) "I often feel pain in my lower legs when I take my walk." B) "I sometimes feel pain in the middle of my chest during exercise." C) "I get hot and break out in a sweat during the night." D) "I get out of breath when I go up a flight of stairs

C Coronary artery thrombosis causes an inclusion of the artery, leading to myocardial death. An aneurysm is an outpouching of a vessel and doesn't cause an MI. Renal failure can be associated with MI but isn't a direct cause. Heart failure is usually a result from an MI.

Which of the following conditions is most commonly responsible for myocardial infarction? A Aneurysm B Heart failure C Coronary artery thrombosis D Renal failure

C Inadequate deactivation of aldosterone by the liver after right-sided heart failure leads to fluid retention, which causes oliguria.

Which of the following symptoms might a client with right-sided heart failure exhibit? A Adequate urine output B Polyuria C Oliguria D Polydipsia

Begin CPR

a patient on the cardiac telemetry unit goes into ventricular fibrillation and is unresponsive. Following initiation of the Emergency call system (code blue), what is the next priority for the nurse in caring for this patient?

call the cardiologist

While providing discharge instructions to the patient who has had an implantable cardioverter-defibrillator inserted, the nurse teaches the patient that if the ICD fires,he or she should do what?

The nurse is preparing to administer digoxin to a patient with heart failure. In preparation, laboratory results are reviewed with the following findings: sodium 139 mEq/L, potassium 5.6 mEq/L, chloride 103 mEq/L, and glucose 106 mg/dL. What is the priority action by the nurse? Withhold the daily dose until the following day. Withhold the dose and report the potassium level. Give the digoxin with a salty snack, such as crackers. Give the digoxin with extra fluids to dilute the sodium level.

Withhold the dose and report the potassium level. The normal potassium level is 3.5 to 5.0 mEq/L. The patient is hyperkalemic, which makes the patient more prone to digoxin toxicity. For this reason, the nurse should withhold the dose and wait for the potassium level to normalize. The physician may order the digoxin to be given once the potassium level has been treated and decreases to within normal range.

The nurse is preparing to administer digoxin to a patient with heart failure. In preparation, laboratory results are reviewed with the following findings: sodium 139 mEq/L, potassium 5.6 mEq/L, chloride 103 mEq/L, and glucose 106 mg/dL. What is the priority action by the nurse? Withhold the daily dose until the following day. Withhold the dose and report the potassium level. Give the digoxin with a salty snack, such as crackers. Give the digoxin with extra fluids to dilute the sodium level.

Withhold the dose and report the potassium level. The normal potassium level is 3.5 to 5.0 mEq/L. The patient is hyperkalemic, which makes the patient more prone to digoxin toxicity. For this reason, the nurse should withhold the dose and wait for the potassium level to normalize. The physician may order the digoxin to be given once the potassium level has been treated and decreases to within normal range.

A patient with newly discovered high BP has an average reading of 158/98 mm Hg after 3 months of exercise and diet modifications. Which management strategy will be a priority for this patient? a. Medication will be required because the BP is still not at goal. b. BP monitoring should continue for another 3 months to confirm a diagnosis of hypertension. c. Lifestyle changes are less important, since they were not effective, and medications will be started. d. More vigorous changes in the patient's lifestyle are needed for a longer time before starting medications.

a

The nurse is providing care for a patient who has decreased cardiac output related to heart failure. What should the nurse recognize about cardiac output? a. Calculated by multiplying the patient's stroke volume by the heart rate b. The average amount of blood ejected during one complete cardiac cycle c. Determined by measuring the electrical activity of the heart and the patient's heart rate d. The patient's average resting heart rate multiplied by the patient's mean arterial blood pressure

a

A patient with chronic HF and atrial fibrillation is treated with a digitalis glycoside and a loop diuretic. To prevent possible complications of this combination of drugs, what does the nurse need to do (select all that apply)? a. Monitor serum potassium levels b. teach the patient how to take a pulse rate. c. keep an accurate measure of intake and output d. Teach the patient about dietary restriction or potassium e. Withhold digitalis and notify health care provider if heart rate is irregular

a & b Rationale: Hypokalemia, which can be caused by the use of potassium-depleting diuretics (e.g., thiazides, loop diuretics), is one of the most common causes of digitalis toxicity. Low serum levels of potassium enhance the action of digitalis, causing a therapeutic dose to achieve toxic levels. Hypokalemia can also precipitate dysrhythmias. Monitoring the serum potassium levels of patients receiving digitalis preparations and potassium-depleting diuretics is essential. Patients taking digitalis preparations should be taught how to measure their pulse rate because bradycardia and atrioventricular blocks are late signs of digitalis toxicity. In addition, patients should know what pulse rate would necessitate a call to the health care provider.

marked bradycardia and hypotension

a 54 year old patient who has no structural heart disease has an episode of syncope. An upright tilt table test is performed to rule out neurocardiogenic syncope. The nurse explains to the patient that is neurocardiogenic syncope is the problem, the patient will experience?

The health care provider prescribes sprinolactone for the patient with chronic heart failure, what diet modifications related to the use of this drug should the nurse include in the patient teaching

a Decrease in both sodium and potassium

Despite a high dosage, a male patient who is taking nifedipine (Procardia XL) for antihypertensive therapy continues to have blood pressures over 140/90 mmHg. What should the nurse do next? a) Assess his adherence to therapy. b) Ask him to make an exercise plan. c) Instruct him to use the DASH diet. d) Request a prescription for a thiazide diuretic.

a) Assess his adherence to therapy. A long-acting calcium-channel blocker such as nifedipine causes vascular smooth muscle relaxation resulting in decreased SVR and arterial BP and related side effects. The patient data the nurse has about this patient is very limited, so the nurse needs to assess his adherence to therapy.

The nurse teaches a patient with hypertension that uncontrolled hypertension may damage organs in the body primarily by which mechanism? a) Hypertension promotes atherosclerosis and damage to the walls of the arteries. b) Hypertension causes direct pressure on organs, resulting in necrosis and replacement of cells with scar tissue. c) Hypertension causes thickening of the capillary membranes, leading to hypoxia of organ systems. d) Hypertension increases blood viscosity, which contributes to intravascular coagulation and tissue necrosis distal to occlusions.

a) Hypertension promotes atherosclerosis and damage to the walls of the arteries. Hypertension is a major risk factor for the development of atherosclerosis by mechanisms not yet fully known. However, once atherosclerosis develops, it damages the walls of arteries and reduces circulation to target organs and tissues.

The nurse is caring for a patient with hypertension who is scheduled to receive a dose of esmolol (Brevibloc). The nurse should withhold the dose and consult the prescribing physician for which vital sign taken just before administration? a) Pulse 48 b) Respirations 24 c) Blood pressure 118/74 d) Oxygen saturation 93%

a) Pulse 48 Because esmolol is a β1-adrenergic blocking agent, it can cause hypotension and bradycardia as adverse effects. The nurse should withhold the dose and consult with the health care provider for parameters regarding pulse rate limits.

A patient admitted with heart failure appears very anxious and complains of shortness of breath. Which nursing actions would be appropriate to alleviate this patient's anxiety (select all that apply)? a. Administer ordered morphine sulfate. b. Position patient in a semi-Fowler's position. c. Position patient on left side with head of bed flat. d. Instruct patient on the use of relaxation techniques. e. Use a calm, reassuring approach while talking to patient.

a, b, d, & e Morphine sulfate reduces anxiety and may assist in reducing dyspnea. The patient should be positioned in semi-Fowler's position to improve ventilation that will reduce anxiety. Relaxation techniques and a calm reassuring approach will also serve to reduce anxiety.

Which statements accurately describe heart failure (HF) (select all that apply)? a. A common cause of diastolic failure is left ventricular dysfunction. b. A primary risk factor for heart failure is coronary artery disease (CAD). c. Systolic heart failure results in a normal left ventricular ejection fraction. d. Systolic failure is characterized by abnormal resistance to ventricular filling. e. Hypervolemia precipitates heart failure by decreasing cardiac output and increasing oxygen consumption.

a, b. Diastolic failure is characterized by abnormal resistance to ventricular filling. Hypertension, coronary artery disease (CAD), advanced age, and diabetes are all risk factors for heart failure (HF). Ejection fraction (EF) is decreased in systolic HF. Decreased cardiac output (CO) and increased workload and oxygen requirements of the myocardium precipitate HF due to left ventricle dysfunction.

When caring for elderly patients with hypertension, which information should the nurse consider when planning care (select all that apply.)? a. Systolic blood pressure increases with aging. b. Blood pressures should be maintained near 120/80 mm Hg. c. White coat syndrome is prevalent in elderly patients. d. Volume depletion contributes to orthostatic hypotension. e. Blood pressure drops 1 hour postprandially in many older patients. f. Older patients will require higher doses of antihypertensive medications.

a, c, d, e

A patient is admitted to the hospital in hypertensive emergency (BP 244/142 mm Hg). Sodium nitroprusside is started to treat the elevated BP. Which management strategy(ies) would be most appropriate for this patient (select all that apply)? a. Measuring hourly urine output b. Decreasing the MAP by 50% within the first hour c. Continuous BP monitoring with an arterial line d. Maintaining bed rest and providing tranquilizers to lower the BP e. Assessing the patient for signs and symptoms of heart failure and changes in mental status

a, c, e

Which BP-regulating mechanism(s) can result in the development of hypertension if defective (select all that apply)? a. Release of norepinephrine b. Secretion of prostaglandins c. Stimulation of the sympathetic nervous system d. Stimulation of the parasympathetic nervous system e. Activation of the renin-angiotensin-aldosterone system

a, c, e

Which BP-regulating mechanism(s) can result in the development of hypertension if defective (select all that apply)? a. Release of norepinephrine b. Secretion of prostaglandins c. Stimulation of the sympathetic nervous system d. Stimulation of the parasympathetic nervous system e. Activation of the renin-angiotensin-aldosterone system

a, c, e

The patient has heart failure (HF) with an ejection fraction of less than 40%. What core measures should the nurse expect to include in the plan of care for this patient (select all that apply)? a. Left ventricular function is documented. b. Controlling dysrhythmias will eliminate HF. c. Prescription for digoxin (Lanoxin) at discharge d. Prescription for angiotensin-converting enzyme (ACE) inhibitor at discharge e. Education materials about activity, medications, weight monitoring, and what to do if symptoms worsen

a, d, & e The Joint Commission has identified these three core measures for heart failure patients. Although controlling dysrhythmias will improve CO and workload, it will not eliminate HF. Prescribing digoxin for all HF patients is no longer done because there are newer effective drugs and digoxin toxicity occurs easily related to electrolyte levels and the therapeutic range must be maintained.

A patient is admitted to the hospital in hypertensive emergency (BP 244/142 mmHg). Sodium nitroprusside is started to treat the elevated BP. Which management strategy(ies) would be appropriate for this patient (select all that apply)? a. Measuring hourly urine output b. Decreasing the MAP by 50% within the first hour c. Continuous BP monitoring with an intraarterial line d. Maintaining bed rest and providing tranquilizers to lower the BP e. Assessing the patient for signs and symptoms of heart failure and changes in mental status

a,c, & e

What are nonmodifiable risk factors for primary hypertension (select all that apply)? a. Age b. Obesity c. Gender d. Genetic link e. Ethnicity

a,c,d,e Hypertension progresses with increasing age. It is more prevalent in men up to age 45 and above the age of 64 in women. African Americans have a higher incidence of hypertension than do white Americans. Children and siblings of patients with hypertension should be screened and taught about healthy lifestyles.

A 78-year-old patient is admitted with a BP of 180/98 mm Hg. Which age-related physical changes may contribute to this patient's hypertension (select all that apply)? a. Decreased renal function b. Increased adrenergic receptor sensitivity c. Increased baroreceptor reflexes d. Increased collagen and stiffness of the myocardium e. Increased peripheral vascular resistance f. Loss of elasticity in large arteries from arteriosclerosis

a,d,e,f The age-related changes that contribute to hypertension include decreased renal function, increased peripheral vascular resistance, increased collagen and stiffness of the myocardium, and decreased elasticity in large arteries from arteriosclerosis.

The nurse teaches a patient with hypertension that uncontrolled hypertension may damage organs in the body primarily by which mechanism? a. Hypertension promotes atherosclerosis and damage to the walls of the arteries. b. Hypertension causes direct pressure on organs, resulting in necrosis and replacement of cells with scar tissue. c. Hypertension causes thickening of the capillary membranes, leading to hypoxia of organ systems. d. Hypertension increases blood viscosity, which contributes to intravascular coagulation and tissue necrosis distal to occlusions.

a.

The patient with chronic heart failure is being discharged with a diuretic, a renin-angiotensin-aldosterone system (RAAS) inhibitor, and a β-adrenergic blocker. When received from the pharmacy, which medication should not be included for this patient? a. Dopamine b. Losartan (Cozaar) c. Carvedilol (Coreg) d. Hydrochlorothiazide

a. Dopamine is a β-adrenergic agonist that is a positive inotrope given IV, not orally, and used for acute HF. Losartan (Cozaar) is an angiotensin II receptor blocker used for patients who do not tolerate angiotensin- converting enzyme (ACE) inhibitors. Carvedilol (Coreg) is the β-adrenergic blocker that blocks the sympathetic nervous system's negative effects on the failing heart. Hydrochlorothiazide is the diuretic.

Which initial physical assessment finding would the nurse expect to be present in a patient with acute left-sided heart failure? a. Bubbling crackles and tachycardia. b. Hepatosplenomegaly and tachypnea. c. Peripheral edema and cool, diaphoretic skin. d. Frothy, blood-tinged sputum and distended jugular veins.

a. Early clinical manifestations of acute left-sided HF are those of interstitial edema, with bubbling crackles and tachycardia, as well as tachypnea. Later frothy, blood- tinged sputum; severe dyspnea; and orthopnea develop with alveolar edema. Severe tachycardia and cool, clammy skin are present as a result of stimulation of the SNS from hypoxemia. Systemic edema reflected by jugular vein distention, peripheral edema, and hepatosplenomegaly are characteristic of right-sided heart failure.

What is the patient with primary hypertension likely to report? a. No symptoms b. Dyspnea on exertion c. Cardiac palpitations d. Dizziness and vertigo

a. Hypertension is often asymptomatic, especially if it is mild or moderate, and has been called the "silent killer."

The nurse plans long-term goals for the patient who has had a heart transplant with the knowledge that what is the most common cause of death in heart transplant patients during the first year? a. Infection b. Heart failure c. Embolization d. Malignant conditions

a. In the first year after transplant, with the need for long-term immunosuppressant therapy to prevent rejection, the patient with a transplant is at high risk for infection, a leading cause of death in transplant patients. Acute rejection episodes may also cause death in patients with transplants, but many can be treated successfully with augmented immunosuppressive therapy. Malignancies occur in patients with organ transplants after taking immunosuppressants for a number of years.

During treatment of a patient with a BP of 222/148 mm Hg and confusion, nausea, and vomiting, the nurse initially titrates the medications to achieve which goal? a. Decrease the mean arterial pressure (MAP) to 129 mm Hg b. Lower the BP to the patient's normal within the second to third hour c. Reduce the systolic BP (SBP) to 158 mm Hg and the diastolic BP (DBP) to 111 mm Hg within the first 2 hours d. Decrease the SBP to 160 mm Hg and the DBP to between 100 and 110 mm Hg as quickly as possible

a. Initially the treatment goal in hypertensive emergencies is to reduce the mean arterial pressure (MAP) by no more than 20% to 25% in the first hour, with further gradual reduction over the next 24 hours. In this case the MAP is 172, so decreasing it by 25% equals 129. MAP = [(2 x diastolic)+systolic] / 3

The health care provider prescribes spironolactone (Aldactone) for the patient with chronic heart failure. What diet modifications related to the use of this drug should the nurse include in the patient teaching? a. Decrease both sodium and potassium intake. b. Increase calcium intake and decrease sodium intake. c. Decrease sodium intake and increase potassium intake. d. Decrease sodium intake by using salt substitutes for seasoning.

a. Spironolactone is a potassium-sparing diuretic, and when it is the only diuretic used in the treatment of heart failure, moderate to low levels of potassium intake should be maintained to prevent development of hyperkalemia. Sodium intake is usually reduced to at least 2400 mg/day in patients with heart failure, but salt substitutes cannot be freely used because many contain high concentrations of potassium. Calcium intake is not increased.

10] A 60-year-old male client comes into the emergency department with a complaint of crushing substernal chest pain that radiates to his shoulder and left arm. The admitting diagnosis is acute myocardial infarction (MI). Immediate admission orders include oxygen by nasal cannula at 4L/minute, blood work, a chest radiograph, a 12-lead electrocardiogram (ECG), and 2mg of morphine sulfate given IV. The nurse should first: a. Administer the morphine b. Obtain a 12-lead ECG c. Obtain the blood work d. Order the chest radiograph

a. Although obtaining the ECG, chest radiograph, and blood work are all important, the nurse's priority action should be to relieve the crushing chest pain. Therefore, administering morphine sulfate is the priority action.

A 42-year-old man has been diagnosed with primary hypertension with an average BP of 162/92 mm Hg on three consecutive clinic visits. What are four priority lifestyle modifications that should be explored in the initial treatment of the patient? a. b. c. d.

a. Dietary modifications to restrict sodium, cholesterol, and saturated fat; maintain intake of potassium, calcium, and magnesium; and promote weight reduction if overweight b. Daily moderate-intensity physical activity for at least 30 minutes on most days of the week c. Cessation of smoking (if a smoker) d. Moderation or cessation of alcohol intake; usually medications and monitor BP at home. Also, psychosocial risk factors must be addressed.

A patient with newly discovered high BP has an average reading of 158/98 mmHg after 3 months of exercise and diet modifications. Which management strategy will be a priority for this patient? a. Medication will be required because the BP is still not at goal b. BP monitoring should continue for another 3 months to confirm a diagnosis of hypertension c. Lifestyle changes are less important, since they were not effective, and medications will be started d. More vigorous changes in the patient's lifestyle are needed for a longer time before starting medications

a. Medication will be required because the BP is still not at goal

A patient with a recent diagnosis of heart failure has been prescribed furosemide (Lasix) in an effort to physiologically do what for the patient? a. Reduce preload. b. Decrease afterload. c. Increase contractility. d. Promote vasodilation.

a. Reduce preload. Diuretics such as furosemide are used in the treatment of HF to mobilize edematous fluid, reduce pulmonary venous pressure, and reduce preload. They do not directly influence afterload, contractility, or vessel tone.

Priority Decision: The nurse reviews the following vital signs recorded by an unlicensed assistive personnel (UAP) on a patient with acute decompensated heart failure (ADHF): BP 98/60, HR 102 bpm, RR 24, temp 98.2° F (36.7° C), SpO2 84% on 2 L/min via nasal cannula. a. Which of these findings is of highest priority? b. What should the nurse do next?

a. SpO2 of 84% on 2 L/min via nasal cannula indicates impaired oxygen saturation. The patient is having trouble with gas exchange. Airway and breathing are the priority (follow ABCs). b. The nurse should place the patient in high Fowler's position, assess the patient immediately, recheck SpO2, auscultate breath sounds, assess level of consciousness (LOC), check the oxygen connection and rate setting (2 L/min), and talk with the patient about her or his breathing.

The patient with chronic heart failure is being discharged from the hospital. What information should the nurse emphasize in the patient's discharge teaching to prevent progression of the disease to ADHF? a. Take medications as prescribed. b. Use oxygen when feeling short of breath. c. Only ask the physician's office questions. d. Encourage most activity in the morning when rested.

a. Take medications as prescribed. The goal for the patient with chronic HF is to avoid exacerbations and hospitalization. Taking the medications as prescribed along with nondrug therapies such as alternating activity with rest will help the patient meet this goal. If the patient needs to use oxygen at home, it will probably be used all the time or with activity to prevent respiratory acidosis. Many HF patients are monitored by a care manager or in a transitional program to assess the patient for medication effectiveness and monitor for patient deterioration and encourage the patient. This nurse manager can be asked questions or can contact the health care provider if there is evidence of worsening HF.

The nurse recognizes that primary manifestations of systolic failure include: a. decreased EF and increased PAWP b. decreased PAWP and increased EF. c. decreased pulmonary hypertension associated with normal EF d. decreased afterload and decreased left ventricular end-diastolic pressure

a. decreased EF and increased PAWP Rationale: Systolic heart failure results in systolic failure in the left ventricle (LV). The LV loses its ability to generate enough pressure to eject blood forward through the aorta. This results in increased pulmonary artery wedge pressure (PAWP). The hallmark of systolic failure is a decrease in the left ventricular ejection fraction (EF).

A patient with chronic heart failure has atrial fibrillation and a left ventricular ejection fraction of 18%. To decrease the risk of complications form these conditions, what drug does the nurse anticipate giving?

anti-coagulants

What is most organ damage in hypertension related to? a. Increased fluid pressure exerted against organ tissue b. Atherosclerotic changes in vessels that supply the organs c. Erosion and thinning of blood vessels from constant pressure d. Increased hydrostatic pressure causing leakage of plasma into organ interstitial spaces

b

When teaching a patient about dietary management of stage 1 hypertension, which instruction is most appropriate? a. Restrict all caffeine. b. Restrict sodium intake. c. Increase protein intake. d. Use calcium supplements.

b

While obtaining subjective assessment data from a patient with hypertension, the nurse recognizes that a modifiable risk factor for the development of hypertension is a. a low-calcium diet. b. excessive alcohol intake. c. a family history of hypertension. d. consumption of a high-protein diet.

b

While obtaining subjective assessment date from a patient with hypertension, the nurse recognizes that a modifiable risk factor for the development of hypertension is: a. a low-calcium diet b. excessive alcohol consumption c. a family history of hypertension d. consumption of a high-protein diet

b. excessive alcohol consumption

The nurse is caring for a patient admitted with a history of hypertension. The patient's medication history includes hydrochlorothiazide (Hydrodiuril) daily for the past 10 years. Which parameter would indicate the optimal intended effect of this drug therapy? a) Weight loss of 2 lb b) Blood pressure 128/86 c) Absence of ankle edema d) Output of 600 mL per 8 hours

b) Blood pressure 128/86 Hydrochlorothiazide may be used alone as monotherapy to manage hypertension or in combination with other medications if not effective alone. After the first few weeks of therapy, the diuretic effect diminishes, but the antihypertensive effect remains. Since the patient has been taking this medication for 10 years, the most direct measurement of its intended effect would be the blood pressure.

When teaching a patient about dietary management of stage 1 hypertension, which instruction is most appropriate? a) Restrict all caffeine. b) Restrict sodium intake. c) Increase protein intake. d) Use calcium supplements.

b) Restrict sodium intake. The patient should decrease intake of sodium. This will help to control hypertension, which can be aggravated by excessive salt intake, which in turn leads to fluid retention. Caffeine and protein intake do not affect hypertension. Calcium supplements are not recommended to lower BP.

When providing dietary instruction to a patient with hypertension, the nurse would advise the patient to restrict intake of which meat? a) Broiled fish b) Roasted duck c) Roasted turkey d) Baked chicken breast

b) Roasted duck Roasted duck is high in fat, which should be avoided by the patient with hypertension. Weight loss may slow the progress of atherosclerosis and overall CVD risk. The other meats are lower in fat and are therefore acceptable in the diet.

When assessing the patient for orthostatic hypotension, after taking the blood pressure (BP) and pulse (P) in the supine position, what should the nurse do next? a) Repeat BP and P in this position. b) Take BP and P with patient sitting. c) Record the BP and P measurements. d) Take BP and P with patient standing.

b) Take BP and P with patient sitting. When assessing for orthostatic changes in BP after measuring BP in the supine position, the patient is placed in a sitting position and BP is measured within 1 to 2 minutes and then repositioned to the standing position with BP measured again, within 1 to 2 minutes. The results are then recorded with a decrease of 20 mm Hg or more in SBP, a decrease of 10 mm Hg or more in DBP, and/or an increase in pulse of greater than or equal to 20 beats/minute from supine to standing indicating orthostatic hypotension.

Patients with a heart transplantation are at risk for which complications in the first year after transplantation (select all that apply)? a. cancer b. infection c. rejection d. vasculopathy e. sudden cardiac death

b, c, & e Rationale: A variety of complications can occur after heart transplantation. In the first year after transplantation, the major causes of death are acute rejection and infection. Heart transplant recipients also are at risk for sudden cardiac death. Later, malignancy (especially lymphoma) and cardiac vasculopathy (accelerated CAD) are major causes of death.

In the patient with heart failure, which medications or treatments require careful monitoring of the patient's serum potassium level to prevent further cardiac dysfunction (select all that apply)? a. Enalapril (Vasotec) b. Furosemide (Lasix) c. Nesiritide (Natrecor) d. Spironolactone (Aldactone) e. Metoprolol CR/XL (Toprol XL)

b, d. Furosemide is a diuretic that eliminates potassium and spironolactone is a potassium-sparing diuretic that retains potassium. The other treatments and medications are used for patients with HF, but they do not directly affect serum potassium levels.

Which classification of drugs used to treat hypertension prevents the action of angiotensin II and promotes increased salt and water excretion? a. Thiazide diuretics b. Angiotensin II receptor blockers (ARBs) c. Direct vasodilators d. Angiotensin-converting enzyme (ACE) inhibitors

b. Angiotensin II receptor blockers (ARBs) prevent the action of angiotensin II and produce vasodilation and increased salt and water excretion.

Priority Decision: A patient with chronic heart failure is treated with hydrochlorothiazide, digoxin, and lisinopril. To prevent the risk of digitalis toxicity with these drugs, what is most important that the nurse monitor for this patient? a. Heart rate (HR) b. Potassium levels c. Blood pressure (BP) d. Gastrointestinal function

b. Hypokalemia is one of the most common causes of digitalis toxicity because low serum potassium levels enhance ectopic pacemaker activity. When a patient is receiving potassium-losing diuretics, such as hydrochlorothiazide or furosemide, it is essential to monitor the patient's serum potassium levels to prevent digitalis toxicity. Monitoring the heart rate (HR) assesses for complications related to digoxin but does not prevent toxicity.

What is the pathophysiologic mechanism that results in the pulmonary edema of left-sided heart failure? a. Increased right ventricular preload. b. Increased pulmonary hydrostatic pressure. c. Impaired alveolar oxygen and carbon dioxide exchange. d. Increased lymphatic flow of pulmonary extravascular fluid

b. In left-sided HF, blood backs up into the pulmonary veins and capillaries. This increased hydrostatic pressure in the vessels causes fluid to move out of the vessels and into the pulmonary interstitial space. When increased lymphatic flow cannot remove enough fluid from the interstitial space, fluid moves into the alveoli, resulting in pulmonary edema and impaired alveolar oxygen and carbon dioxide exchange. Initially the right side of the heart is not involved.

A patient with chronic heart failure has atrial fibrillation and a left ventricular ejection fraction (LVEF) of 18%. To decrease the risk of complications from these conditions, what drug does the nurse anticipate giving? a. Diuretic b. Anticoagulant c. β-Adrenergic blocker d. Potassium supplement

b. Thrombus formation occurs in the heart when the chambers do not contract normally and empty completely. Both atrial fibrillation and very low left ventricular output (LVEF <20%) lead to thrombus formation, which is treated with anticoagulants to prevent the release of emboli into the circulation as well as antidysrhythmics or cardioversion to control atrial fibrillation.

The unit is very busy and short staffed. What could be delegated to the unlicensed assistive personnel (UAP)? a. Administer antihypertensive medications to stable patients. b. Obtain orthostatic blood pressure (BP) readings for older patients. c. Check BP readings for the patient receiving IV enalapril (Vasotec). d. Teach about home BP monitoring and use of automatic BP monitoring equipment

b. Unlicensed assistive personnel (UAP) may check postural changes in BP as directed. The licensed practical nurse (LPN) may administer antihypertensive medications to stable patients. The RN must monitor the patient receiving IV enalapril (Vasotec), as he or she is in a hypertensive crisis. The RN must also do the teaching related to home BP monitoring.

7] The nurse has completed an assessment on a client with a decreased cardiac output. Which findings should receive highest priority? a. BP 110/62, atrial fibrillation with HR 82, bibasilar crackles b. Confusion, urine output 15mL over the last 2 hours, orthopnea. c. SpO2 92 on 2 liters nasal cannula, respirations 20, 1+ edema of lower extremities. d. Weight gain of 1kg in 3 days, BP 130/80, mild dyspnea with exercise.

b. A low urine output and confusion are signs of decreased tissue perfusion. Orthopnea is a sign of left-sided heart failure. Crackles, edema and weight gain should be monitored closely, but the levels are not as high a priority. With atrial fibrillation there is a loss of atrial kick, but the blood pressure and heart rate are stable

A patient with a diagnosis of heart failure has been started on a nitroglycerin patch by his primary care provider. What should this patient be taught to avoid? a. High-potassium foods b. Drugs to treat erectile dysfunction c. Nonsteroidal antiinflammatory drugs d. Over-the-counter H2 -receptor blockers

b. Drugs to treat erectile dysfunction The use of erectile drugs concurrent with nitrates creates a risk of severe hypotension and possibly death. High-potassium foods, NSAIDs, and H2-receptor blockers do not pose a risk in combination with nitrates.

20] Which of the following is an expected outcome when a client is receiving an IV administration of furosemide? a. Increased blood pressure b. Increased urine output c. Decreased pain d. Decreased premature ventricular contractions

b. Furosemide is a loop diuretic that acts to increase urine output. Furosemide does not increase blood pressure, decrease pain, or decrease arrhythmias

What should the nurse recognize as an indication for the use of dopamine (Intropin) in the care of a patient with heart failure? a. Acute anxiety b. Hypotension and tachycardia c. Peripheral edema and weight gain d. Paroxysmal nocturnal dyspnea (PND)

b. Hypotension and tachycardia Dopamine is a β-adrenergic agonist whose inotropic action is used for treatment of severe heart failure accompanied by hemodynamic instability. Such a state may be indicated by tachycardia accompanied by hypotension. PND, anxiety, edema, and weight gain are common signs and symptoms of heart failure, but these do not necessarily warrant the use of dopamine.

40] Which of the following is not a risk factor for the development of atherosclerosis? a. Family history of early heart attack b. Late onset of puberty c. Total blood cholesterol level greater than 220 mg/dL d. Elevated fasting blood glucose concentration

b. Late onset of puberty is not generally considered to be a risk factor for the development of atherosclerosis. Risk factors for atherosclerosis include family history of atherosclerosis, cigarette smoking, hypertension, high blood cholesterol level, male gender, diabetes mellitus, obesity, and physical inactivity

The nurse is preparing to administer digoxin to a patient with heart failure. In preparation, laboratory results are reviewed with the following findings: sodium 139 mEq/L, potassium 5.6 mEq/L, chloride 103 mEq/L, and glucose 106 mg/dL. What should the nurse do next? a. Withhold the daily dose until the following day. b. Withhold the dose and report the potassium level. c. Give the digoxin with a salty snack, such as crackers. d. Give the digoxin with extra fluids to dilute the sodium level.

b. Withhold the dose and report the potassium level. The normal potassium level is 3.5 to 5.0 mEq/L. The patient is hyperkalemic, which makes the patient more prone to digoxin toxicity. For this reason, the nurse should withhold the dose and report the potassium level. The physician may order the digoxin to be given once the potassium level has been treated and decreases to within normal range.

An elderly client has been diagnosed with chronic heart failure. He is prescribed an ACE inhibitor to treat the symptoms and improve his quality of life. This drug will alleviate the client's symptoms of heart failure by:

blocking the conversion of angiotensin I to angiotensin II

An elderly client has been diagnosed with chronic heart failure. He is prescribed an ACE inhibitor to treat the symptoms and improve his quality of life. This drug will alleviate the client's symptoms of heart failure by:

blocking the conversion of angiotensin I to angiotensin II.

atropine

blocks vagal effects andi ncreases heart rate

The nurse teaches a 28-yr-old man newly diagnosed with hypertension about lifestyle modifications to reduce his blood pressure. Which patient statement requires reinforcement of teaching? a. "I will avoid adding salt to my food during or after cooking." b. "If I lose weight, I might not need to continue taking medications." c. "I can lower my blood pressure by switching to smokeless tobacco." d. "Diet changes can be as effective as taking blood pressure medications.

c

How is secondary hypertension differentiated from primary hypertension? a. Has a more gradual onset than primary hypertension b. Does not cause the target organ damage that occurs with primary hypertension c. Has a specific cause, such as renal disease, that often can be treated by medicine or surgery d. Is caused by age-related changes in BP regulatory mechanisms in people over 65 years of age

c Secondary hypertension has an underlying cause that can often be treated, in contrast to primary or essential hypertension,which has no single known cause.

You are caring for a patient with ADHF who is receiving IV dobutamine (Dobutrex). You know that this drug is ordered because it (select all that apply): a. incerases SVR b. produces diuresis c. improves contractility d. dilates renal blood vessels e. works on the B1-receptors in the heart.

c & e Rationale: Dobutamine (Dobutrex) has a positive chronotropic effect and increases heart rate and improves contractility. It is a selective β-adrenergic agonist and works primarily on the β1-adrenergic receptors in the heart. It is frequently used in the short-term management of acute decompensated heart failure (ADHF).

The nurse is caring for a patient admitted with chronic obstructive pulmonary disease (COPD), angina, and hypertension. Before administering the prescribed daily dose of atenolol 100 mg PO, the nurse assesses the patient carefully. Which adverse effect is this patient at risk for, given the patient's health history? a) Hypocapnia b) Tachycardia c) Bronchospasm d) Nausea and vomiting

c) Bronchospasm Atenolol is a cardioselective β1-adrenergic blocker that reduces blood pressure and could affect the β2-receptors in the lungs with larger doses or with drug accumulation. Although the risk of bronchospasm is less with cardioselective β-blockers than nonselective β-blockers, atenolol should be used cautiously in patients with COPD.

The patient has chronic hypertension. Today she has gone to the ED, and her blood pressure has risen to 200/140. What is the priority assessment for the nurse to make? a) Is the patient pregnant? b) Does the patient need to urinate? c) Does the patient have a headache or confusion? d) Is the patient taking antiseizure medications as prescribed?

c) Does the patient have a headache or confusion? The nurse's priority assessments include neurologic deficits, retinal damage, heart failure, pulmonary edema, and renal failure. The headache or confusion could be seen with hypertensive encephalopathy from increased cerebral capillary permeability leading to cerebral edema. Pregnancy can lead to secondary hypertension. Needing to urinate and taking antiseizure medication do not support a hypertensive emergency.

When teaching how lisinopril (Zestril) will help lower the patient's blood pressure, which mechanism of action should the nurse use to explain it? a) Blocks β-adrenergic effects. b) Relaxes arterial and venous smooth muscle. c) Inhibits conversion of angiotensin I to angiotensin II. d) Reduces sympathetic outflow from central nervous system.

c) Inhibits conversion of angiotensin I to angiotensin II. Lisinopril is an angiotensin-converting enzyme (ACE) inhibitor that inhibits the conversion of angiotensin I to angiotensin II, which reduces angiotensin II-mediated vasoconstriction and sodium and water retention. Beta blockers result in vasodilation and decreased heart rate. Direct vasodilators relax arterial and venous smooth muscle. Central acting α-adrenergic antagonists reduce sympathetic outflow from the CNS to produce vasodilation and decreased SVR and BP.

In caring for a patient admitted with poorly controlled hypertension, which laboratory test result should the nurse understand as indicating the presence of target organ damage? a) BUN of 15 mg/dL b) Serum uric acid of 3.8 mg/dL c) Serum creatinine of 2.6 mg/dL d) Serum potassium of 3.5 mEq/L

c) Serum creatinine of 2.6 mg/dL The normal serum creatinine level is 0.6-1.3 mg/dL. This elevated level indicates target organ damage to the kidneys. The other lab results are within normal limits.

The nurse is teaching a women's group about prevention of hypertension. What information should be included in the teaching for all the women (select all that apply)? a) Lose weight. b) Limit nuts and seeds. c) Limit sodium and fat intake. d) Increase fruits and vegetables. e) Exercise 30 minutes most days.

c, d, & e Primary prevention of hypertension is to make lifestyle modifications that prevent or delay the increase in BP. Along with exercise for 30 minutes on most days, the DASH eating plan is a healthy way to lower BP by limiting sodium and fat intake, increasing fruits and vegetables, and increasing nutrients that are associated with lowering BP. Nuts and seeds and dried beans are used for protein intake. Weight loss may or may not be necessary for the individual.

The nurse is teaching a women's group about prevention of hypertension. What information should be included in the teaching for all the women (select all that apply)? a. Lose weight. b. Limit nuts and seeds. c. Limit sodium and fat intake. d. Increase fruits and vegetables. e. Exercise 30 minutes most days.

c, d, e

What describes the action of the natriuretic peptides and nitric oxide in their counterregulatory processes in response to heart failure (HF)? a. Excretion of potassium b. Increased release of ADH c. Vasodilation and decreased blood pressure (BP) d. Decreased glomerular filtration rate and edema

c. Both the natriuretic peptides and nitric oxide contribute to vasodilation, decreased BP, and decreased afterload. The natriuretic peptides also increase excretion of sodium by increasing glomerular filtration rate and diuresis (renal effects) as well as interfere with ADH release and inhibit aldosterone and renin secretion (hormonal effects).

Which manifestation is an indication that a patient is having a hypertensive emergency? a. Symptoms of a stroke with an elevated BP b. A systolic BP >200 mm Hg and a diastolic BP >120 mm Hg c. A sudden rise in BP accompanied by neurologic impairment d. A severe elevation of BP that occurs over several days or weeks

c. Hypertensive emergency, a type of hypertensive crisis, is a situation that develops over hours or days in which a patient's BP is severely elevated with evidence of acute target organ disease (e.g., cerebrovascular, cardiovascular, renal, or retinal). The neurologic manifestations are often similar to the presentation of a stroke but do not show the focal or lateralizing symptoms of stroke.

Priority Decision: A patient is admitted to the emergency department with ADHF. Which IV medication would the nurse expect to administer first? a. Digoxin (Lanoxin) b. Morphine sulfate c. Nesiritide (Natrecor) d. Bumetanide (Bumex)

c. Nesiritide (Natrecor) is a recombinant form of a natriuretic peptide that decreases preload and afterload by reducing pulmonary artery wedge pressure (PAWP) and systolic BP, which decreases the workload of the heart for short-term emergency treatment of acute decompensated heart failure (ADHF). Digoxin requires a loading dose and time to work, so it is not recommended for emergency treatment of ADHF. Morphine sulfate relieves dyspnea but has more adverse effects. Bumetanide (Bumex) will decrease fluid volume but also will decrease potassium levels and activate the sympathetic nervous system and RAAS, which can exacerbate HF symptoms.

A 2400-mg sodium diet is prescribed for a patient with chronic heart failure. The nurse recognizes that additional teaching is necessary when the patient makes which statement? a. "I should limit my milk intake to 2 cups a day." b. "I can eat fresh fruits and vegetables without worrying about sodium content." c. "I can eat most foods as long as I do not add salt when cooking or at the table." d. "I need to read the labels on prepared foods and medicines for their sodium content."

c. Not adding salt to foods will not eliminate enough sodium for the 2400-mg sodium diet. All foods that are high in sodium should be eliminated in a 2400-mg sodium diet, in addition to the elimination of salt during cooking. Examples include obviously salted foods as well as unexpected sodium sources that are identified by reading the label of prepared foods and medicines.

The nurse determines that treatment of heart failure has been successful when the patient experiences a. weight loss and diuresis. b. warm skin and less fatigue. c. clear lung sounds and decreased HR. d. absence of chest pain and improved level of consciousness (LOC).

c. Successful treatment of heart failure is indicated by an absence of symptoms of pulmonary edema and hypoxemia, such as clear lung sounds and a normal HR. Weight loss and diuresis, warm skin, less fatigue, and improved LOC may occur without resolution of pulmonary symptoms. Chest pain is not a common finding in heart failure unless coronary artery perfusion is impaired.

The patient who is being admitted has had a history of uncontrolled hypertension. High SVR is most likely to cause damage to which organ? a. Brain b. Retina c. Heart d. Kidney

c. The increased systemic vascular resistance (SVR) of hypertension directly increases the workload of the heart and heart failure occurs when the heart can no longer pump effectively against the increased resistance.

Which diagnostic test is most useful in differentiating dyspnea related to pulmonary effects of heart failure from dyspnea related to pulmonary disease? a. Exercise stress testing. b. Cardiac catheterization. c. B-type natriuretic peptide (BNP) levels. d. Determination of blood urea nitrogen (BUN)

c. b-type natriuretic peptide (BNP) is released from the ventricles in response to increased blood volume in the heart and is a good marker for heart failure. If BNP is elevated, shortness of breath is due to heart failure; if BNP is normal, dyspnea is due to pulmonary disease. BNP opposes the actions of the RAAS, resulting in vasodilation and reduction in blood volume. Exercise stress testing and cardiac catheterization are more important tests to diagnose coronary artery disease, and although the blood urea nitrogen (BUN) may be elevated in heart failure, it is a reflection of decreased renal perfusion. (See Table 31-6.)

25] If a client displays risk factors for coronary artery disease, such as smoking cigarettes, eating a diet high in saturated fat, or leading a sedentary lifestyle, techniques of behavior modification may be used to help the client change the behavior. The nurse can best reinforce new adaptive behaviors by: a. Explaining how the old behavior leads to poor health b. Withholding praise until the new behavior is well established c. Rewarding the client whenever the acceptable behavior is performed d. Instilling mild fear into the client to extinguish the behavior

c. A basic principle of behavior modification is that behavior that is learned and continued is behavior that has been rewarded. Other reinforcement techniques have not been found to be as effective as reward.

What is the priority assessment by the nurse caring for a patient receiving IV nesiritide (Natrecor) to treat heart failure? a. Urine output b. Lung sounds c. Blood pressure d. Respiratory rate

c. Blood pressure Although all identified assessments are appropriate for a patient receiving IV nesiritide, the priority assessment would be monitoring for hypotension, the main adverse effect of nesiritide.

24] A 58-year-old female with a family history of CAD is being seen for her annual physical exam. Fasting lab test results include: Total cholesterol 198; LDL cholesterol 120; HDL cholesterol 58; Triglycerides 148; Blood sugar 102; and C-reactive protein (CRP) 4.2. The health care provider informs the client that she will be started on a statin medication and aspirin. The client asks the nurse why she needs to take these medications. Which is the best response by the nurse? a. "The labs indicate severe hyperlipidemia and the medications will lower your LDL, along with a low-fat diet." b. "The triglycerides are elevated and will not return to normal without these medications." c. "The CRP is elevated indicating inflammation seen in cardiovascular disease, which can be lowered by the medications ordered." d. "The medications are not indicated since your lab values are all normal."

c. CRP is a marker of inflammation and is elevated in the presence of cardiovascular disease. The high sensitivity CRP (hs-CRP) is the blood test for greater accuracy in measuring the CRP to evaluate cardiovascular risk. The family history, post-menopausal age, LDL above optimum levels and elevated CRP place the client at risk of CAD. Statin medications can decrease LDL, whereas statins and aspirin can reduce CRP and decrease the risk of MI and stroke.

41] As an initial step in treating a client with angina, the physician prescribes nitroglycerin tablets, 0.3 mg given sublingually. This drug's principal effects are produced by: a. Antispasmodic effects on the pericardium b. Causing an increased myocardial oxygen demand c. Vasodilation of peripheral vasculature d. Improved conductivity in the myocardium

c. Nitroglycerin produces peripheral vasodilation, which reduces myocardial oxygen consumption and demand. Vasodilation in coronary arteries and collateral vessels may also increase blood flow to the ischemic areas of the heart. Nitroglycerin does not have an effect on pericardial spasticity or conductivity in the myocardium.

36] The client who experiences angina has been told to follow a low-cholesterol diet. Which of the following meals should the nurse tell the client would be best on her low-cholesterol diet? a. Hamburger, salad, and milkshake b. Baked liver, green beans, and coffee c. Spaghetti with tomato sauce, salad, and coffee d. Fried chicken, green beans, and skim milk

c. Pasta, tomato sauce, salad, and coffee would be the best selection for the client following a low-cholesterol diet. Hamburgers, milkshakes, liver, and fried foods tend to be high in cholesterol.

43] Sublingual nitroglycerine tablets begin to work within 1 to 2 minutes. How should the nurse instruct the client to use the drug when chest pain occurs? a. Take one tablet every 2 to 5 minutes until the pains stops b. Take one tablet and rest for 10 minutes. Call the physician if pain persists after 10 minutes c. Take one tablet, then an additional tablet every 5 minutes for a total of three tablets. Call the physician if pain persists after three tablets d. Take one tablet. If pain persists after 5 minutes, take two tablets. If pain still persists 5 minutes later, call the physician

c. The correct protocol for nitroglycerin use involves immediate administration, with subsequent doses taken at 5-minute intervals as needed, for a total dose of three tablets. Sublingual nitroglycerin appears in the bloodstream within 2 to 3 minutes and is metabolized within about 10 minutes.

A compensatory mechanism involved in HF that leads to inappropriate fluid retention and additional workload of the heart is: a. ventricular dilation b. ventricular hypertrophy c. neurohormonal response d. sympathetic nervous system activation

c. neurohormonal response Rationale: The following mechanisms in heart failure lead to inappropriate fluid retention and additional workload of the heart: activation of the renin-angiotensin-aldosterone system (RAAS) cascade and release of antidiuretic hormone from the posterior pituitary gland in response to low cerebral perfusion pressure that results from low cardiac output.

Digoxin

cardiac Glycoside

pulse vtach patient awake

cardioversion, have crash cart ready

________ __________ is the term for right-sided HF that is the result of chronic pulmonary dz.

cor pulmonale

Vfib

chicken scratch looking

The nurse determines that treatment of heart failure has been successful when the patient experiences

clear lung sounds and decreased HR

The pathophysiology of heart failure involves an interaction between decreased pumping ability and the ________ to maintain cardiac output.

compensatory mechanisms

BBBs

conduction delay or block within one of the two main bundle of HIS branches

A priority consideration in the management of the older adult with hypertension is to a. prevent primary hypertension from converting to secondary hypertension. b. recognize that the older adult is less likely to adhere to the drug therapy regimen than a younger adult. c. ensure that the patient receives larger initial doses of antihypertensive drugs because of impaired absorption. d. use careful technique in assessing the BP of the patient because of the possible presence of an auscultatory gap.

d

In teaching a patient with hypertension about controlling the illness, the nurse recognizes that a. all patients with elevated BP require medication. b. obese persons must achieve a normal weight to lower BP. c. it is not necessary to limit salt in the diet if taking a diuretic. d. lifestyle modifications are indicated for all persons with elevated BP.

d

The blood pressure of a 71-year-old patient admitted with pneumonia is 160/70 mm Hg. What is an age-related change that contributes to this finding? a. Stenosis of the heart valves Incorrect b. Decreased adrenergic sensitivity c. Increased parasympathetic activity d. Loss of elasticity in arterial vessels

d

In reviewing medication instructions with a patient being discharged on antihypertensive medications, which statement would be most appropriate for the nurse to make when discussing guanethidine (Ismelin)? a) "A fast heart rate is a side effect to watch for while taking guanethidine." b) "Stop the drug and notify your doctor if you experience any nausea or vomiting." c) "Because this drug may affect the lungs in large doses, it may also help your breathing." d) "Make position changes slowly, especially when rising from lying down to a standing position."

d) "Make position changes slowly, especially when rising from lying down to a standing position." Guanethidine is a peripheral-acting α-adrenergic antagonist and can cause marked orthostatic hypotension. For this reason, the patient should be instructed to rise slowly, especially when moving from a recumbent to a standing position. Support stockings may also be helpful. Tachycardia or lung effects are not evident with guanethidine.

The nurse monitors the patient receiving treatment for acute decompensated heart failure with the knowledge that marked hypotension is most likely to occur with the IV administration of which medication? a. Milrinone b. Furosemide c. Nitroglycerin d. Nitroprusside

d. Although all of these drugs may cause hypotension, nitroprusside is a potent dilator of both arteries and veins and may cause such marked hypotension that an inotropic agent (e.g., dobutamine) administration may be necessary to maintain the BP during its administration. Furosemide may cause hypotension because of diuretic-induced depletion of intravascular fluid volume. Milrinone has a positive inotropic effect in addition to peripheral vasodilation. Nitroglycerin is a vasodilator and can decrease BP but not as severely as nitroprusside. It primarily dilates veins and increases myocardial oxygen supply.

What is included in the correct technique for BP measurements? a. Always take the BP in both arms. b. Position the patient supine for all readings. c. Place the cuff loosely around the upper arm. d. Take readings at least two times at least 1 minute apart.

d. Correct technique in measuring BP includes taking two or more readings at least 1 minute apart. Initially BP measurements should be taken in both arms to detect any differences. If there is a difference, the arm with the higher reading should be used for all subsequent BP readings. The patient may be supine or sitting. The important point is that the arm being used is at the heart level and the cuff needs to fit snugly.

The acronym FACES is used to help teach patients to identify early symptoms of heart failure. What does this acronym mean? a. Frequent activity leads to cough in the elderly and swelling b. Factors of risk: activity, cough, emotional upsets, salt intake c. Follow activity plan, continue exercise, and know signs of problems d. Fatigue, limitation of activities, chest congestion/cough, edema, shortness of breath

d. FACES is used to teach patients to identify early HF symptoms. F = Fatigue; A = Activity limitations; C =Chest congestion/cough; E = Edema; S = Shortness of breath. The other options are not correct.

Which statement by the patient with chronic heart failure should cause the nurse to determine that additional discharge teaching is needed? a. "I will call my health clinic if I wake up breathless at night." b. "I will look for sodium content on labels of foods and over-the-counter medicines." c. "I plan to organize my household tasks so I don't have to constantly go up and down the stairs." d. "I should weigh myself every morning and go on a diet if I gain more than 2 or 3 pounds in 2 days."

d. Further teaching is needed if the patient believes a weight gain of 2 to 3 pounds in 2 days is an indication for dieting. In a patient with heart failure, this type of weight gain reflects fluid retention and is a sign of heart failure that should be reported to the HCP. The other options show patient understanding of the heart failure management teaching (see Table 34-10).

What does the nursing responsibility in the management of the patient with hypertensive urgency often include? a. Monitoring hourly urine output for drug effectiveness b. Titrating IV drug dosages based on BP measurements every 2 to 3 minutes c. Providing continuous electrocardiographic (ECG) monitoring to detect side effects of the drugs d. Instructing the patient to follow up with a health care professional within 24 hours after outpatient treatment

d. Hypertensive urgencies are often treated with oral drugs on an outpatient basis but it is important for the patient to be seen by a health care professional within 24 hours to evaluate the effectiveness of the treatment. Hourly urine measurements, titration of IV drugs, and ECG monitoring are indicated for hypertensive emergencies.

Which medication shows improvement in HF for African American patients for hypertension and angina? a. Captopril b. Nitroglycerin c. Spironolactone (Aldactone) d. Isosorbide dinitrate and hydralazine (BiDil)

d. Isosorbide dinitrate and hydralazine (BiDil) is recommended for use in African American patients with HF to treat hypertension and angina. Captopril is used for hypertension by all patients. Nitroglycerin is used with hydralazine for patients who cannot tolerate RAAS inhibitors (ACE inhibitors or ARBs) for heart failure management. Spironolactone (Aldactone) is used for hypertension.

The nurse assesses the patient with chronic biventricular heart failure for paroxysmal nocturnal dyspnea (PND) by questioning the patient regarding a. the presence of restlessness and confusion. b. frequent awakening to void during the night. c. the presence of swelling in dependent body areas. d. waking in a panic with a feeling of suffocation.

d. Paroxysmal nocturnal dyspnea (PND) is awakening from sleep with a feeling of suffocation and a need to sit up to be able to breathe. Patients learn that sleeping with the upper body elevated on several pillows helps to prevent PND. Behavior changes are seen in late stages of HF. Nocturia occurs with HF as fluid moves back into the vascular system during recumbency, increasing renal blood flow. Dependent edema does not indicate PND.

The evaluation team for cardiac transplantation is evaluating patients. Which patient is most likely to receive the most benefit from a new heart? a. A 24-year-old man with Down syndrome who has received excellent care from parents in their 60s b. A 46-year-old single woman with a limited support system who has alcohol-induced cardiomyopathy c. A 60-year-old man with inoperable coronary artery disease who has not been compliant with lifestyle changes and rehabilitation programs d. A 52-year-old woman with end-stage coronary artery disease who has limited financial resources but is emotionally stable and has strong social support

d. The 52-yr-old woman does not have any contraindications for cardiac transplantation, even though she lacks the indication of adequate financial resources. The 24-yr-old man does not have a current cardiac diagnosis. The postoperative transplant regimen is complex and rigorous, and patients who have not been compliant with other treatments or who may not have the means to understand the care would not be good candidates. A history of drug or alcohol abuse is usually a contraindication to heart transplant.

The nurse is administering a dose of digoxin (Lanoxin) to a patient with heart failure (HF). The nurse would become concerned with the possibility of digitalis toxicity if the patient reported which symptom(s)? a. Muscle aches b. Constipation c. Pounding headache d. Anorexia and nausea

d. Anorexia and nausea Anorexia, nausea, vomiting, blurred or yellow vision, and cardiac dysrhythmias are all signs of digitalis toxicity. The nurse would become concerned and notify the health care provider if the patient exhibited any of these symptoms.

A stable patient with acute decompensated heart failure (ADHF) suddenly becomes dyspneic. Before positioning the patient on the bedside, what should the nurse assess first? a. Urine output b. Heart rhythm c. Breath sounds d. Blood pressure

d. Blood pressure The nurse should evaluate the blood pressure before dangling the patient on the bedside because the blood pressure can decrease as blood pools in the periphery and preload decreases. If the patient's blood pressure is low or marginal, the nurse should put the patient in the semi-Fowler's position and use other measures to improve gas exchange.

Beyond the first year after a heart transplant, the nurse knows that what is a major cause of death? a. Infection b. Acute rejection c. Immunosuppression d. Cardiac vasculopathy

d. Cardiac vasculopathy Beyond the first year after a heart transplant, malignancy (especially lymphoma) and cardiac vasculopathy (accelerated CAD) are the major causes of death. During the first year after transplant, infection and acute rejection are the major causes of death. Immunosuppressive therapy will be used for posttransplant management to prevent rejection and increases the patient's risk of an infection.

A male patient with a long-standing history of heart failure has recently qualified for hospice care. What measure should the nurse now prioritize when providing care for this patient? a. Taper the patient off his current medications. b. Continue education for the patient and his family. c. Pursue experimental therapies or surgical options. d. Choose interventions to promote comfort and prevent suffering.

d. Choose interventions to promote comfort and prevent suffering. The central focus of hospice care is the promotion of comfort and the prevention of suffering. Patient education should continue, but providing comfort is paramount. Medications should be continued unless they are not tolerated. Experimental therapies and surgeries are not commonly used in the care of hospice patients.

31] Crackles heard on lung auscultation indicate which of the following? a. Cyanosis b. Bronchospasm c. Airway narrowing d. Fluid-filled alveoli

d. Crackles are auscultated over fluid-filled alveoli. Crackles heard on lung auscultation do not have to be associated with cyanosis. Bronchospasm and airway narrowing generally are associated with wheezing sounds.

First Degree AV Block

delay in conduction of electrical impulse in the AV node but it still makes it to the ventricles Regular p-wave long P-R interval usually no symptoms but may be caused by lyte imbalance or medicine

After having an MI, the nurse notes the patient has jugular venous distention, gained weight, developed peripheral edema, and has a heart rate of 108/minute. What should the nurse suspect is happening? a. ADHF b. Chronic HF c. Left-sided HF d. Right-sided HF

d. Right-sided HF An MI is a primary cause of heart failure. The jugular venous distention, weight gain, peripheral edema, and increased heart rate are manifestations of right-sided heart failure.

23] The nurse is assessing clients at a health fair. Which client is at greatest risk for coronary artery disease? a. a 32-year-old female with mitral valve prolapse who quit smoking 10 years ago. b. a 43-year-old male with a family history of CAD and cholesterol level of 158 c. A 56-year-old male with an HDL of 60 who takes atorvastatin (Lipitor) d. A 65-year-old female who is obese with an LDL of 188

d. The woman who is 65-years-old, over weight and has an elevated LDL is at greatest risk. Total cholesterol >200, LDL >100, HDL <40 in men, HDL <50 in women, men 45-years and older, women 55-years and older, smoking and obesity increase the risk of CAD. Atorvastatin is a medication to reduce LDL and decrease risk of CAD. The combination of postmenopausal, obesity and high LDL cholesterol places this client at greatest risk.

In teaching a patient with hypertension about controlling the condition, the nurse recognizes that: a. all patients with elevated BP require medication b. obese persons must achieve a normal weight to lower BP c. It is not necessary to limit salt in the diet if taking a diuretic d. lifestyle modifications are indicated for all persons with elevated BP

d. lifestyle modifications are indicated for all persons with elevated BP

A major consideration in the management of the other adult with hypertension is to: a. prevent primary hypertension from converting to secondary hypertension b. recognize that the older adult is less likely to adhere to the drug therapy regimen than a younger adult c. ensure that the patient receives larger initial doses of antihypertensive drugs because of impaired absorption d. use careful technique in assessing the BP of the patient because of the possible presence of ab auscultatory gap

d. use careful technique in assessing the BP of the patient because of the possible presence of ab auscultatory gap

the nurse recognizes that primary manifestations of systolic failure include

decrease EF and increase PAWP

Digitalis

digoxin (Lanoxin) Improves cardiac contractility Increases force of contraction & slows conduction at AV node Decreasing S/S of systolic HF Caution: renal dysfunction (dig. tox risk) Key: monitor for dig. toxicity & bradycardia

Atrial Fibrillation

disorganization of atrial electrical activity from multiple ectopic foci can be paroxysmal or persistent (7 days or more) 350-600 bpm ventricular response of 60-100= controlled, ventricular response >100= uncontrolled results in decreased CO because of ineffective atrial contractions can cause thrombi from blood pooling and lead to strokes treatment: decrease ventricular response, CCB (diltiazem), BB, Digoxin (lanoxin), dronedarone

The action of ANP and BNP most resembles the action of which class of meds: Beta 1 adrenergic receptor antagonists aldosterone diuretics ADH

diuretics

The patient with chronic heart failure is being discharged with a diuretic, a renin-angiotensin-aldosterone system inhibitor, and a Beta-adrenergic blocker. When recieved from pharmacy, which medication should not be included for this patient?

dopamine

Systolic heart failure can be caused by anything that: -Impedes expansion of ventricle such as pericardial effusion & constrictive pericarditis -Conditions that decrease wall thickness while increasing chamber size such as dilated hypertrophic cardiomyopathy -Conditions that delay diastolic relaxationIschemic heart dz -Aging -Conditions that unduly increase heart rate T/F

false These are all true about diastolic HF

Wilbur had a mild MI 20 years ago, but has not been able to control his HTN due to lack of compliance with his meds. For the last several months, he has experienced DOE, bouts of unproductive coughing, and is unable to sleep without being propped up on 2 to 3 pillows. It would be reasonable to suspect right-sided heart failure. T/F

false These are signs/symptoms of LEFT side HF

Loop diuretics

furosemide (Lasix) Inhibit sodium & chloride reabsorption in loop of Henle Usually initial treatment for fluid volume overload in HF Increase K excretion Monitor: electrolyte imbalances (hypokalemia, hyponatremia), renal dysfunction, diuretic resistance, hypotension

pulseless vtach you do..

get defibrillator, call code, CPR

premature arterial complexes

impulse fires before next sinus impulse, no pwave.

chest pain and decreased mentation

in the patient with a dysrhythmia, which assessment indicates decreased cardiac output

What is the pathophysiologic mechanism that results in the pulmonary edema of left-sided heart failure

increase hydrostatic pressure

The nurse plans long-term goals for the patient who has had a heart transplant with the knowledge that what is the most common cause of death in heart transplant patients during the first year?

infection

Second Degree AV block: type 2

intermittent drop in beats that happens randomly R-waves are irregular (dropped QRS complexes) usually occurs in an anterior MI (blocked bundle branch)

Match the etiology/S & S Anaphylactic shock

itching, hives, wheezing severe allergy to peanuts

Angiotensin-Converting Enzyme (ACE) Inhibitors

lisinopril (Prinivil), enapralapril (Vasotec) Promotes vasodilation & diuresis Diminishes heart workload & improves ventricular emptying Decrease aldosterone secretion (decrease BP & afterload) Excretes sodium & fluid--retains potassium Started low & slow Adverse: hypotension, hyperkalemia (potassium retention--D/C if 5.5 or higher), renal dysfunction, dry cough, ANGIOEDEMA

Furosemide

loop diuretic

Reduce Preload & Intravascular volume

loop diuretic lasix Foley catheter to measure - easier than a bedpan Check K+ and BP vasodilators (Nitroglycerin) NTG IV continuous - vasodilate High dose - will dilate both arterial and venous (low dose does JUST venous) position high fowlers (dangle feet) Don't want blood coming back to the heart/lungs

Neurogenic shock

low heart rate and warm, dry skin spinal cord injury

Which of the following is not a likely complication of shock? acute lung injury gastrointestinal ischemia and necrosis acute renal failure malignant hypertension DIC

malignant hypertension

Thiazide diuretics

metolazone (Zaroxolyn), hydrochlorothiazide (HCTZ) Inhibit Na and Cl reabsorption in distal tubules Also increases K excretion Monitor: electrolyte imbalances, renal dysfunction, diuretic resistance, hypotension

Beta-Adrenergic Blocking Agents (Beta-Blockers)

metoprolol (Lopressor), carvedilol (Coreg) Vasodilation, decrease afterload & cardiac workload (block adverse effects of SNS) Recommended: asymptomatic systolic HF (prevent disease progression) Adverse: hypotension, bradycardia, fatigue, depression Started low & slow Caution: hx of bronchospasm (ex asthma)

a patient with chronic HF and atrial fibrillation is treated witha digitalis glycoside and a loop diuretic. To prevent possible complications of this combination of drugs, what does the nurse need to do

monitor serum potassium levels Teach the patient how to take pulse rate

ventricular arrhythmias

more common in ADULTS (cause of ped arrest is usually respiratory)

The nurse monitors receiving treatment for acute decompensated heart failure with the knowledge that marked by hypotension is most likely to occur with the IV administration of which medication?

nitroprusside

afib

no clear p wave, no atrial contraction, irregular ventricular response

Pulseless electrical activity (PEA)

no palpable pulse, but shows electrical activity

third degree block

no regular p wave exist "buried",

Septic shock

overwhelming infection with a gram-negative bacteria

normal sinus rhythm

p wave is present consistent and preceding each QRS complex, heart beat 60-100bpm

second degree AV block type 2

p waves randomly not conduct, usually no lengthening of PR interval before missing QRS

Premature Atrial Contraction (PAC)

p-wave followed by a normal QRS can be caused by caffeine, alcohol, drugs, electrolyte imbalance, COPD usually not treated...maybe BB p-wave can look like its part of the T wave

Wandering Atrial Pacemaker

pacemaker site shifts from the SA node to the AV node Rate is still 60-100 p-waves vary in size/shape P-R intervals are normal to short 1:1

Hypocalcemia can cause these heart rhythms

prolonged QT and prolonged ST interval

Hypovolemic shock

prolonged vomiting or diarrhea burns excessive polyuria from uncontrolled diabetes mellitus acute blood loss

Obstructive shock

pulmonary embolism cardiac tamponade increased right-sided heart pressures with JVD

shockable rhythms

pulseless Vtach and Vfib

Supraventricular tachycardia (SVT)

rapid rhythm over 100 generated outside of the ventricles symptoms: palpitations, lightheadedness, dizziness treatment: BB, CCB, antiarrythmics

Atrial Tachycardia

rate is usually 160-240 beats a type of Supraventricular Tachycardia- generation of impulse outside the SA node can be a result of drug toxicity (digitalis)

atrial flutter

see regular QRS complex, p waves "flutter"

Cardiogenic shock

severe left lateral wall MI end-stage heart failure

Noepinephrine, Dopamine , Dobutamine

shock medications

Third Degree Heart Block

signal is blocked completely multiple escape beats from the ventricles because they are not receiving any impulse from the AV node symptoms: chest pain, dyspnea, syncope

The nurse includes the definition of HF in the teaching plan for the client. An accurate description of the mechanism of HF is that [Hint] there is too much fluid in the heart. the heart cannot get oxygen. the heart is unable to pump effectively. there is too much fluid in the lungs.

the heart is unable to pump effectively.

induce dysrhythmias that may require cardioversion or defibrillation to correct

the nurse plans close monitoring for the patient during electrophysiologic testing because this test

ST segment Elevation

the patient is brought to the ED with acute coronary syndrome. What changes should the nurse expect to see on the ECG is only myocardial injury has occurred?

Mucolytics

thins secretions, tastes and smells very bad used with cystic fibrosis kids, near drowning pts

Hypokalemia and hypomagnesemia can cause this heart rhythm

torsades; can be the result of prolonged QT; often ends in vfib

Anaphylactic shock is directly associated with:

type I hypersensitivity response.

Angiotensin-Receptor Blockers (ARBs)

valsartan (Diovan), losartan (Cozaar) Block angiotensin II to block vasoconstriction Alternative to ACE inhibitors for HF Reduces BP and afterload Relieves S/S of HF & prevents disease progression Monitor: hypotension, hyperkalemia, renal dysfunction

___________ ___________ is another name for distributive shock.

vascular shock

____________ _____________ ___________ are mechanical pumps placed within the thorax to support ventricular pumping.

ventricular assist devices

cannot be defibrillated only CPR

ventricular asystole

Wolff- Parkinson- White (WPW)

ventricular pre-excitation presence of delta wave

vtach

wide complex, patient may or may not have pulse


Related study sets

Intro to Business final- Duplicate of Business Ownership Review

View Set

Lab Simulation 5-2: Test the Efficiency of your Network Network +

View Set

Биохимия 1 (50 тестов укр )

View Set

Nat. Geo. 13 Colonies: Section 1

View Set

Developmental Psychology Chapter 12: Family

View Set